%%%%%%%%%%%%%%%%%%%%%%%%%%%%%%%%%%%%%%%%%%%%%%%%%%%%%%%%%%%%%%%%%%%%%%% % % % Scientific Word Wrap/Unwrap Version 2.5 % % Scientific Word Wrap/Unwrap Version 3.0 % % % % If you are separating the files in this message by hand, you will % % need to identify the file type and place it in the appropriate % % directory. The possible types are: Document, DocAssoc, Other, % % Macro, Style, Graphic, PastedPict, and PlotPict. Extract files % % tagged as Document, DocAssoc, or Other into your TeX source file % % directory. Macro files go into your TeX macros directory. Style % % files are used by Scientific Word and do not need to be extracted. % % Graphic, PastedPict, and PlotPict files should be placed in a % % graphics directory. % % % % Graphic files need to be converted from the text format (this is % % done for e-mail compatability) to the original 8-bit binary format. % % % %%%%%%%%%%%%%%%%%%%%%%%%%%%%%%%%%%%%%%%%%%%%%%%%%%%%%%%%%%%%%%%%%%%%%%% % % % Files included: % % % % "/document/Chap11.tex", Document, 99710, 4/28/2014, 12:49:17, "" % % "/document/webmath.cst", Document, 20031, 5/21/2002, 15:19:10, "" % % "/document/graphics/Image10.gif", ImportPict, 735, 10/28/1998, 14:48:44, ""% % "/document/graphics/Image20.gif", ImportPict, 2987, 10/28/1998, 14:48:44, ""% % "/document/graphics/Image30.gif", ImportPict, 680, 10/28/1998, 14:48:44, ""% % "/document/graphics/Image40.gif", ImportPict, 1269, 10/28/1998, 14:48:44, ""% % "/document/graphics/Image50.gif", ImportPict, 735, 10/28/1998, 14:48:44, ""% % "/document/graphics/Image60.gif", ImportPict, 2815, 10/28/1998, 14:48:44, ""% % "/document/graphics/Image70.gif", ImportPict, 928, 10/28/1998, 14:48:44, ""% % "/document/N4QRHX00.wmf", PlotPict, 13666, 4/11/2012, 16:01:49, "" % % "/document/N4QRHX0F.xvz", PlotPict, 30203, 4/3/2014, 9:31:36, "" % % "/document/graphics/Image80.gif", ImportPict, 2039, 10/28/1998, 14:48:44, ""% % "/document/N4QRHX0D.xvz", PlotPict, 15360, 3/28/2012, 16:15:19, "" % % "/document/graphics/Image90.gif", ImportPict, 1092, 10/28/1998, 14:48:44, ""% % % %%%%%%%%%%%%%%%%%%%%%%%%%%%%%%%%%%%%%%%%%%%%%%%%%%%%%%%%%%%%%%%%%%%%%%% %%%%%%%%%%%%%%%%%%%%%% Start /document/Chap11.tex %%%%%%%%%%%%%%%%%%%%% %% This document created by Scientific Notebook (R) Version 3.0 \documentclass[12pt,thmsa]{article} %%%%%%%%%%%%%%%%%%%%%%%%%%%%%%%%%%%%%%%%%%%%%%%%%%%%%%%%%%%%%%%%%%%%%%%%%%%%%%%%%%%%%%%%%%%%%%%%%%%%%%%%%%%%%%%%%%%%%%%%%%%%%%%%%%%%%%%%%%%%%%%%%%%%%%%%%%%%%%%%%%%%%%%%%%%%%%%%%%%%%%%%%%%%%%%%%%%%%%%%%%%%%%%%%%%%%%%%%%%%%%%%%%%%%%%%%%%%%%%%%%%%%%%%%%%% \usepackage{amsmath} \usepackage{sw20jart} \setcounter{MaxMatrixCols}{10} %TCIDATA{TCIstyle=article/art4.lat,jart,sw20jart} %TCIDATA{OutputFilter=LATEX.DLL} %TCIDATA{Version=5.50.0.2960} %TCIDATA{} %TCIDATA{BibliographyScheme=Manual} %TCIDATA{Created=Mon Aug 19 14:52:24 1996} %TCIDATA{LastRevised=Monday, April 28, 2014 08:49:17} %TCIDATA{} %TCIDATA{Language=American English} %TCIDATA{CSTFile=webmath.cst} %TCIDATA{PageSetup=72,72,72,72,0} %TCIDATA{ComputeDefs= %$u\left( x,t\right) =-3\sin \left( \frac{5\pi x}{2}\right) \cos \left( \frac{% %5\pi t}{8}\right) +23\sin \left( \frac{11\pi x}{2}\right) \cos \frac{11\pi t% %}{8}$ %$y=c_{1}x$ %} %TCIDATA{AllPages= %F=36,\PARA{038

\hfill \thepage} %} \input{tcilatex} \begin{document} \section{Ma 221} \section{\protect\Large BOUNDARY VALUE PROBLEMS} \vspace{1pt} \subsection{Homogeneous Boundary Value Problems} \vspace{1pt} Consider the following problem: \vspace{1pt} \ \vspace{1pt}% \begin{equation*} \left. \begin{array}{c} \text{\textbf{D.E}}\mathbf{.}L\left[ y\right] =a_{0}\left( x\right) y^{\prime \prime }+a_{1}\left( x\right) y^{\prime }+a_{2}\left( x\right) y=0\qquad a\leq x\leq b \\ \qquad \text{B.C. \ }\alpha _{1}y\left( a\right) +\beta _{1}y^{\prime }\left( a\right) =0\qquad \alpha _{1}^{2}+\beta _{1}^{2}\neq 0 \\ \qquad \text{B.C. \ }\alpha _{2}y\left( b\right) +\beta _{2}y^{\prime }\left( b\right) =0\qquad \alpha _{2}^{2}+\beta _{2}^{2}\neq 0% \end{array}% \right\} \qquad \qquad \left( 1\right) \end{equation*} Here $\alpha _{1},\alpha _{2},\beta _{1},$ and $\beta _{2}$ are constants. \vspace{1pt} \begin{example} \begin{equation*} y^{\prime \prime }=0\qquad y^{\prime }(0)=y^{\prime }(1)=0 \end{equation*} \end{example} $\left( \text{Here }\alpha _{1}=\alpha _{2}=0\right) $ \qquad \qquad $\Longrightarrow y=Ax+b\qquad y^{\prime }(x)=A\qquad y^{\prime }(0)=y^{\prime }(1)=A=0$ \qquad \qquad $\Longrightarrow y(x)=b\qquad b$ any constant. The Boundary Value Problem $(1)$ is called linear and homogeneous since if $% u_{1}(x)$\ and $u_{2}(x)$ satisfy it, $\Longrightarrow c_{1}u_{1}(x)+c_{2}u_{2}\left( x\right) $\ also does. \vspace{1pt} \begin{example} \begin{equation*} y^{\prime \prime }-6y^{\prime }+5y=0\text{ \ \ \ \ }y\left( 0\right) =1\text{ \ \ \ }y\left( 2\right) =1 \end{equation*} \end{example} Solution: \ The characteristic equation is% \begin{equation*} r^{2}-6r+5=\left( r-5\right) \left( r-1\right) =0 \end{equation*} so $r=1,5$ \vspace{1pt} Thus% \begin{equation*} y\left( x\right) =c_{1}e^{x}+c_{2}e^{5x} \end{equation*} \begin{eqnarray*} y\left( 0\right) &=&c_{1}+c_{2}=1 \\ y\left( 2\right) &=&c_{1}e^{2}+c_{2}e^{10}=1 \end{eqnarray*} Thus from the first equation $c_{2}=1-c_{1}$ and the second equation becomes% \begin{equation*} c_{1}e^{2}+\left( 1-c_{1}\right) e^{10}=1 \end{equation*} \begin{equation*} c_{1}\left( e^{2}-e^{10}\right) =1-e^{10} \end{equation*} \begin{equation*} c_{1}=\frac{1-e^{10}}{e^{2}-e^{10}} \end{equation*} \begin{equation*} c_{2}=1-\frac{1-e^{10}}{e^{2}-e^{10}}=\frac{1}{e^{2}-e^{10}}\left( e^{2}-1\right) \end{equation*} \begin{equation*} y=\frac{1-e^{10}}{e^{2}-e^{10}}e^{x}+\frac{e^{2}-1}{e^{2}-e^{10}}e^{5x} \end{equation*} SNB check% \begin{eqnarray*} y^{\prime \prime }-6y^{\prime }+5y &=&0 \\ y\left( 0\right) &=&1 \\ y\left( 2\right) &=&1 \end{eqnarray*}% , Exact solution is: $\left\{ \frac{e^{5x}}{e^{2}-e^{10}}\left( e^{2}-1\right) -\frac{e^{x}}{e^{2}-e^{10}}\left( e^{10}-1\right) \right\} \allowbreak $ \ \ \ \ \vspace{1pt} Remark. The homogeneous Boundary Value Problem (B.V.P.) always possesses the solution $y\left( x\right) =0$. \vspace{1pt} Question. When does there exist a nonzero solution to $(1)$? \qquad Let $y_{1}\left( x\right) $ and $y_{2}\left( x\right) $ be two linearly independent solutions of $L\left[ y\right] =0$. $\ \ \Longrightarrow $ $y\left( x\right) =c_{1}y_{1}+c_{2}y_{2}$ is the general solution of the DE. B.C. \ $\Longrightarrow \left. \begin{array}{c} \alpha _{1}y\left( a\right) +\beta _{1}y^{\prime }\left( a\right) =0 \\ \alpha _{2}y\left( b\right) +\beta _{2}y^{\prime }\left( b\right) =0% \end{array}% \right\} \qquad $ and $y\left( x\right) =c_{1}y_{1}+c_{2}y_{2}\Longrightarrow $ \vspace{1pt} \qquad \qquad $c_{1}\left[ \alpha _{1}y_{1}\left( a\right) +\beta _{1}y_{1}^{\prime }\left( a\right) \right] +c_{2}\left[ \alpha _{1}y_{2}\left( a\right) +\beta _{1}y_{2}^{\prime }\left( a\right) \right] =0 $ \qquad \qquad $c_{1}\left[ \alpha _{2}y_{1}\left( b\right) +\beta _{2}y_{1}^{\prime }\left( b\right) \right] +c_{2}\left[ \alpha _{2}y_{2}\left( b\right) +\beta _{2}y_{2}^{\prime }\left( b\right) \right] =0 $. \qquad The above are two equations for $c_{1}$ and $c_{2}$. We want a nontrivial solution. Let $B_{a}\left( u\right) =\alpha _{1}u\left( a\right) +\beta _{1}u^{\prime }\left( a\right) $ and $B_{b}\left( u\right) =\alpha _{2}u\left( b\right) +\beta _{2}u^{\prime }\left( b\right) $. Then the determinant of the coefficients of the above system must equal zero. Thus we require \vspace{1pt}% \begin{equation*} \left| \begin{array}{ll} B_{a}\left( y_{1}\right) & B_{a}\left( y_{2}\right) \\ B_{b}\left( y_{1}\right) & B_{b}\left( y_{2}\right)% \end{array}% \right| =0\qquad \left( 2\right) \end{equation*} \vspace{1pt} Theorem 1. The homogeneous linear B.V.P. $(1)$ has a nontrivial solution if and if $(2)$ holds. \vspace{1pt} Theorem 2. If $u\left( x\right) $ is a particular nontrivial solution of the B.V.P. $(1)$, then all solutions are given by $y=cu\left( x\right) $ where $% c $ is an arbitrary constant. \vspace{1pt} Proof. Let $v\left( x\right) $ be any solution, $u\left( x\right) $ a particular solution of the B.V.P. $(1)$ $\ \Longrightarrow \alpha _{1}u\left( a\right) +\beta _{1}u^{\prime }\left( a\right) =0$ and $\alpha _{1}v\left( a\right) +\beta _{1}v^{\prime }\left( a\right) =0$ since $u$ and $v$ both satisfy the first B.C. These equations may be regarded as equations for $\alpha _{1},\beta _{1}$. However, since by assumption $\alpha _{1}$ and $\beta _{1}$ are not both zero $\Longrightarrow $ \vspace{1pt} \qquad \qquad $\left| \begin{array}{ll} u\left( a\right) & u^{\prime }\left( a\right) \\ v\left( a\right) & v^{\prime }\left( a\right)% \end{array} \right| =0=W\left[ u,v\right] _{x=a}\Longrightarrow W\left[ u\left( x\right) ,v\left( x\right) \right] =0$ for $a\leq x\leq b$ \vspace{1pt} $\Longrightarrow u$ and $v$ are two LD solutions of the D.E. $% \Longrightarrow $ there exist constants $c_{1},c_{2}\neq 0$ such that $% c_{1}u\left( x\right) +c_{2}v\left( x\right) =0$ for $a\leq x\leq b\Longrightarrow v\left( x\right) =-\dfrac{c_{1}}{c_{2}}u\left( x\right) =cu\left( x\right) $. \vspace{1pt} \begin{example} \begin{equation*} y^{\prime \prime }-\lambda ^{2}y=0\qquad \lambda \neq 0\qquad y\left( 0\right) =y\left( 1\right) =0 \end{equation*} \end{example} \vspace{1pt} The general solution is $y=c_{1}e^{\lambda x}+c_{2}e^{-\lambda x}.$ The B.C \ $y\left( 0\right) =0$. $\Longrightarrow c_{1}+c_{2}=0,$ whereas the condition $y\left( 1\right) =0$ leads to $c_{1}e^{\lambda }+c_{2}e^{-\lambda }=0.$ The two equations for $c_{1}$ and $% c_{2}$ are \begin{eqnarray*} c_{1}+c_{2} &=&0 \\ c_{1}e^{\lambda }+c_{2}e^{-\lambda } &=&0 \end{eqnarray*} The determinant of the coefficients is $\left\vert \begin{array}{ll} 1 & 1 \\ e^{\lambda } & e^{-\lambda }% \end{array}% \right\vert \neq 0$. $\Longrightarrow c_{1}=c_{2}=0\Longrightarrow $ the only solution is $y\equiv 0$. \vspace{1pt} \subsection{Eigenvalue Problems} \vspace{1pt} The following special kind of B.V.P. is called an eigenvalue problem. \begin{equation*} \left. \begin{array}{lll} L\left[ y\right] +\lambda y=0 & & a\leq x\leq b \\ \text{B.C. \ }\alpha _{1}y\left( a\right) +\beta _{1}y^{\prime }\left( a\right) =0\qquad & & \alpha _{1}^{2}+\beta _{1}^{2}\neq 0 \\ \text{B.C. \ }\alpha _{2}y\left( b\right) +\beta _{1}y^{\prime }\left( b\right) =0\qquad & & \alpha _{2}^{2}+\beta _{2}^{2}\neq 0% \end{array}% \right\} (\ast ) \end{equation*} \vspace{1pt} Here $L\left[ y\right] =a_{0}\left( x\right) y^{\prime \prime }+a_{1}\left( x\right) y^{\prime }+a_{2}\left( x\right) y,$ and $\lambda $ is a parameter. \vspace{1pt} Again $y\equiv 0$ is a solution \emph{for all} $\lambda $. However, we are interested in nontrivial (nonzero) solutions. \vspace{1pt} Definition. If a nontrivial solution of the B.V.P. $(\ast )$ exists for a value $\lambda =\lambda _{i}$, then $\lambda _{i}$ is called an \emph{% eigenvalue} of $L$ (relevant to the B.Cs.). The corresponding nontrivial solution $y_{i}\left( x\right) $ is called an \emph{eigenfunction}. \begin{example} Find the eigenvalues and eigenfunctions for \end{example} \qquad \qquad \qquad \qquad \qquad \qquad \qquad \qquad \begin{equation*} y^{\prime \prime }+\lambda y=0,\qquad y^{\prime }\left( 0\right) =0,\qquad y\left( 1\right) =0 \end{equation*} We must consider three cases; $\lambda <0,\lambda =0,$, and $\lambda >0$ . \vspace{1pt} I. $\lambda <0$. Let $\lambda =-\alpha ^{2}$ where $\alpha \neq 0$. Then the differential equation becomes \begin{equation*} y^{\prime \prime }-\alpha ^{2}y=0 \end{equation*} and has the general solution \begin{equation*} y=c_{1}e^{\alpha x}+c_{2}e^{-\alpha x}. \end{equation*}% The boundary conditions $\Longrightarrow $ $y^{\prime }\left( 0\right) =c_{1}\alpha -c_{2}\alpha =0$ or $c_{1}=c_{2}$ , and $y\left( 1\right) =c_{1}e^{\alpha }+c_{2}e^{-\alpha }=0$ .$% \Longrightarrow c_{1}=c_{2}=0$. Thus for $\lambda <0$, the only solution is $y=0$. \vspace{1pt} II. $\lambda =0$. The solution is $y=c_{1}x+c_{2}$. The BCs imply $% c_{1}=c_{2}=0$. Again the only solution is $y=0$. III. $\lambda >0$. Let $\lambda =\beta ^{2}$ where $\beta \neq 0$. The DE becomes \begin{equation*} y^{\prime \prime }+\beta ^{2}y=0 \end{equation*} and has the general solution \begin{equation*} y=c_{1}\sin \beta x+c_{2}\cos \beta x. \end{equation*} The BCs imply \vspace{1pt} $y^{\prime }\left( 0\right) =c_{1}\beta \cos 0-c_{2}\beta \sin 0=c_{1}\beta =0$. \qquad Hence\qquad $c_{1}=0,$ , since $\beta \neq 0$.\qquad Thus \begin{equation*} y=c_{2}\cos \beta x. \end{equation*} Now $y\left( 1\right) =c_{2}\cos \beta =0$. Since we want a nontrivial solution we cannot have $c_{2}=0$. Hence \begin{equation*} \cos \beta =0\Longrightarrow \beta =\frac{2n+1}{2}\pi ,n=0,\pm 1,\pm 2,... \end{equation*}% We therefore have the eigenvalues \begin{equation*} \lambda _{n}=\left( \frac{2n+1}{2}\right) ^{2}\pi ^{2}, \end{equation*}% and eigenfunctions \begin{equation*} y_{n}\left( x\right) =C_{n}\cos \left( \frac{2n+1}{2}\right) x, \end{equation*}% for $n=0,1,2,...$ Note the negative values of $n$ do not give additional eigenfunctions since $\cos \left( -t\right) =\cos t$. \begin{example} \vspace{1pt}Find the eigenvalues and eigenfunctions for% \begin{equation*} y^{\prime \prime }-12y^{\prime }+4\left( 7+\lambda \right) y=0\text{ \ \ \ }% y\left( 0\right) =y\left( 5\right) =0 \end{equation*} \end{example} Solution: The characteristic equation is \begin{equation*} r^{2}-12r+4\left( 7+\lambda \right) =0 \end{equation*} so% \begin{equation*} r=\frac{+12\pm \sqrt{144-4\left( 4\right) \left( 7+\lambda \right) }}{2}% =6\pm 2\sqrt{2-\lambda } \end{equation*} Thus we have 3 cases to deal with, $2-\lambda <0,2-\lambda =0,$ and $% 2-\lambda >0.$ Case I: $2-\lambda >0.$ Let $2-\lambda =\alpha ^{2}$ where $\alpha \neq 0.$ The the general homogeneous solution is% \begin{equation*} y\left( x\right) =C_{1}e^{\left( 6+2\alpha \right) x}+C_{2}e^{\left( 6-2\alpha \right) x} \end{equation*} The BCs imply% \begin{eqnarray*} C_{1}+C_{2} &=&0 \\ C_{1}e^{\left( 6+2\alpha \right) 5}+C_{2}e^{\left( 6-2\alpha \right) 5} &=&0 \end{eqnarray*}% , Solution is: $\left\{ C_{2}=0,C_{1}=0\right\} .$ Thus $y=0$ and there are no eigenvalues for this case. \vspace{1pt} Case II: $\lambda =2.$ Then \begin{equation*} y\left( x\right) =C_{1}e^{6x}+C_{2}xe^{6x} \end{equation*} The BCs imply% \begin{eqnarray*} C_{1} &=&0 \\ C_{2}\left( 5\right) e^{30} &=&0\Rightarrow C_{2}=0 \end{eqnarray*} Therefore $\lambda =2$ is not an eigenvalue. \vspace{1pt} Case III: $2-\lambda <0.$ Let $2-\lambda =-\beta ^{2}$ where $\beta \neq 0.$ Then $r=6\pm 2\beta i.$ The solution to the DE is \begin{equation*} y\left( x\right) =C_{1}e^{6x}\sin 2\beta x+C_{2}e^{6x}\cos 2\beta x \end{equation*} The BCs imply% \begin{eqnarray*} y\left( 0\right) &=&C_{2}=0 \\ y\left( 5\right) &=&C_{1}e^{30}\sin 10\beta =0 \end{eqnarray*} Thus \begin{equation*} 10\beta =n\pi ,\text{ \ \ }n=1,2,\ldots \end{equation*} or% \begin{equation*} \beta =\frac{n\pi }{10}\text{ \ \ }n=1,2,\ldots \text{\ } \end{equation*} and the eigenvalues are \begin{equation*} \lambda =2+\beta ^{2}=2+\frac{n^{2}\pi ^{2}}{100}\text{ \ \ }n=1,2,\ldots \end{equation*} \vspace{1pt} The eigenfunctions are \begin{equation*} y_{n}\left( x\right) =A_{n}e^{6x}\sin \left( \frac{n\pi }{5}\right) x \end{equation*} \begin{example} \begin{equation*} \ y^{\prime \prime }+\lambda y=0\qquad y\left( \pi \right) =y\left( 2\pi \right) =0 \end{equation*} \end{example} Solution: \ There are 3 cases to consider. \ $\lambda <0,\lambda =0,$, and $% \lambda >0$ . \vspace{1pt} I. $\lambda <0$. Let $\lambda =-\alpha ^{2}$ where $\alpha \neq 0$. Then the differential equation becomes \begin{equation*} y^{\prime \prime }-\alpha ^{2}y=0 \end{equation*}% and has the general solution \begin{equation*} y\left( x\right) =c_{1}e^{\alpha x}+c_{2}e^{-\alpha x}. \end{equation*} Then \begin{eqnarray*} y\left( \pi \right) &=&c_{1}e^{\alpha \pi }+c_{2}e^{-\alpha \pi }=0 \\ y\left( 2\pi \right) &=&c_{1}e^{2\alpha \pi }+c_{2}e^{-2\alpha \pi }=0 \end{eqnarray*} Thus from the first equation \begin{equation*} c_{2}=-c_{1}e^{2\alpha \pi } \end{equation*}% and the second equation implies% \begin{equation*} c_{1}\left( e^{2\alpha \pi }-1\right) =0 \end{equation*} Hence $c_{1}=0$ and thus $c_{2}=0,$ so $y=0$ is the only solution. \ There are no negative eigenvalues. \vspace{1pt} II. $\lambda =0.$ \ Then we have $y^{\prime \prime }=0$ so \begin{equation*} y\left( x\right) =c_{1}x+c_{2} \end{equation*} \begin{eqnarray*} y\left( \pi \right) &=&c_{1}\pi +c_{2}=0 \\ y\left( 2\pi \right) &=&2c_{1}\pi +c_{2}=0 \end{eqnarray*} Therefore $c_{1}=c_{2}=0$ and $y=0,$ so $0$ is not an eigenvalue. \vspace{1pt} III. \ $\lambda >0.$ Let $\lambda =\beta ^{2}$ The DE becomes% \begin{equation*} y^{\prime \prime }+\beta ^{2}y=0 \end{equation*}% so% \begin{equation*} y\left( x\right) =c_{1}\sin \beta x+c_{2}\cos \beta x \end{equation*}% The initial conditions yield \begin{eqnarray*} y\left( \pi \right) &=&c_{1}\sin \beta \pi +c_{2}\cos \beta \pi =0 \\ y\left( 2\pi \right) &=&c_{1}\sin 2\beta \pi +c_{2}\cos 2\beta \pi =0 \end{eqnarray*} This system will have a non-trivial solution if and only if% \begin{equation*} \left\vert \begin{array}{cc} \sin \beta \pi & \cos \beta \pi \\ \sin 2\beta \pi & \cos 2\beta \pi% \end{array}% \right\vert =0 \end{equation*}% That is if and only if% \begin{equation*} \sin \beta \pi \cos 2\beta \pi -\cos \beta \pi \sin 2\beta \pi =\sin \left( \beta \pi -2\beta \pi \right) =-\sin \beta \pi =0 \end{equation*} Thus we must have \begin{equation*} \beta \pi =n\pi \text{ \ \ \ \ }n=1,2,3,\ldots \end{equation*} \vspace{1pt}or \begin{equation*} \beta =n\text{ \ \ \ \ }n=1,2,3,\ldots \end{equation*} Hence the eigenvalues are \begin{equation*} \lambda =\beta ^{2}=n^{2}\text{ \ }n=1,2,3,\ldots \end{equation*} The two equations above for $c_{1}$ and $c_{2}$ become \begin{eqnarray*} c_{1}\sin n\pi +c_{2}\cos n\pi &=&0 \\ c_{1}\sin 2n\pi +c_{2}\cos 2n\pi &=&0 \end{eqnarray*}% Thus $c_{2}=0$ and $c_{1}$ is arbitrary. The eigenfunctions are \begin{equation*} y_{n}\left( x\right) =a_{n}\sin nx \end{equation*} \vspace{1pt} Remark. If $\overrightarrow{u}$ and $\overrightarrow{v}$ are $2$ vectors, then $\overrightarrow{u}\perp \overrightarrow{v}\Leftrightarrow \overrightarrow{u}\cdot \overrightarrow{v}=0$ $\overrightarrow{u}=\left( x_{1},...,x_{n}\right) \qquad \overrightarrow{v}% \left( y_{1},...,y_{n}\right) $ \ As $n\rightarrow \infty \qquad \overrightarrow{u}\cdot \overrightarrow{v}\rightarrow \int x_{i}y_{i}$. \vspace{1pt} Definition. Let $f\left( x\right) ,g\left( x\right) $ be two continuous functions on $\left[ a,b\right] $. We define the \emph{inner product} of $f$ and $g$ in an interval $a\leq x\leq b$, denoted by $,$ by \qquad \qquad \qquad \qquad \qquad \qquad \qquad \begin{equation*} =\int_{a}^{b}f\left( x\right) g\left( x\right) dx. \end{equation*} Definition. Two functions $f$ and $g$ are said to be \emph{orthogonal} on $% \left[ a,b\right] $ if \begin{equation*} =0. \end{equation*} \vspace{1pt} Example. $\int_{0}^{\pi }\sin x\cos xdx=\frac{\sin ^{2}x}{2}|_{0}^{\pi }=0$ Therefore $\sin x$ and $\cos x$ are orthogonal on$\left[ 0,\pi \right] $. \vspace{1pt} Definition. The set of functions $\left\{ f_{1},f_{2},...\right\} $ is called an \emph{orthogonal} set $=0\qquad i\neq j$. \vspace{1pt} Example. $\left\{ 1,\cos \dfrac{\pi x}{L},\cos \dfrac{2\pi x}{L},\,...,\cos \dfrac{n\pi x}{L},\text{ }...\right\} $ is an orthogonal set on $\left[ 0,L% \right] $ Remark. For vectors we have the following: if $\overrightarrow{u}=\left( u_{1},...,u_{n}\right) $ then the length of $\overrightarrow{u}=\left\| \overrightarrow{u}\right\| =\left( \sum u_{i}^{2}\right) ^{\frac{1}{2}}=% \sqrt{\overrightarrow{u}\cdot \overrightarrow{u}}$. Motivated by this we have the following definition. \vspace{1pt} Definition. Let $f\left( x\right) $ be a continuous function on a$\leq x\leq b$. Then the \emph{norm} of $f$ is defined by \begin{center} \begin{equation*} \left\| f\right\| =\sqrt{}=\sqrt{\int_{a}^{b}f^{2}\left( x\right) dx}. \end{equation*} \end{center} \vspace{1pt} Example. $0\leq x\leq 1\qquad \left\| x^{2}\right\| ^{2}==\int_{0}^{1}x^{4}dx=\dfrac{x^{5}}{5}|_{0}^{1}=\frac{1}{5}$ $\Longrightarrow \left\| x^{2}\right\| =\frac{1}{\sqrt{5}}$. Remark. Let $y=\frac{x^{2}}{\left\| x^{2}\right\| }=\frac{x^{2}}{\sqrt{5}}% \Longrightarrow \left\| y\right\| =\frac{\left\| x^{2}\right\| }{\sqrt{5}}=1$% . Definition. If $\left\| f\right\| =1$, then $f$ \ is said to be \emph{% normalized}. \vspace{1pt} Definition. A set of functions $\left\{ \phi _{1},\phi _{2},...\right\} $ is called \emph{orthonormal} if (1) the set is orthogonal, and (2) each has norm 1. Therefore $\left\{ \phi _{1},\phi _{2},...\right\} $ is an orthonormal set $\Leftrightarrow $ \vspace{1pt} \begin{center} \begin{equation*} <\phi _{i},\phi _{j}>=\delta _{ij}=\left\{ \begin{array}{c} 0\qquad i\neq j \\ 1\qquad i=j% \end{array}% \right. \end{equation*} \end{center} \vspace{1pt} \begin{example} $\left\{ \sin \left( nx\right) \right\} =\left\{ \sin x,\sin 2x,\sin 3x,\,...\right\} $ on$\left[ 0,\pi \right] $ \ is an orthogonal set since \end{example} \begin{eqnarray*} &<&\sin \left( mx\right) ,\sin \left( nx\right) >=\int_{0}^{\pi }\sin mx\sin nx\,dx=\frac{1}{2}\int_{0}^{\pi }\left[ \cos \left( m-n\right) x-\cos \left( m+n\right) x\right] dx\qquad m\neq n \\ &=&\frac{1}{2}\left[ \dfrac{\sin \left( m-n\right) x}{m-n}-\dfrac{\sin \left( m+n\right) x}{m-n}\right] _{0}^{\pi } \\ &=&\frac{1}{2}\left[ \dfrac{\sin \left( m-n\right) \pi }{m-n}-\dfrac{\sin \left( m+n\right) \pi }{m+n}\right] =0\text{ \ \ \ }m\neq n \end{eqnarray*} \vspace{1pt} since $m$ and $n$ are integers. Now \begin{eqnarray*} &<&\sin nx,\sin nx>=\int_{0}^{\pi }\sin ^{2}nxdx \\ &=&\frac{1}{2}\int_{0}^{\pi }\left( 1-\cos 2nx\right) dx \\ &=&\frac{1}{2}\left( x-\frac{\sin 2nx}{2n}\right) |_{0}^{\pi }=\frac{\pi }{2}% . \end{eqnarray*} \qquad \qquad \qquad \qquad \qquad \qquad \qquad \qquad \qquad \qquad Therefore \begin{equation*} \left\| \sin nx\right\| =<\sin nx,\sin nx>^{\frac{1}{2}}=\sqrt{\frac{\pi }{2}% } \end{equation*}% $\Longrightarrow $ this set is not orthonormal. We can make an orthonormal set from these functions by dividing each element in the original by $\sqrt{% \frac{\pi }{2}}\Longrightarrow \left\{ \sqrt{\frac{2}{\pi }}\sin nx\right\} $ is orthonormal set $(n=1,2,$ $...)$. \vspace{1pt} \paragraph{Properties of the inner product.} \begin{equation*} 1.=\text{since}\qquad \int_{a}^{b}f\left( x\right) g\left( x\right) dx=\int_{a}^{b}g\left( x\right) f\left( x\right) dx \end{equation*} \begin{equation*} 2.<\alpha f+\beta g,h>=\alpha +\beta \text{ since }\int \left( \alpha f+\beta g\right) dx=\alpha \int fdx+\beta \int gdx\text{ } \end{equation*} \qquad \qquad \begin{eqnarray*} 3.a. &<&f,f>=0iff=0 \\ b. &<&f,f>>0iff\neq 0 \end{eqnarray*} \ \ Remarks. (1) It will be necessary when dealing with partial differential equations to \textquotedblleft expand\textquotedblright\ an arbitrary function $f\left( x\right) $ in terms of an orthogonal set of functions $% \left\{ \psi _{n}\right\} $. \vspace{1pt} (2) Recall that in 3 space, if $\overrightarrow{u}_{1}=\left( 1,0,0\right) ,% \overrightarrow{u}_{2}=\left( 0,1,0\right) ,$ and $\overrightarrow{u}% _{3}=\left( 0,0,1\right) $ then $\overrightarrow{v}=\left( \alpha _{1},\alpha _{2},\alpha _{3}\right) =\alpha _{1}u_{1}+\alpha _{2}u_{2}+\alpha _{3}u_{3}$. Note that $<\overrightarrow{u}_{1},\overrightarrow{v}>=\overrightarrow{u}_{1}\cdot \overrightarrow{v}=<\overrightarrow{u}_{1},\alpha _{1}\overrightarrow{u}% _{1}+\alpha _{2}\overrightarrow{u}_{2}+\alpha _{3}\overrightarrow{u}_{3}>=<% \overrightarrow{u}_{1},\alpha _{1}\overrightarrow{u}_{1}>+<\overrightarrow{u}% _{1},\alpha _{2}\overrightarrow{u}_{2}>+<\overrightarrow{u}_{1},\alpha _{3}% \overrightarrow{u}_{3}>$ \qquad \qquad \qquad $\qquad \qquad \qquad \qquad \qquad \qquad \qquad \qquad \qquad =\alpha _{1}<\overrightarrow{u}_{1},\overrightarrow{u}% _{1}>+\alpha _{2}<\overrightarrow{u}_{1},\overrightarrow{u}_{2}>+\alpha _{3}<% \overrightarrow{u}_{1},\overrightarrow{u}_{3}>=\alpha _{1}$ Also $<\overrightarrow{u}_{2},\overrightarrow{v}>=\alpha _{2}$ and $<% \overrightarrow{u}_{3},\overrightarrow{v}>=\alpha _{3}$. \vspace{1pt} Suppose we are given a set of orthogonal \ functions $\left\{ \psi _{n}\right\} $ on $\left[ 0,L\right] ,$ and we desire to expand a function $% f\left( x\right) $ given on $\left[ 0,L\right] $ in terms of them. Then we want \qquad \qquad \qquad \qquad \qquad \qquad \qquad \qquad \qquad \begin{equation*} f\left( x\right) =\sum\limits_{n=1}^{\infty }\alpha _{n}\psi _{n}\left( x\right) . \end{equation*} \vspace{1pt} Question. What does $\alpha _{k}=$? \vspace{1pt} Consider \begin{eqnarray*} &<&\psi _{k},f\left( x\right) >=<\psi _{k},\sum\limits_{1}^{\infty }\alpha _{n}\psi _{n}> \\ &=&<\psi _{k},\alpha _{1}\psi _{1}+\alpha _{2}\psi _{2}+\cdots > \\ &=&\alpha _{1}<\psi _{k},\psi _{1}>+\cdots +\alpha _{k}<\psi _{k},\psi _{k}>+% \vspace{1pt}\alpha _{k+1}<\psi _{k},\psi _{k+1}>+\cdots \end{eqnarray*} \qquad \qquad But $<\psi _{k},\psi _{j}>=0$ if $j\neq k$ since the set $\left\{ \psi _{k}\right\} $ is orthogonal. \vspace{1pt} $\Longrightarrow $% \begin{equation*} <\psi _{k},f\left( x\right) >=\alpha _{k}<\psi _{k},\psi _{k}>=\alpha _{k}\left\| \psi _{k}\right\| ^{2} \end{equation*} \vspace{1pt} Therefore \begin{equation*} \alpha _{k}=\dfrac{\int_{0}^{L}f\left( x\right) \psi _{k}\left( x\right) dx}{% \left\| \psi _{k}\right\| ^{2}}=\dfrac{\int_{0}^{L}f\left( x\right) \psi _{k}\left( x\right) dx}{\int_{0}^{L}\left[ \psi _{k}\left( x\right) \right] ^{2}dx}\qquad k=1,2,...\qquad \qquad \qquad \left( \ast \right) \end{equation*} $\left( \ast \right) $ is the formula for the coefficients in the expansion of a function $f\left( x\right) $\ in terms of a set of orthogonal functions. \vspace{1pt} \subsubsection{Ordinary Fourier Series} \vspace{1pt} \paragraph{Fourier Sine Series} \vspace{1pt} Consider the eigenvalue problem \begin{equation*} D.E.y^{\prime \prime }+\lambda y=0\qquad 0\leq x\leq L\qquad B.C.y\left( 0\right) =y\left( L\right) =0\qquad \end{equation*} \vspace{1pt} We shall first solve this problem. There are 3 cases to consider - \ $% \lambda \,<0,\lambda =0,\lambda >0.$ \vspace{1pt} I. $\lambda \,<0.$ \ Let $\lambda \,=-\alpha ^{2}$ where $\alpha \neq 0.$ \ The DE becomes\qquad \vspace{1pt} \begin{equation*} y^{\prime \prime }-\alpha ^{2}y=0 \end{equation*}% so \begin{equation*} y\left( x\right) =c_{1}e^{\alpha x}+c_{2}e^{-\alpha x} \end{equation*}% Then $y\left( 0\right) =0$ implies \vspace{1pt}% \begin{equation*} c_{1}+c_{2}=0 \end{equation*}% so $c_{2}=-c_{1}$ and \begin{equation*} y\left( x\right) =c_{1}\left[ e^{\alpha x}-e^{-\alpha x}\right] \end{equation*} But then \begin{equation*} y\left( L\right) =c_{1}\left[ e^{\alpha L}-e^{-\alpha L}\right] =0 \end{equation*} So $c_{1}=0$ and hence $c_{2}=0$ \ and thus $y\left( x\right) =0$ and there are no negative eigenvalues. \vspace{1pt} II. $\lambda \,=0$ The the equation becomes $y^{\prime \prime }=0$ and $% y=c_{1}x+c_{2}$ \ and the BCs imply $y=0.$ \vspace{1pt} III. \ $\lambda \,>0.$ \ Let $\lambda =\beta ^{2}$ where $\beta \neq 0$ \ The DE becomes% \begin{equation*} y^{\prime \prime }+\beta ^{2}y=0 \end{equation*} Thus% \begin{equation*} y=c_{1}\sin \beta x+c_{2}\cos \beta x \end{equation*}% $y\left( 0\right) =c_{2}=0$. Also% \begin{equation*} y\left( L\right) =c_{1}\sin \beta L=0 \end{equation*}% so% \begin{equation*} \beta =\frac{n\pi }{L}\text{ \ \ \ }n=1,2,3,\ldots \end{equation*} $\Longrightarrow $% \begin{equation*} \lambda _{n}=\dfrac{n^{2}\pi ^{2}}{L^{2}}\text{ \ }n=1,2,3,\ldots \end{equation*}% are the eigenvalues, whereas the eigenfunctions are \begin{equation*} \sin \sqrt{\lambda _{n}}x=\sin \dfrac{n\pi }{L}x=\psi _{n}\text{ \ }% n=1,2,3,\ldots \end{equation*}% These functions form an orthogonal set. \vspace{1pt} Hence if \begin{equation*} f\left( x\right) =\sum\limits_{1}^{\infty }\alpha _{k}\sin \dfrac{k\pi x}{L} \end{equation*} then from $\left( \ast \right) $ above \begin{equation*} \alpha _{k}=\frac{2}{L}\int_{0}^{L}f\left( x\right) \sin \dfrac{k\pi x}{L}dx, \end{equation*}% since% \begin{equation*} \int_{0}^{L}\left[ \psi _{k}\left( x\right) \right] ^{2}dx=\frac{L}{2}. \end{equation*} \vspace{1pt} These formulas are for the Fourier \textit{sine} series for $f\left( x\right) $ on $0-x>0$ \qquad \qquad \qquad \qquad \qquad \qquad $=-f\left( -x\right) ,$ where $% f\left( x\right) $ is value of series in $0=\psi _{0},\beta _{0}\psi _{0}>=<1,1>\beta _{0}$ \vspace{1pt} \qquad $\Longrightarrow \beta _{0}=\dfrac{\int_{0}^{L}1\cdot f\left( x\right) dx}{\int_{0}^{L}1^{2}dx}=\frac{1}{L}\int_{0}^{L}f\left( x\right) dx$% . Note the book writes% \begin{equation*} f\left( x\right) \symbol{126}\frac{a_{0}}{2}+\sum\limits_{1}^{\infty }a_{n}\cos \dfrac{n\pi x}{L} \end{equation*}% and% \begin{equation*} a_{n}=\frac{2}{L}\int_{0}^{L}f\left( x\right) \cos \dfrac{n\pi x}{L}dx\text{ \ }n-0,1,2,\ldots \end{equation*} \vspace{1pt} Thus \begin{equation*} \beta _{0}=\frac{a_{0}}{2} \end{equation*} \vspace{1pt} Again the Fourier series is periodic with period $2L$. However, now $f\left( -x\right) =f\left( x\right) $ since \textit{cosine} is an even function. Here the Fourier Cosine series extends $f\left( x\right) $ which is given on $\left[ 0,L\right] $ to a function $F\left( x\right) $ which is defined on $% -\infty 0\Longrightarrow X=c_{1}e^{\sqrt{k}x}+c_{2}e^{-\sqrt{k}x}$. and the boundary conditions $\Longrightarrow c_{1}=c_{2}=0$. \vspace{1pt} For the case $k<0,$ let $k=-\lambda ^{2}$ $\vspace{1pt}\Longrightarrow $% \begin{equation*} X^{\prime \prime }+\lambda ^{2}X=0\qquad X\left( 0\right) =X\left( L\right) =0 \end{equation*} \vspace{1pt} This is an eigenvalue problem. The solution to the DE is \begin{equation*} X=c_{1}\sin \lambda x+c_{2}\cos \lambda x \end{equation*} $X\left( 0\right) =0\Longrightarrow c_{2}=0$ whereas $X\left( L\right) =0$ $% \ \Longrightarrow c_{1}\sin \lambda =0$ $\ \Longrightarrow \lambda =\frac{% n\pi }{L}$ for $n=\pm 1,\pm 2,\pm 3,...$. Since $\sin \left( -x\right) =-\sin x$ we may disregard the negative values of $n$. Therefore \vspace{1pt} \begin{center} \begin{equation*} X_{n}\left( x\right) =c_{n}\sin \dfrac{n\pi }{L}x\qquad n=1,2,3,... \end{equation*} \end{center} \vspace{1pt} For $T\left( t\right) $ we have the equation \begin{equation*} T^{\prime \prime }+\alpha ^{2}\lambda ^{2}T=0, \end{equation*}% since $k=-\lambda ^{2}$. Thus $\vspace{1pt}$ \begin{center} \begin{equation*} \NEG{T}_{n}\left( t\right) =c\sin \alpha \lambda t+d\cos \alpha \lambda t=a_{n}\sin \dfrac{n\pi \alpha }{L}t+b_{n}\cos \dfrac{n\pi \alpha t}{L}. \end{equation*} \end{center} \vspace{1pt} But $y_{t}\left( x,0\right) =0\Longrightarrow T^{\prime }\left( 0\right) =0.$ Now $T^{\prime }\left( t\right) =a_{n}\left( \alpha \dfrac{n\pi }{L}\right) \cos \alpha \dfrac{n\pi }{L}t-b_{n}\left( \alpha \dfrac{n\pi }{L}\right) \sin \alpha \dfrac{n\pi t}{L}$, so $T^{\prime }\left( 0\right) =0$ $% \Rightarrow a_{n}=0$ for all $n$ . Therefore $\vspace{1pt}$ \begin{center} \begin{equation*} T_{n}\left( t\right) =b_{n}\cos \dfrac{n\pi \alpha t}{L}, \end{equation*} \end{center} \vspace{1pt} and we have finally that \vspace{1pt} \qquad \qquad \qquad \qquad \qquad \begin{equation*} y_{n}\left( x,t\right) =X_{n}\left( x\right) T_{n}\left( t\right) =c_{n}\sin \dfrac{n\pi x}{L}\times b_{n}\cos \dfrac{n\pi \alpha t}{L} \end{equation*} \vspace{1pt} Let $c_{n}\times b_{n}=d_{n}.$ We note that \begin{equation*} y_{n}\left( x,t\right) =d_{n}\sin \dfrac{n\pi x}{L}\cos \dfrac{n\pi \alpha t% }{L}\text{ \ \ }n=1,2,3,\ldots \end{equation*} \vspace{1pt} satisfies the P.D.E. $y_{xx}=\dfrac{1}{\alpha ^{2}}y_{tt}$ $\ \left( 1\right) \qquad $and the boundary conditions $y\left( 0,t\right) =y\left( L,t\right) =0$ $\left( 2a,2b\right) ,$ as well as the initial condition $% y_{t}\left( 0\right) =0$ $\left( 3b\right) $. \vspace{1pt} What about the condition $y\left( x,0\right) =f\left( x\right) $? Notice that \begin{equation*} y\left( x,t\right) =\sum\limits_{1}^{\infty }d_{n}\sin \dfrac{n\pi x}{L}\cos \dfrac{n\pi \alpha t}{L} \end{equation*} is also a solution since of $\left( 1\right) ,\left( 2a,b\right) $ and $% \left( 3b\right) $. Thus $y\left( x,t\right) $ is solution of everything except condition $\left( 3a\right) ,$ namely, $y\left( x,0\right) =f\left( x\right) $. But \vspace{1pt} \qquad \qquad \qquad \qquad \qquad \begin{equation*} y\left( x,0\right) =\sum\limits_{1}^{\infty }d_{n}\sin \dfrac{n\pi x}{L}% =f\left( x\right) . \end{equation*} \vspace{1pt} Therefore if $f$ has a Fourier sine series expansion we let \begin{center} $\vspace{1pt}$ \end{center} $\Longrightarrow $% \begin{equation*} d_{n}=\frac{2}{L}\int_{0}^{L}f\left( x\right) \sin \dfrac{n\pi x}{L}dx. \end{equation*} \vspace{1pt} Now with these coefficients $d_{n}$ \vspace{1pt} \qquad \qquad \qquad \qquad \qquad \qquad \qquad \begin{equation*} y\left( x,t\right) =\sum\limits_{1}^{\infty }d_{n}\sin \dfrac{n\pi x}{L}\cos \dfrac{n\pi \alpha t}{L} \end{equation*} is a solution to entire problem $\left( 1\right) ,\left( 2a,2b\right) ,\left( 3a,3b\right) $. \vspace{1pt} \begin{example} \begin{eqnarray*} y_{xx} &=&y_{tt}\qquad y\left( 0,t\right) =y\left( L,t\right) =0 \\ y_{t}\left( x,0\right) &=&0 \\ y\left( x,0\right) &=&2\sin \dfrac{\pi x}{L} \end{eqnarray*} \end{example} \qquad \qquad \qquad \qquad \qquad Here $\alpha =1$ and $f\left( x\right) =2\sin \dfrac{\pi x}{L}$\qquad \qquad Now \begin{equation*} y\left( x,t\right) =\sum\limits_{1}^{\infty }d_{n}\sin \dfrac{n\pi x}{L}\cos \dfrac{n\pi t}{L} \end{equation*} \vspace{1pt} \begin{equation*} d_{n}=\frac{2}{L}\int_{0}^{L}\sin \dfrac{\pi x}{L}\sin \dfrac{n\pi x}{L}dx=% \frac{2}{L}\int_{0}^{L}2\sin \dfrac{\pi x}{L}\sin \dfrac{n\pi x}{L}% dx=0\qquad n=2,3,... \end{equation*} \vspace{1pt} \begin{equation*} d_{1}=\frac{2}{L}\int_{0}^{L}\left( 2\right) \sin ^{2}\dfrac{n\pi x}{L}dx=% \frac{4}{L}\left[ \int_{0}^{L}\left( \frac{1-\cos \tfrac{2n\pi x}{L}}{2}% \right) \right] dx=\frac{4}{L}\left[ \frac{x}{2}-\left( \tfrac{\sin 2\tfrac{% n\pi x}{L}}{\dfrac{2n\pi }{L}}\right) \right] _{0}^{L}=2 \end{equation*} \vspace{1pt} $\Longrightarrow $ solution is \vspace{1pt} \qquad \qquad \qquad \qquad \qquad \qquad \qquad \qquad \begin{equation*} y\left( x,t\right) =2\sin \dfrac{n\pi }{L}\cos \dfrac{\pi t}{L}. \end{equation*} \begin{example} Solve:\qquad \end{example} \vspace{1pt} \begin{eqnarray*} \text{P.D.E.} &\text{:}&\ u_{xx}-16u_{tt}=0 \\ \text{B.C.'s} &\text{:}&\ \ u(0,t)=0\qquad u_{x}(1,t)=0 \\ \text{I.C.} &\text{:}&\ \ u(x,0)=-3\sin \frac{5\pi x}{2}+23\sin \frac{11\pi x% }{2};\text{ \ \ \ }u_{t}(x,0)=0 \end{eqnarray*} Solution: We assume \begin{equation*} u\left( x,t\right) =X\left( x\right) T\left( t\right) \end{equation*} The PDE implies% \begin{equation*} \frac{X^{\prime \prime }}{X}=16\frac{T^{\prime \prime }}{T}=k\text{ \ }k% \text{ a constant} \end{equation*} Then we have the two ordinary DEs% \begin{eqnarray*} X^{\prime \prime }-kX &=&0 \\ T^{\prime \prime }-\frac{1}{16}kT &=&0 \end{eqnarray*} The boundary conditions for $X\left( x\right) $ are% \begin{equation*} X\left( 0\right) =X^{\prime }\left( 1\right) =0 \end{equation*} so that the eigenvalue problem for $X$ is% \begin{equation*} X^{\prime \prime }-kX=0\text{ \ \ }X\left( 0\right) =X^{\prime }\left( 1\right) =0 \end{equation*} For nontrivial solutions we let $k=-\beta ^{2},\beta \neq 0$ and get% \begin{equation*} X^{\prime \prime }+\beta ^{2}X=0 \end{equation*}% so% \begin{equation*} X\left( x\right) =C_{1}\sin \beta x+C_{2}\cos \beta x \end{equation*} \begin{equation*} X\left( 0\right) =0\Rightarrow C_{2}=0 \end{equation*} Thus% \begin{equation*} X^{\prime }\left( x\right) =C_{1}\beta \cos \beta x \end{equation*} and $X^{\prime }\left( 1\right) =0\Rightarrow $% \begin{equation*} \beta =\left( \frac{2n+1}{2}\right) \pi \text{ \ \ \ }n=0,1,2,\ldots \end{equation*} Therefore% \begin{equation*} X_{n}\left( x\right) =a_{n}\sin \left( \frac{2n+1}{2}\right) \pi x\text{ \ \ }n=0,1,2,\ldots \end{equation*} Since% \begin{equation*} k=-\beta ^{2}=\left( \frac{2n+1}{2}\right) ^{2}\pi ^{2} \end{equation*} The equation for $T\left( t\right) $ becomes% \begin{equation*} T^{\prime \prime }+\frac{1}{16}\left( \frac{2n+1}{2}\right) ^{2}\pi ^{2}T=0 \end{equation*} so% \begin{equation*} T_{n}\left( t\right) =b_{n}\sin \left( \frac{2n+1}{8}\right) \pi t+c_{n}\cos \left( \frac{2n+1}{8}\right) \pi t\text{ \ \ \ \ }n=0,1,2,\ldots \text{\ } \end{equation*} The BC $u_{t}(x,0)=0\Rightarrow T^{\prime }\left( 0\right) =0.$ Since \begin{equation*} T_{n}^{\prime }\left( t\right) =b_{n}\left( \frac{2n+1}{8}\right) \pi \cos \left( \frac{2n+1}{8}\right) \pi t-c_{n}\left( \frac{2n+1}{8}\right) \pi \sin \left( \frac{2n+1}{8}\right) \pi t \end{equation*} we see that $b_{n}=0$ so that \begin{equation*} T_{n}\left( t\right) =c_{n}\cos \left( \frac{2n+1}{8}\right) \pi t\text{ \ \ \ \ }n=0,1,2,\ldots \text{\ } \end{equation*} Thus% \begin{equation*} u_{n}\left( x,t\right) =X_{n}\left( x\right) T_{n}\left( t\right) =D_{n}\sin \left( \frac{2n+1}{2}\right) \pi x\cos \left( \frac{2n+1}{8}\right) \pi t% \text{ \ \ }n=0,1,2,\ldots \end{equation*} Let \begin{equation*} u\left( x,t\right) =\sum_{n=0}^{\infty }u_{n}\left( x,t\right) =\sum_{n=0}^{\infty }D_{n}\sin \left( \frac{2n+1}{2}\right) \pi x\cos \left( \frac{2n+1}{8}\right) \pi t\text{ } \end{equation*} Then \begin{equation*} u(x,0)=\sum_{n=0}^{\infty }D_{n}\sin \left( \frac{2n+1}{2}\right) \pi x=-3\sin \frac{5\pi x}{2}+23\sin \frac{11\pi x}{2} \end{equation*} Therefore% \begin{equation*} D_{2}=-3\text{ \ \ }D_{5}=23\text{ \ \ }D_{n}=0\text{ \ }n\neq 2,5 \end{equation*} The final solution is then% \begin{equation*} u\left( x,t\right) =-3\sin \left( \frac{5\pi x}{2}\right) \cos \left( \frac{% 5\pi t}{8}\right) +23\sin \left( \frac{11\pi x}{2}\right) \cos \frac{11\pi t% }{8} \end{equation*} $u\left( x,0\right) =\allowbreak -3\sin \frac{5}{2}\pi x+23\sin \frac{11}{2}% \pi x$\FRAME{dtbpFX}{3.0007in}{1.9993in}{0pt}{}{}{Plot}{\special{language "Scientific Word";type "MAPLEPLOT";width 3.0007in;height 1.9993in;depth 0pt;display "USEDEF";plot_snapshots FALSE;mustRecompute FALSE;lastEngine "MuPAD";xmin "0";xmax "1";xviewmin "-0.00010000010002";xviewmax "1.00010000010002";yviewmin "-26.0050277671315";yviewmax "24.2826490723315";plottype 4;axesFont "Times New Roman,12,0000000000,useDefault,normal";numpoints 49;plotstyle "patch";axesstyle "normal";axestips FALSE;xis \TEXUX{x};var1name \TEXUX{$x$};function \TEXUX{$-3\sin \frac{5}{2}\pi x+23\sin \frac{11}{2}\pi x$};linecolor "black";linestyle 1;pointstyle "point";linethickness 2;lineAttributes "Solid";var1range "0,1";num-x-gridlines 49;curveColor "[flat::RGB:0000000000]";curveStyle "Line";rangeset"X";function \TEXUX{$-3\sin \frac{5}{2}\pi x\cos 0.062\,5\pi +23\sin \frac{11}{2}\pi x\cos 0.137\,5\pi \allowbreak $};linecolor "blue";linestyle 1;pointstyle "point";linethickness 2;lineAttributes "Solid";var1range "0,1";num-x-gridlines 49;curveColor "[flat::RGB:0x000000ff]";curveStyle "Line";function \TEXUX{$-3\sin \frac{5}{2}\pi x\cos 0.25\pi +23\sin \frac{11}{2}\pi x\cos 0.55\pi \allowbreak $};linecolor "green";linestyle 1;pointstyle "point";linethickness 2;lineAttributes "Solid";var1range "0,1";num-x-gridlines 49;curveColor "[flat::RGB:0x00008000]";curveStyle "Line";function \TEXUX{$23\sin \frac{11}{2}\pi x\cos 1.\,\allowbreak 1\pi $};linecolor "cyan";linestyle 1;pointstyle "point";linethickness 1;lineAttributes "Solid";var1range "0,1";num-x-gridlines 49;curveColor "[flat::RGB:0x0000ffff]";curveStyle "Line";function \TEXUX{$-3\sin \frac{5}{2}\pi x\cos 0.375\,\pi +23\sin \frac{11}{2}\pi x\cos 0.825\,\pi $};linecolor "cyan";linestyle 1;pointstyle "point";linethickness 1;lineAttributes "Solid";var1range "0,1";num-x-gridlines 49;curveColor "[flat::RGB:0x00008080]";curveStyle "Line";VCamFile 'N4QRHX0D.xvz';}} $u\left( x,.1\right) =\allowbreak -3\sin \frac{5}{2}\pi x\cos 0.062\,5\pi +23\sin \frac{11}{2}\pi x\cos 0.137\,5\pi \allowbreak $ $u\left( x,.4\right) =\allowbreak -3\sin \frac{5}{2}\pi x\cos 0.25\pi +23\sin \frac{11}{2}\pi x\cos 0.55\pi \allowbreak $ $u\left( x,.6\right) =\allowbreak -3\sin \frac{5}{2}\pi x\cos 0.375\,\pi +23\sin \frac{11}{2}\pi x\cos 0.825\,\pi \allowbreak $ $u\left( x,.8\right) =\allowbreak 23\sin \frac{11}{2}\pi x\cos 1.\,\allowbreak 1\pi $ \vspace{1pt} \begin{example} Solve \end{example} \ \ \begin{eqnarray*} \text{PDE \ \ \ \ \ \ \ }u_{xx}-16u_{tt} &=&0 \\ \text{BCs \ \ }u(0,t) &=&0\qquad u_{x}(1,t)=0 \\ \text{IC \ \ \ \ }u(x,0) &=&-6\sin \left( \frac{3\pi x}{2}\right) +13\sin \left( \frac{11\pi x}{2}\right) \\ \text{IC \ \ }u_{t}(x,0) &=&0 \end{eqnarray*} You must derive the solution. \ Your solution should not have any arbitrary constants in it. Show \textbf{all }steps. Solution: \ Let $u\left( x,t\right) =X\left( x\right) T\left( t\right) .$ Then the PDE implies \vspace{1pt}% \begin{equation*} X^{\prime \prime }T=16XT^{\prime \prime } \end{equation*}% or% \begin{equation*} \frac{X^{\prime \prime }}{X}=16\frac{T^{\prime \prime }}{T}=-\lambda ^{2} \end{equation*}% since we will need sines and cosines in the $X$ part of the solution. Thus% \begin{eqnarray*} X^{\prime \prime }+\lambda ^{2}X &=&0 \\ T^{\prime \prime }+\frac{\lambda ^{2}}{16}T &=&0 \end{eqnarray*}% The BCs are \begin{equation*} X\left( 0\right) =X^{\prime }\left( 1\right) =0 \end{equation*}% \begin{equation*} X\left( x\right) =a_{n}\sin \lambda x+b_{n}\cos \lambda x \end{equation*} $X\left( 0\right) =0$ implies that $b_{n}=0,$ so \begin{equation*} X\left( x\right) =a_{n}\sin \lambda x \end{equation*}% \begin{equation*} X^{\prime }\left( x\right) =a_{n}\lambda \cos \lambda x \end{equation*}% so% \begin{equation*} X^{\prime }\left( 1\right) =a_{n}\lambda \cos \lambda =0 \end{equation*}% Hence $\lambda =\frac{2n+1}{2}\pi ,$ \ $n=0,1,2,\ldots $ and% \begin{equation*} X_{n}\left( x\right) =A_{n}\sin \left( \frac{2n+1}{2}\right) \pi x\text{ \ \ }\ n=0,1,2,\ldots \end{equation*}% Also% \begin{equation*} T^{\prime \prime }+\frac{\lambda ^{2}}{16}T=T^{\prime \prime }+\frac{\left( 2n+1\right) ^{2}\pi ^{2}}{64}T=0 \end{equation*} \begin{equation*} T_{n}\left( t\right) =c_{n}\sin \left( \frac{2n+1}{8}\right) \pi t+d_{n}\cos \left( \frac{2n+1}{8}\right) \pi t \end{equation*} $u_{t}(x,0)=0$ implies that $c_{n}=0$ and \begin{equation*} T_{n}\left( t\right) =d_{n}\cos \left( \frac{2n+1}{8}\right) \pi t \end{equation*}% Thus% \begin{equation*} u_{n}\left( x.t\right) =B_{n}\sin \left( \frac{2n+1}{2}\right) \pi x\cos \left( \frac{2n+1}{8}\right) \pi t\text{ \ \ }n=0,1,2,\ldots \end{equation*}% Let \begin{equation*} u\left( x,t\right) =\sum_{n=0}^{\infty }u_{n}\left( x.t\right) =\sum_{n=0}^{\infty }B_{n}\sin \left( \frac{2n+1}{2}\right) \pi x\cos \left( \frac{2n+1}{8}\right) \pi t \end{equation*} \begin{equation*} u\left( x,0\right) =\sum_{n=0}^{\infty }B_{n}\sin \left( \frac{2n+1}{2}% \right) \pi x=-6\sin \left( \frac{3\pi x}{2}\right) +13\sin \left( \frac{% 11\pi x}{2}\right) \end{equation*}% Therefore $B_{1}=-6,B_{5}=13$ and $B_{n}=0$ for $n\neq 1,5$ so \begin{equation*} u\left( x,t\right) =-6\sin \left( \frac{3\pi x}{2}\right) \cos \left( \frac{% 3\pi }{8}\right) t+13\sin \left( \frac{11\pi x}{2}\right) \cos \left( \frac{% 11\pi }{8}\right) t \end{equation*} \vspace{1pt} \begin{example} Solve:\qquad \end{example} \vspace{1pt} \begin{eqnarray*} \text{P.D.E.} &\text{:}&\ u_{xx}-16u_{tt}=0 \\ \text{B.C.'s} &\text{:}&\ \ u(0,t)=0\qquad u_{x}(1,t)=0 \\ \text{I.C.} &\text{:}&\ \ u(x,0)=-3\sin \frac{5\pi x}{2}+23\sin \frac{11\pi x% }{2};\text{ \ \ \ }u_{t}(x,0)=2\pi \sin \frac{3\pi x}{2} \end{eqnarray*} Solution: We assume \begin{equation*} u\left( x,t\right) =X\left( x\right) T\left( t\right) \end{equation*} The PDE implies% \begin{equation*} \frac{X^{\prime \prime }}{X}=16\frac{T^{\prime \prime }}{T}=k\text{ \ }k% \text{ a constant} \end{equation*} Then we have the two ordinary DEs% \begin{eqnarray*} X^{\prime \prime }-kX &=&0 \\ T^{\prime \prime }-\frac{1}{16}kT &=&0 \end{eqnarray*} The boundary conditions for $X\left( x\right) $ are% \begin{equation*} X\left( 0\right) =X^{\prime }\left( 1\right) =0 \end{equation*} so that the eigenvalue problem for $X$ is% \begin{equation*} X^{\prime \prime }-kX=0\text{ \ \ }X\left( 0\right) =X^{\prime }\left( 1\right) =0 \end{equation*} For nontrivial solutions we let $k=-\beta ^{2},\beta \neq 0$ and get% \begin{equation*} X^{\prime \prime }+\beta ^{2}X=0 \end{equation*}% so% \begin{equation*} X\left( x\right) =C_{1}\sin \beta x+C_{2}\cos \beta x \end{equation*} \begin{equation*} X\left( 0\right) =0\Rightarrow C_{2}=0 \end{equation*} Thus% \begin{equation*} X^{\prime }\left( x\right) =C_{1}\beta \cos \beta x \end{equation*} and $X^{\prime }\left( 1\right) =0\Rightarrow $% \begin{equation*} \beta =\left( \frac{2n+1}{2}\right) \pi \text{ \ \ \ }n=0,1,2,\ldots \end{equation*} Therefore% \begin{equation*} X_{n}\left( x\right) =a_{n}\sin \left( \frac{2n+1}{2}\right) \pi x\text{ \ \ }n=0,1,2,\ldots \end{equation*} Since% \begin{equation*} k=-\beta ^{2}=\left( \frac{2n+1}{2}\right) ^{2}\pi ^{2} \end{equation*} The equation for $T\left( t\right) $ becomes% \begin{equation*} T^{\prime \prime }+\frac{1}{16}\left( \frac{2n+1}{2}\right) ^{2}\pi ^{2}T=0 \end{equation*} so% \begin{equation*} T_{n}\left( t\right) =b_{n}\sin \left( \frac{2n+1}{8}\right) \pi t+c_{n}\cos \left( \frac{2n+1}{8}\right) \pi t\text{ \ \ \ \ }n=0,1,2,\ldots \text{\ } \end{equation*} Thus% \begin{equation*} u_{n}\left( x,t\right) =X_{n}\left( x\right) T_{n}\left( t\right) =D_{n}\sin \left( \frac{2n+1}{2}\right) \pi x\sin \left( \frac{2n+1}{8}\right) \pi t+E_{n}\sin \left( \frac{2n+1}{2}\right) \pi x\cos \left( \frac{2n+1}{8}% \right) \pi t\text{ \ \ }n=0,1,2,\ldots \end{equation*} Let \begin{equation*} u\left( x,t\right) =\sum_{n=0}^{\infty }u_{n}\left( x,t\right) =\sum_{n=0}^{\infty }\left[ D_{n}\sin \left( \frac{2n+1}{2}\right) \pi x\sin \left( \frac{2n+1}{8}\right) \pi t\text{ }+E_{n}\sin \left( \frac{2n+1}{2}% \right) \pi x\cos \left( \frac{2n+1}{8}\right) \pi t\right] \end{equation*} \begin{eqnarray*} &&u_{t}\left( x,t\right) \\ &=&\sum_{n=0}^{\infty }\left[ D_{n}\left( \frac{2n+1}{8}\right) \pi \sin \left( \frac{2n+1}{2}\right) \pi x\cos \left( \frac{2n+1}{8}\right) \pi t% \text{ }-E_{n}\left( \frac{2n+1}{8}\right) \pi \sin \left( \frac{2n+1}{2}% \right) \pi x\sin \left( \frac{2n+1}{8}\right) \pi t\right] \end{eqnarray*} Then \begin{equation*} u\left( x,0\right) =\sum_{n=0}^{\infty }E_{n}\sin \left( \frac{2n+1}{2}% \right) \pi x=-3\sin \frac{5\pi x}{2}+23\sin \frac{11\pi x}{2} \end{equation*} Therefore% \begin{equation*} E_{2}=-3\text{ \ \ }E_{5}=23\text{ \ \ }E_{n}=0\text{ \ }n\neq 2,5 \end{equation*} \begin{equation*} u_{t}\left( x.0\right) =\sum_{n=0}^{\infty }D_{n}\left( \frac{2n+1}{8}% \right) \pi \sin \left( \frac{2n+1}{2}\right) \pi x=2\pi \sin \frac{3\pi x}{2% } \end{equation*} Thus $D_{1}\left( \frac{3}{8}\right) \pi =2\pi $ so $D_{1}=\frac{16}{3}$ and $D_{n}=0$ \ $n\neq 1$ The final solution is then% \begin{equation*} u\left( x,t\right) =\frac{16}{3}\sin \left( \frac{3\pi x}{2}\right) \cos \frac{3\pi t}{8}-3\sin \left( \frac{5\pi x}{2}\right) \cos \left( \frac{5\pi t}{8}\right) +23\sin \left( \frac{11\pi x}{2}\right) \cos \frac{11\pi t}{8} \end{equation*} \subsection{\protect\Large The Heat Equation} \vspace{1pt} Consider a cylinder parallel to $x-$axis \vspace{1pt} \vspace{1pt}\FRAME{dtbpF}{3.3442in}{2.1914in}{0pt}{}{}{image9.gif}{\special% {language "Scientific Word";type "GRAPHIC";maintain-aspect-ratio TRUE;display "USEDEF";valid_file "F";width 3.3442in;height 2.1914in;depth 0pt;original-width 283.0625pt;original-height 184.4375pt;cropleft "0";croptop "1";cropright "1";cropbottom "0";filename '/document/graphics/Image90.gif';file-properties "XNPEU";}} Let $u$ denote the temperature in the cylinder. Suppose the ends $x=0$ and $% x=L$ are kept at zero temperature whereas at $t=0$ the initial temperature distribution is $u=f\left( x\right) $. It may be shown that $u=u\left( x,t\right) $ satisfies the P.D.E. \vspace{1pt} $\qquad $% \begin{equation*} u_{xx}=\dfrac{1}{k}u_{t}\qquad 00,\qquad (1) \end{equation*} \vspace{1pt} where $k$\ is a constant and $k>0$ . Equation $(1)$ is called the heat equation. The physical conditions of the problem imply \vspace{1pt} \qquad \begin{equation*} \ B.C.\ u\left( 0,t\right) =0=u\left( L,t\right) \qquad t\geq 0\qquad \ \ \ \ (2) \end{equation*} \qquad \begin{equation*} \ I.C.\ \ u\left( x,0\right) =f\left( x\right) \qquad 0\leq x\leq L\qquad \qquad (3) \end{equation*} \vspace{1pt} We want to determine $u\left( x,t\right) $, i.e. the temperature in the cylinder at any point $x$ at any time $t$. Again we use separation of variables. The assumption $u\left( x,t\right) =X\left( x\right) T\left( t\right) $ leads to \vspace{1pt} \qquad \qquad \qquad \qquad \begin{equation*} \dfrac{X^{\prime \prime }\left( x\right) }{X\left( x\right) }=\dfrac{1}{k}% \dfrac{T^{\prime }\left( t\right) }{T\left( t\right) }=-\lambda ^{2} \end{equation*} $\vspace{1pt}$ $\Longrightarrow X^{\prime \prime }+\lambda ^{2}X=0\qquad X\left( 0\right) =X\left( L\right) =0$ and $T^{\prime }+k\lambda ^{2}T=0$. \begin{center} \vspace{1pt} \end{center} $\Longrightarrow X_{n}=c_{n}\sin \dfrac{n\pi x}{L}\qquad n=1,2,...\qquad \lambda _{n}=\dfrac{n\pi }{L}\Longrightarrow $ $\vspace{1pt}$ \begin{center} \begin{equation*} T^{\prime }+k\dfrac{n^{2}\pi ^{2}}{L^{2}}T=0 \end{equation*} \end{center} $\vspace{1pt}$ $\Longrightarrow $% \begin{equation*} T\left( t\right) =d_{n}e^{-\left( \frac{n\pi }{L}\right) ^{2}kt} \end{equation*}% $\Longrightarrow $ $\vspace{1pt}$ \begin{center} \begin{equation*} u_{n}\left( x,t\right) =a_{n}e^{-\left( \frac{n\pi }{L}\right) ^{2}kt}\sin \dfrac{n\pi x}{L} \end{equation*} \end{center} \vspace{1pt} satisfies $(1)$ and $\left( 2\right) $ $\ \Longrightarrow $% \begin{equation*} u_{n}\left( x,t\right) =\sum_{1}^{\infty }a_{n}e^{-\left( \frac{n\pi }{L}% \right) ^{2}kt}\sin \dfrac{n\pi x}{L} \end{equation*} \vspace{1pt} also satisfies $\left( 1\right) $ and \ $\left( 2\right) $. \vspace{1pt} We need to satisfy $\left( 3\right) $ namely, $u\left( x,0\right) =f\left( x\right) $ However, $\vspace{1pt}$ \begin{center} \begin{equation*} u\left( x,0\right) =\sum\limits_{1}^{\infty }a_{n}\sin \dfrac{n\pi x}{L} \end{equation*} \end{center} Thus we take $a_{n}$ to be the Fourier sine coefficients of $f\left( x\right) .$ Hence $\vspace{1pt}$ \begin{center} \begin{equation*} a_{n}=\frac{2}{L}\int_{0}^{L}f\left( x\right) \sin \dfrac{n\pi x}{L}dx. \end{equation*} \end{center} Remark. The factor $e^{-\left( \frac{n\pi }{L}\right) ^{2}kt}\longrightarrow 0$ as $t\longrightarrow \infty \Longrightarrow \underset{t\rightarrow \infty }{\lim }u\left( x,t\right) =0$ as expected from the physical problem. \begin{example} Solve the problem: \end{example} \qquad \qquad \begin{eqnarray*} \text{P.D.E.}\text{: \ \ \ } &&\ u_{xx}-8u_{t}=0 \\ \text{B.C.} &\text{:}&\ \ u(0,t)=0\qquad u_{x}(1,t)=0 \\ \text{I.C.} &\text{:}&u(x,0)=-2\sin \frac{3\pi }{2}x+10\sin \frac{9\pi }{2}x \end{eqnarray*} Solution: Let $u\left( x,t\right) =X\left( x\right) T\left( t\right) .$ Then the PDE implies% \begin{equation*} \frac{X^{\prime \prime }}{X}=8\frac{T^{\prime }}{T}=k\text{ \ }k\text{ a constant} \end{equation*} Then we have the two ODEs% \begin{eqnarray*} X^{\prime \prime }-kX &=&0 \\ T^{\prime }-\frac{1}{8}kT &=&0 \end{eqnarray*} The BCs for $X\left( x\right) $ are% \begin{equation*} X\left( 0\right) =X^{\prime }\left( 1\right) =0 \end{equation*} The boundary conditions for $X\left( x\right) $ are% \begin{equation*} X\left( 0\right) =X^{\prime }\left( 1\right) =0 \end{equation*} so that the eigenvalue problem for $X$ is% \begin{equation*} X^{\prime \prime }-kX=0\text{ \ \ }X\left( 0\right) =X^{\prime }\left( 1\right) =0 \end{equation*} For nontrivial solutions we let $k=-\beta ^{2},\beta \neq 0$ and get% \begin{equation*} X^{\prime \prime }+\beta ^{2}X=0 \end{equation*}% so% \begin{equation*} X\left( x\right) =C_{1}\sin \beta x+C_{2}\cos \beta x \end{equation*} \begin{equation*} X\left( 0\right) =0\Rightarrow C_{2}=0 \end{equation*} Thus% \begin{equation*} X^{\prime }\left( x\right) =C_{1}\beta \cos \beta x \end{equation*} and $X^{\prime }\left( 1\right) =0\Rightarrow $% \begin{equation*} \beta =\left( \frac{2n+1}{2}\right) \pi \text{ \ \ \ }n=0,1,2,\ldots \end{equation*} Therefore% \begin{equation*} X_{n}\left( x\right) =a_{n}\sin \left( \frac{2n+1}{2}\right) \pi x\text{ \ \ }n=0,1,2,\ldots \end{equation*} The equation for $T\left( t\right) $ with $k=-\beta ^{2}=\left( \frac{2n+1}{2% }\right) ^{2}\pi ^{2}$ is% \begin{equation*} T^{\prime }+\frac{1}{8}\left( \frac{2n+1}{2}\right) ^{2}\pi ^{2}T=0 \end{equation*} Thus% \begin{equation*} T_{n}\left( t\right) =b_{n}e^{-\frac{1}{8}\left( \frac{2n+1}{2}\right) ^{2}\pi ^{2}t}\text{ \ \ }n=0,1,2,\ldots \end{equation*} Therefore we have% \begin{equation*} u_{n}\left( x,t\right) =D_{n}\sin \left( \frac{2n+1}{2}\right) \pi xe^{-% \frac{1}{8}\left( \frac{2n+1}{2}\right) ^{2}\pi ^{2}t}\text{ \ \ }% n=0,1,2,\ldots \end{equation*} To satisfy the initial condition we let% \begin{equation*} u\left( x,t\right) =\sum_{n=0}^{\infty }D_{n}\sin \left( \frac{2n+1}{2}% \right) \pi xe^{-\frac{1}{8}\left( \frac{2n+1}{2}\right) ^{2}\pi ^{2}t} \end{equation*} Now% \begin{equation*} u\left( x,0\right) =\sum_{n=0}^{\infty }D_{n}\sin \left( \frac{2n+1}{2}% \right) \pi x=-2\sin \frac{3\pi }{2}x+10\sin \frac{9\pi }{2}x \end{equation*} This means% \begin{equation*} D_{1}=-2\text{ \ \ }D_{4}=10\text{ \ and \ }D_{n}=0,\text{ \ }n\neq 1,2 \end{equation*} The solution to the problem is then% \begin{equation*} u\left( x,t\right) =-2\sin \left( \frac{3}{2}\right) \pi xe^{-\frac{1}{8}% \left( \frac{3}{2}\right) ^{2}\pi ^{2}t}+10\sin \left( \frac{9}{2}\right) \pi xe^{-\frac{1}{8}\left( \frac{9}{2}\right) ^{2}\pi ^{2}t} \end{equation*} \vspace{1pt}\pagebreak \subsubsection{Additional Examples} \vspace{1pt} \begin{example} Wave Equation Example \end{example} \vspace{1pt} Problem 1 Section 10.6 \vspace{1pt} Find a formal solution to the vibrating string problem governed by the given initial-boundary value problem. \vspace{1pt} \begin{eqnarray*} u_{tt} &=&u_{xx},\text{ \ \ \ \ }00 \\ u\left( 0,t\right) &=&u\left( 1,t\right) =0,\text{ \ \ }t>0 \\ u\left( x,0\right) &=&x\left( 1-x\right) ,\text{ \ \ }00,$the only solution is $X=0.$ For $k=0$ we have $X=Ax+B.$ $X^{\prime }\left( x\right) =A,$ so the BCs imply that $X^{\prime }\left( 0\right) =X^{\prime }\left( \pi \right) =A=0.$ \begin{equation*} X\left( x\right) =B,\text{ \ \ }B\neq 0 \end{equation*} \vspace{1pt}is a nontrivial solution corresponding to the eigenvalue $k=0.$ For $k<0,$ let $-k=\alpha ^{2},$ where $\alpha \neq 0.$ Then we have the equation% \begin{equation*} X^{\prime \prime }+\alpha ^{2}X=0 \end{equation*} and \begin{eqnarray*} X\left( x\right) &=&c_{1}\sin \alpha x+c_{2}\cos \alpha x \\ X^{\prime }\left( x\right) &=&c_{1}\alpha \cos \alpha x-c_{2}\alpha \sin \alpha x \end{eqnarray*} \begin{equation*} X^{\prime }\left( 0\right) =c_{1}\alpha =0 \end{equation*} so $c_{1}=0.$% \begin{equation*} X^{\prime }\left( \pi \right) =-c_{2}\alpha \sin \alpha \pi =0 \end{equation*} Therefore $\alpha =n,$ $n=1,2,\ldots $ and the solution is \QTP{Body Math} $\vspace{1pt}$ \begin{center} \begin{equation*} k=-n^{2}\qquad X_{n}\left( x\right) =a_{n}\cos nx\qquad n=1,2,3,\ldots \end{equation*} \end{center} \vspace{1pt}The case $k=0$ implies that the equation for $T$ becomes $% T^{\prime \prime }=0,$ so $T=At+B.$ The initial condition $u\left( x,0\right) =0$ implies $X\left( x\right) T\left( 0\right) =0$ so that $% T\left( 0\right) =0.$ Thus $B=0$ and $T=At$ for $k=0.$ Substituting the values of $k=-n^{2}$ into the equation for $T\left( t\right) $ leads to $\vspace{1pt}$ \begin{equation*} T^{\prime \prime }+\dfrac{n^{2}}{4}T=0 \end{equation*} \vspace{1pt} which has the solution \begin{equation*} T_{n}\left( t\right) =B_{n}\sin \dfrac{nt}{2}+C_{n}\cos \dfrac{nt}{2},\text{ \ \ }n=1,2,3,... \end{equation*} The initial condition $u\left( x,0\right) =0$ implies $X\left( x\right) T\left( 0\right) =0$ so that $T\left( 0\right) =0.$ Thus $c_{n}=0.$ For $n=0$ the equation for $T$ becomes $T^{\prime \prime }=0,$ and has the solution $% T\left( t\right) =B_{0}t+C_{0}.$ The condition $T\left( 0\right) =0$ implies that $C_{0}=0,$ so $T_{0}\left( t\right) =B_{0}t$ We now have the solutions $\vspace{1pt}$ \begin{eqnarray*} u_{n}\left( x,t\right) &=&X_{n}\left( x\right) T_{n}\left( t\right) =A_{n}\cos nx\sin \dfrac{nt}{2}\qquad n=1,2,3,... \\ u_{0}\left( x,t\right) &=&A_{0}t \end{eqnarray*} \vspace{1pt} Since the boundary conditions and the equation are linear and homogeneous, it follows that $\vspace{1pt}$ \begin{equation*} u\left( x,t\right) =\sum_{n=0}^{\infty }u_{n}\left( x,t\right) =A_{0}t+\sum_{n=1}^{\infty }A_{n}\cos nx\sin \dfrac{nt}{2} \end{equation*} \vspace{1pt} satisfies the PDE, the boundary conditions, and the first initial condition. Since \QTP{Body Math} $\vspace{1pt}$ \begin{center} $\vspace{1pt}$% \begin{equation*} u_{t}\left( x,t\right) =A_{0}+\sum_{n=1}^{\infty }A_{n}\left( \dfrac{n}{2}% \right) \cos nx\cos \dfrac{nt}{2} \end{equation*} \end{center} the last initial condition leads to $\vspace{1pt}$ \begin{equation*} u_{t}(x,0)=-9\cos (4x)+16\cos (8x)=A_{0}+\sum_{n=1}^{\infty }A_{n}\left( \dfrac{n}{2}\right) \cos nx. \end{equation*} \vspace{1pt} Matching the cosine terms on both sides of this equation leads to $\vspace{1pt}$ $A_{4}\left( \dfrac{4}{2}\right) =-9$ \ so that $A_{4}=-\frac{9}{2}$ \ and $% A_{8}\left( \dfrac{8}{2}\right) =16$ so that $A_{8}=4.$ All of the other constants must be zero, since there are no cosine terms or constant terms on the left to match with. Thus $\vspace{1pt}$ \begin{equation*} u\left( x,t\right) =-\frac{9}{2}\cos 4x\sin 2t+4\cos 8x\sin 4t \end{equation*} \vspace{1pt} \begin{example} Consider the non-homogeneous problem\qquad \end{example} \vspace{1pt} \begin{eqnarray*} \text{P.D.E.} &:&\ u_{xx}=9u_{t} \\ \text{B.C.}\prime \text{s} &:&\ \ u_{x}(0,t)=0\qquad u(1,t)=2 \\ \text{I.C.} &:&\ \ u(x,0)=-3\cos \frac{7\pi }{2}x+2 \end{eqnarray*} \subparagraph{i) \ } Let \begin{equation*} v(x,t)=u(x,t)-2 \end{equation*} and show that $v(x,t)$ satisfies the\qquad \qquad \qquad \qquad\ homogeneous problem \qquad \qquad \qquad\ \begin{eqnarray*} \text{P.D.E}. &:&v_{xx}=9v_{t} \\ \text{B.C.} &:&\ \ \ v_{x}(0,t)=0\qquad v(1,t)=0 \\ \text{\ I.C.} &:&\ \ \ \ v(x,0)=-3\cos \frac{7\pi }{2}x \end{eqnarray*} \qquad \qquad \qquad\ \vspace{1pt} Solution to i)$\qquad $% \begin{equation*} u_{xx}\left( x,t\right) =v_{xx}\left( x,t\right) \qquad u_{x}\left( x,t\right) =v_{x}\left( x,t\right) \end{equation*} \qquad \qquad \qquad \begin{equation*} u_{tt}\left( x,t\right) =v_{tt}\left( x,t\right) \qquad u_{t}\left( x,t\right) =v_{t}\left( x,t\right) \end{equation*} \qquad \qquad \qquad \begin{equation*} u\left( 1,t\right) =2\text{ and }u\left( x,t\right) -2=v\left( x,t\right) \Longrightarrow v\left( 1,t\right) =0 \end{equation*} \qquad \qquad \qquad \begin{equation*} u_{x}\left( 0,t\right) =0\Longrightarrow v_{x}\left( 0,t\right) =0 \end{equation*}% $\qquad \qquad $ \qquad \qquad \begin{equation*} u\left( x,0\right) =-3\cos \frac{7\pi }{2}+2\ \text{and}\ u\left( x,t\right) -2=v\left( x,t\right) \Longrightarrow v\left( x,0\right) =-3\cos \frac{7\pi }{2} \end{equation*} \vspace{1pt} ii) \ Solve the above problem for $v(x,t)$.\qquad Solution to ii)$\qquad $Let \ $v\left( x,t\right) =X\left( x\right) T\left( t\right) $ \qquad \qquad \qquad then \ \begin{equation*} X^{\prime \prime }T=9XT^{\prime }\Longrightarrow \frac{X^{\prime \prime }}{X}% =9\frac{T^{\prime }}{T}=k \end{equation*} resulting in the ordinary differential equations: \qquad \begin{equation*} X^{\prime \prime }-kX=0\ \ \text{and}\ \ T^{\prime }-\frac{k}{9}T=0 \end{equation*} Boundary Conditions become: \ \ \begin{eqnarray*} X^{\prime }\left( 0\right) T\left( t\right) &=&0\ \text{and}\ X\left( 1\right) T\left( t\right) =0 \\ &\Longrightarrow &X^{\prime }\left( 0\right) =0\ \text{and}\ X\left( 1\right) =1 \end{eqnarray*} \qquad \qquad \qquad \qquad \qquad \qquad Solving the differential equation $X^{\prime \prime }-kX=0$ consider all values of $k$ $k<0$ \ let \ $k=-u^{2};\quad u>0$ $\qquad $% \begin{equation*} X^{\prime \prime }+u^{2}X=0 \end{equation*} \ has the solution: \ \begin{equation*} X\left( x\right) =c_{1}\cos ux+c_{2}\sin ux \end{equation*} \qquad and \ \begin{equation*} X^{\prime }\left( x\right) =-c_{1}u\sin ux+c_{2}u\cos ux \end{equation*} \qquad B.C. $\Longrightarrow X\left( 1\right) =c_{1}\cos u+c_{2}\sin u=0$ \ and \ $X^{\prime }\left( 0\right) =c_{2}u=0$ $\qquad \Longrightarrow c_{2}=0$ \ thus \ $c_{1}\cos u=0$% \begin{equation*} \Longrightarrow u_{n}=\frac{\left( 2n-1\right) \pi }{2}\quad n=1,2,... \end{equation*} \qquad \qquad \qquad \qquad \qquad \qquad \qquad \begin{equation*} \Longrightarrow k_{n}=-\frac{\left( 2n-1\right) ^{2}\pi ^{2}}{4}\quad n=1,2,... \end{equation*} \qquad \qquad \qquad \qquad \qquad \qquad so \ \ \begin{equation*} X_{n}\left( x\right) =c_{n}\cos \frac{\left( 2n-1\right) \pi }{2}x\text{ \ \ }n=1,2,... \end{equation*} \vspace{1pt} The other cases for $k,$ namely $k=0$ and $k>0$ yield only the trivial solution since $k=0\Longrightarrow X^{\prime \prime }=0\quad $which has the solution: \ $% X\left( x\right) =c_{1}x+c_{2}$ and $X^{\prime }\left( x\right) =c_{1}$ \qquad B.C. $\Longrightarrow X\left( 1\right) =c_{1}+c_{2}=0$ \ and \ $% X^{\prime }\left( 0\right) =c_{1}=0\Longrightarrow c_{2}=0$ \qquad thus \ $X\left( x\right) \equiv 0$ \ is the trivial solution. $k>0$ \ let \ $k=u^{2};\quad u>0$ \qquad $X^{\prime \prime }-u^{2}X=0$ \ has the solution: \ $X\left( x\right) =c_{1}e^{ux}+c_{2}e^{-ux}$ \qquad and \ $X^{\prime }\left( x\right) =c_{1}ue^{ux}-c_{2}ue^{-ux}$ \qquad B.C. $\Longrightarrow $ \ $X^{\prime }\left( 0\right) =c_{1}u-c_{2}u=0\Longrightarrow c_{1}=c_{2}$ \qquad and \ \ $X\left( 1\right) =c_{1}e^{u}+c_{2}e^{-u}=0$ \ $% \Longrightarrow c_{1}e^{u}+c_{1}e^{-u}=0\Longrightarrow c_{1}\left( e^{u}+e^{-u}\right) =0$ \qquad $\Longrightarrow c_{1}=c_{2}=0\quad $thus \ $X\left( x\right) \equiv 0 $ \ is the trivial solution. Using the non-trivial solution \ \begin{equation*} k_{n}=-\frac{\left( 2n-1\right) ^{2}\pi ^{2}}{4}\quad \ \ \ \ X_{n}\left( x\right) =c_{n}\cos \frac{\left( 2n-1\right) \pi }{2}x,\text{ \ }n=1,2,... \end{equation*} the equation $\ $% \begin{equation*} T^{\prime }-\frac{k}{9}T=0 \end{equation*} \ becomes \begin{equation*} T^{\prime }+\frac{\left( 2n-1\right) ^{2}\pi ^{2}}{36}T=0 \end{equation*} \qquad solving by separating \ \ \begin{equation*} \frac{T^{\prime }}{T}=-\frac{\left( 2n-1\right) ^{2}\pi ^{2}}{36}% \Longrightarrow \int \frac{T^{\prime }}{T}=-\int \frac{\left( 2n-1\right) ^{2}\pi ^{2}}{36} \end{equation*} $\qquad $% \begin{equation*} \Longrightarrow \ln T=-\frac{\left( 2n-1\right) ^{2}\pi ^{2}}{36}% t+c\Longrightarrow T_{n}\left( t\right) =c_{n}e^{-\frac{\left( 2n-1\right) ^{2}\pi ^{2}}{36}t} \end{equation*} Therefore \ \ \begin{eqnarray*} v_{n}\left( x,t\right) &=&X_{n}\left( x\right) T_{n}\left( t\right) \\ &=&c_{n}\cos \frac{\left( 2n-1\right) \pi x}{2}\,e^{-\frac{\left( 2n-1\right) ^{2}\pi ^{2}}{36}t} \end{eqnarray*} $\qquad $ \qquad so we let \begin{equation*} \ v\left( x,t\right) =\sum\limits_{n=1}^{\infty }c_{n}\cos \frac{\left( 2n-1\right) \pi x}{2}\,e^{-\frac{\left( 2n-1\right) ^{2}\pi ^{2}}{36}t} \end{equation*} Using\ \ I.C. to compute coefficients we have \qquad\ \begin{equation*} v\left( x,0\right) =\sum\limits_{n=1}^{\infty }c_{n}\cos \frac{\left( 2n-1\right) \pi x}{2}\,=-3\cos \frac{7\pi x}{2} \end{equation*} by equating coefficients: $\ c_{1}=0,c_{2}=0,c_{3}=0,c_{4}=-3,c_{4}=0,...$ \begin{equation*} v\left( x,t\right) =-3\cos \frac{7\pi x}{2}\,e^{-\frac{49\pi ^{2}}{36}t} \end{equation*} \ is the solution. \vspace{1pt} iii) \ \ Now use the results of b) i) and ii) to find $u(x,t)$. Solution to iii) \begin{equation*} u\left( x,t\right) =v\left( x.t\right) +2 \end{equation*} so$\qquad $% \begin{equation*} u\left( x,t\right) =-3\cos \frac{7\pi x}{2}\,e^{-\frac{49\pi ^{2}}{36}t}+2 \end{equation*} \vspace{1pt} \qquad $\qquad $ \end{document} %%%%%%%%%%%%%%%%%%%%%%% End /document/Chap11.tex %%%%%%%%%%%%%%%%%%%%%% %%%%%%%%%%%%%%%%%%%%% Start /document/webmath.cst %%%%%%%%%%%%%%%%%%%%% [FILTER] \newtheorem{theorem}{Theorem} \newtheorem{lemma}[theorem]{Lemma} \newtheorem{proposition}[theorem]{Proposition} \newtheorem{corollary}[theorem]{Corollary} \newtheorem{conjecture}{Conjecture} \newtheorem{example}{Example} \newtheorem{definition}{Definition} \newtheorem{remark}{Remark} \newtheorem{exercise}{Exercise} \newtheorem{axiom}{Axiom} \TCIEnd [Defaults] STYLE_NAME=Article ITALICS_MODE=2 NORMAL_SLANT=0 ITALIC_SLANT=1 NORMAL_WEIGHT=400 BOLD_WEIGHT=700 FONT_FACE=Times New Roman FONT_SIZE=12 TEXT_COLOR=R0,G0,B0 FONT_WEIGHT=400 FONT_SLANT=0 MATH_FACE=Times New Roman MATH_COLOR=R255,G0,B0 FUNCTION_COLOR=R128,G128,B128 ITALICS_IN_MATH=1 SCRIPT_SIZE=70 SCRIPTSCRIPT_SIZE=50 OPERATOR_SIZE=120 BIGOPERATOR_SIZE=150 PARAGRAPH_INDENT_RIGHT=18 PARAGRAPH_INDENT_LEFT=18 PARAGRAPH_INDENT_FIRST=0 PARAGRAPH_INDENT_REST=0 PARAGRAPH_LEADING_BEFORE=0 PARAGRAPH_LEADING_AFTER=0 LINE_LEADING=0 LINE_JUSTIFICATION=1 LINE_LEADING=0 TAGLEVEL=3 LISTBULLETING=BIGBULLET,BIGBULLET,BIGBULLET,BIGBULLET LISTNUMBERING=ARABIC,LOWERCASE,SMALLROMAN,LOWERCASE [Body Text] TAG_TYPE=PARA FONT_FACE=Times New Roman PARAGRAPH_LEADING_BEFORE=1 PARAGRAPH_LEADING_AFTER=1 NEXT_TAG=Body Text BKGROUND_COLOR=R:255 G:255 B:255 PARAGRAPH_INDENT_RIGHT=0 PARAGRAPH_INDENT_LEFT=0 PARAGRAPH_INDENT_FIRST=0 PARAGRAPH_INDENT_REST=0 LINE_LEADING=0 LINE_JUSTIFICATION=1 TAGBAR_DELETE=0 FONT_SIZE=10 [Body Math] TAG_TYPE=PARA TAG_BEHAVIOR=FORCESMATH NEXT_TAG=Body Math TEXT_COLOR=R:128 G:0 B:128 BKGROUND_COLOR=R:255 G:255 B:255 PARAGRAPH_INDENT_RIGHT=0 PARAGRAPH_INDENT_LEFT=0 PARAGRAPH_INDENT_FIRST=0 PARAGRAPH_INDENT_REST=0 PARAGRAPH_LEADING_BEFORE=0 PARAGRAPH_LEADING_AFTER=0 LINE_LEADING=0 LINE_JUSTIFICATION=1 TAGBAR_DELETE=0 FONT_SIZE=10 [section] ALIAS=Heading 1 TAGLEVEL=1 TAG_TYPE=STRUCTURE NEXT_TAG=Body Text TAG_BEHAVIOR=NOENVTAGS FONT_FACE=Times New Roman FONT_WEIGHT=700 FONT_SIZE=18 TEXT_COLOR=R:255 G:255 B:255 PARAGRAPH_LEADING_BEFORE=7 PARAGRAPH_LEADING_AFTER=7 BKGROUND_COLOR=R:145 G:30 B:80 PARAGRAPH_INDENT_RIGHT=0 PARAGRAPH_INDENT_LEFT=0 LINE_JUSTIFICATION=2 TAGBAR_DELETE=0 FONT_SLANT=0 PARAGRAPH_INDENT_FIRST=0 PARAGRAPH_INDENT_REST=0 LINE_LEADING=0 [subsection] ALIAS=Heading 2 TAGLEVEL=2 TAG_TYPE=STRUCTURE NEXT_TAG=Body Text TAG_BEHAVIOR=NOENVTAGS FONT_FACE=Times New Roman FONT_WEIGHT=700 FONT_SIZE=16 TEXT_COLOR=R:255 G:255 B:255 PARAGRAPH_LEADING_BEFORE=4 PARAGRAPH_LEADING_AFTER=1 BKGROUND_COLOR=R:145 G:30 B:80 PARAGRAPH_INDENT_FIRST=0 PARAGRAPH_INDENT_REST=0 FONT_SLANT=0 PARAGRAPH_INDENT_RIGHT=0 PARAGRAPH_INDENT_LEFT=0 LINE_LEADING=0 LINE_JUSTIFICATION=1 TAGBAR_DELETE=0 [subsubsection] ALIAS=Heading 3 TAGLEVEL=3 TAG_TYPE=STRUCTURE NEXT_TAG=Body Text TAG_BEHAVIOR=NOENVTAGS FONT_FACE=Times New Roman FONT_WEIGHT=700 TEXT_COLOR=R:25 G:0 B:190 PARAGRAPH_LEADING_BEFORE=4 PARAGRAPH_LEADING_AFTER=1 FONT_SIZE=14 BKGROUND_COLOR=R:255 G:255 B:255 FONT_SLANT=0 PARAGRAPH_INDENT_RIGHT=0 PARAGRAPH_INDENT_LEFT=0 PARAGRAPH_INDENT_FIRST=0 PARAGRAPH_INDENT_REST=0 LINE_LEADING=0 LINE_JUSTIFICATION=1 TAGBAR_DELETE=0 [paragraph] ALIAS=Heading 4 TAGLEVEL=4 TAG_TYPE=STRUCTURE NEXT_TAG=Body Text TAG_BEHAVIOR=NOENVTAGS TEXT_COLOR=R:0 G:128 B:128 PARAGRAPH_LEADING_BEFORE=3 PARAGRAPH_LEADING_AFTER=1 FONT_FACE=Times New Roman BKGROUND_COLOR=R:255 G:255 B:255 FONT_SIZE=12 FONT_WEIGHT=700 FONT_SLANT=0 PARAGRAPH_INDENT_RIGHT=0 PARAGRAPH_INDENT_LEFT=0 PARAGRAPH_INDENT_FIRST=0 PARAGRAPH_INDENT_REST=0 LINE_LEADING=0 LINE_JUSTIFICATION=1 TAGBAR_DELETE=0 [subparagraph] ALIAS=Heading 5 TAGLEVEL=5 TAG_TYPE=STRUCTURE NEXT_TAG=Body Text TAG_BEHAVIOR=NOENVTAGS TEXT_COLOR=R:0 G:128 B:128 FONT_FACE=Times New Roman BKGROUND_COLOR=R:255 G:255 B:255 FONT_SIZE=12 FONT_SLANT=0 PARAGRAPH_INDENT_RIGHT=0 PARAGRAPH_INDENT_LEFT=0 PARAGRAPH_INDENT_FIRST=0 PARAGRAPH_INDENT_REST=0 PARAGRAPH_LEADING_BEFORE=1 PARAGRAPH_LEADING_AFTER=1 LINE_LEADING=0 LINE_JUSTIFICATION=1 TAGBAR_DELETE=0 [em] ALIAS=Emphasized TAG_TYPE=TEXT FONT_SLANT=0 TEXT_COLOR=R:25 G:0 B:220 MATH_COLOR=R:255 G:0 B:255 FONT_FACE=Times New Roman FUNCTION_COLOR=R:128 G:128 B:128 BKGROUND_COLOR=R:255 G:255 B:255 FONT_SIZE=10 FONT_WEIGHT=400 TAGBAR_DELETE=0 [rm] ALIAS=Roman TAG_TYPE=TEXT TAGBAR_DELETE=1 FONT_SIZE=10 [bf] ALIAS=Bold TAG_TYPE=TEXT FONT_WEIGHT=700 FONT_SIZE=10 FONT_SLANT=0 TAGBAR_DELETE=0 [it] ALIAS=Italics TAG_TYPE=TEXT FONT_SLANT=1 MATH_COLOR=R:0 G:255 B:255 FONT_SIZE=10 FONT_WEIGHT=400 TAGBAR_DELETE=0 [sl] ALIAS=Strongly Emphasized TAG_TYPE=TEXT FONT_FACE=Times New Roman FONT_SLANT=1 TEXT_COLOR=R:0 G:128 B:128 FONT_WEIGHT=700 FONT_SIZE=10 TAGBAR_DELETE=0 [sf] ALIAS=Sample Text TAG_TYPE=TEXT FONT_FACE=Times New Roman MATH_COLOR=R:0 G:0 B:0 TEXT_COLOR=R:0 G:128 B:128 ITALICS_IN_MATH=1 FONT_WEIGHT=700 FONT_SLANT=0 TAGBAR_DELETE=0 FONT_SIZE=10 [sc] ALIAS=Keyboard Input TAG_TYPE=TEXT TAG_BEHAVIOR=FORCESTEXT FONT_FACE=Times New Roman Bold TEXT_COLOR=R:0 G:128 B:128 MATH_COLOR=R:0 G:128 B:0 FONT_SIZE=9 TAGBAR_DELETE=0 [tt] ALIAS=Typewriter TAG_TYPE=TEXT FONT_FACE=Courier New MATH_COLOR=R:0 G:0 B:0 FONT_WEIGHT=700 FONT_SLANT=0 FONT_SIZE=10 TAGBAR_DELETE=0 [green] ALIAS=Green TAG_TYPE=TEXT FONT_FACE=Times New Roman Bold TEXT_COLOR=R:0 G:180 B:0 FONT_WEIGHT=700 FONT_SLANT=0 FONT_SIZE=10 TAGBAR_DELETE=0 [brown] ALIAS=Brown TAG_TYPE=TEXT FONT_FACE=Times New Roman TEXT_COLOR=R:153 G:102 B:0 FONT_WEIGHT=700 FONT_SLANT=0 FONT_SIZE=10 TAGBAR_DELETE=0 [blue] ALIAS=Blue TAG_TYPE=TEXT FONT_FACE=Times New Roman TEXT_COLOR=R:0 G:0 B:220 FONT_WEIGHT=700 FONT_SLANT=0 FONT_SIZE=10 TAGBAR_DELETE=0 [red] ALIAS=Red TAG_TYPE=TEXT FONT_FACE=Times New Roman TEXT_COLOR=R:255 G:0 B:0 FONT_WEIGHT=700 FONT_SLANT=0 FONT_SIZE=10 TAGBAR_DELETE=0 [purple] ALIAS=purple TAG_TYPE=TEXT FONT_FACE=Times New Roman TEXT_COLOR=R:190 G:0 B:240 FONT_WEIGHT=700 FONT_SLANT=0 FONT_SIZE=10 TAGBAR_DELETE=0 [yellow] ALIAS=Yellow TAG_TYPE=TEXT FONT_FACE=Times New Roman TEXT_COLOR=R:225 G:225 B:0 FONT_WEIGHT=700 FONT_SLANT=0 FONT_SIZE=10 TAGBAR_DELETE=0 [yellowbig] ALIAS=Yellowbig TAG_TYPE=TEXT FONT_FACE=Times New Roman TEXT_COLOR=R:225 G:225 B:0 FONT_WEIGHT=700 FONT_SLANT=0 FONT_SIZE=18 TAGBAR_DELETE=0 [greenbig] ALIAS=Greenbig TAG_TYPE=TEXT FONT_FACE=Times New Roman Bold TEXT_COLOR=R:0 G:180 B:0 FONT_WEIGHT=700 FONT_SLANT=0 FONT_SIZE=18 TAGBAR_DELETE=0 [brownbig] ALIAS=Brownbig TAG_TYPE=TEXT FONT_FACE=Times New Roman TEXT_COLOR=R:153 G:102 B:0 FONT_WEIGHT=700 FONT_SLANT=0 FONT_SIZE=18 TAGBAR_DELETE=0 [bluebig] ALIAS=Bluebig TAG_TYPE=TEXT FONT_FACE=Times New Roman TEXT_COLOR=R:0 G:0 B:225 FONT_WEIGHT=700 FONT_SLANT=0 FONT_SIZE=18 TAGBAR_DELETE=0 [redbig] ALIAS=Redbig TAG_TYPE=TEXT FONT_FACE=Times New Roman TEXT_COLOR=R:255 G:0 B:0 FONT_WEIGHT=700 FONT_SLANT=0 FONT_SIZE=18 TAGBAR_DELETE=0 [purplebig] ALIAS=purplebig TAG_TYPE=TEXT FONT_FACE=Times New Roman TEXT_COLOR=R:204 G:0 B:255 FONT_WEIGHT=700 FONT_SLANT=0 FONT_SIZE=18 TAGBAR_DELETE=0 [description] ALIAS=Description List Item TAG_TYPE=LISTENV TAG_BEHAVIOR=NOSTRUCTURETAGS LEADIN_BACKGROUND_COLOR=R:145 G:30 B:80 LEADIN_FONT_FACE=Times New Roman LEADIN_TEXT_COLOR=R:255 G:255 B:255 LEADIN_FONT_WEIGHT=700 FONT_FACE=Times New Roman PARAGRAPH_LEADING_BEFORE=4 PARAGRAPH_LEADING_AFTER=4 BKGROUND_COLOR=R:255 G:255 B:255 PARAGRAPH_INDENT_LEFT=36 PARAGRAPH_INDENT_FIRST=-35 PARAGRAPH_LEADING_BEFORE=4 PARAGRAPH_LEADING_AFTER=4 PARAGRAPH_INDENT_REST=0 LEADIN_ALIGNLEFT=1 FONT_SIZE=10 LEADIN_FONT_SLANT=0 LINE_LEADING=0 LINE_JUSTIFICATION=1 TAGBAR_DELETE=0 [enumerate] ALIAS=Numbered List Item TAG_TYPE=LISTENV TAG_LEADIN=# TAG_BEHAVIOR=NOSTRUCTURETAGS PARAGRAPH_INDENT_LEFT=25 PARAGRAPH_INDENT_FIRST=0 NEXT_TAG=Numbered item FONT_FACE=Times New Roman LEADIN_BACKGROUND_COLOR= R:128 G:0 B:56 LEADIN_FONT_FACE=Times New Roman LEADIN_TEXT_COLOR=R:255 G:255 B:255 LEADIN_FONT_WEIGHT=700 LEADIN_LABELSEP=4 LEADIN_FONT_SIZE=11 BKGROUND_COLOR=R:255 G:255 B:255 LEADIN_FONT_SLANT=0 PARAGRAPH_INDENT_RIGHT=25 PARAGRAPH_INDENT_REST=0 PARAGRAPH_LEADING_BEFORE=1 PARAGRAPH_LEADING_AFTER=1 LINE_LEADING=0 LINE_JUSTIFICATION=1 FONT_SIZE=10 LEADIN_LABELWIDTH=14 LEADIN_TEXTJUSTIFY=2 TAGBAR_DELETE=0 [itemize] ALIAS=Bullet List Item TAG_TYPE=LISTENV TAG_LEADIN=· TAG_BEHAVIOR=NOSTRUCTURETAGS PARAGRAPH_INDENT_LEFT=18 PARAGRAPH_INDENT_FIRST=0 NEXT_TAG=Bullet item FONT_FACE=Times New Roman BKGROUND_COLOR=R:255 G:255 B:255 PARAGRAPH_INDENT_RIGHT=18 PARAGRAPH_INDENT_REST=0 PARAGRAPH_LEADING_BEFORE=1 PARAGRAPH_LEADING_AFTER=1 LINE_LEADING=0 LINE_JUSTIFICATION=1 LEADIN_LABELSEP=4 LEADIN_TEXT_COLOR=R:145 G:30 B:80 FONT_SIZE=10 LEADIN_FONT_SIZE=8 LEADIN_LABELWIDTH=14 LEADIN_TEXTJUSTIFY=2 TAGBAR_DELETE=0 [cal] ALIAS=Calligraphic TAG_TYPE=TEXT TAG_BEHAVIOR=FORCESMATH FONT_FACE=Times New Roman TEXT_COLOR=R:192 G:192 B:192 MATH_COLOR=R:255 G:0 B:255 TAGBAR_DELETE=1 FONT_SIZE=10 [newtheorem] TAG_TYPE=TLTEMPLATE PARAGRAPH_INDENT_LEFT=36 PARAGRAPH_INDENT_FIRST=0 TAG_BEHAVIOR=LISTSTART [theorem] ALIAS=Theorem TAG_TYPE=THEOREMENV TAG_LEADIN=Theorem BKGROUND_COLOR=R:255 G:255 B:255 PARAGRAPH_INDENT_RIGHT=36 PARAGRAPH_INDENT_LEFT=36 PARAGRAPH_INDENT_FIRST=0 PARAGRAPH_INDENT_REST=0 PARAGRAPH_LEADING_BEFORE=0 PARAGRAPH_LEADING_AFTER=0 LINE_LEADING=0 LINE_JUSTIFICATION=1 TAGBAR_DELETE=0 TAG_BEHAVIOR=LISTSTART FONT_SIZE=10 [lemma] ALIAS=Lemma TAG_TYPE=THEOREMENV TAG_LEADIN=Lemma BKGROUND_COLOR=R:255 G:255 B:255 PARAGRAPH_INDENT_RIGHT=36 PARAGRAPH_INDENT_LEFT=36 PARAGRAPH_INDENT_FIRST=0 PARAGRAPH_INDENT_REST=0 PARAGRAPH_LEADING_BEFORE=0 PARAGRAPH_LEADING_AFTER=0 LINE_LEADING=0 LINE_JUSTIFICATION=1 TAG_BEHAVIOR=LISTSTART FONT_SIZE=10 TAGBAR_DELETE=0 [proposition] ALIAS=Proposition TAG_TYPE=THEOREMENV TAG_LEADIN=Proposition BKGROUND_COLOR=R:255 G:255 B:255 PARAGRAPH_INDENT_RIGHT=36 PARAGRAPH_INDENT_LEFT=36 PARAGRAPH_INDENT_FIRST=0 PARAGRAPH_INDENT_REST=0 PARAGRAPH_LEADING_BEFORE=0 PARAGRAPH_LEADING_AFTER=0 LINE_LEADING=0 LINE_JUSTIFICATION=1 TAG_BEHAVIOR=LISTSTART FONT_SIZE=10 TAGBAR_DELETE=0 [corollary] ALIAS=Corollary TAG_TYPE=THEOREMENV TAG_LEADIN=Corollary BKGROUND_COLOR=R:255 G:255 B:255 PARAGRAPH_INDENT_RIGHT=36 PARAGRAPH_INDENT_LEFT=36 PARAGRAPH_INDENT_FIRST=0 PARAGRAPH_INDENT_REST=0 PARAGRAPH_LEADING_BEFORE=0 PARAGRAPH_LEADING_AFTER=0 LINE_LEADING=0 LINE_JUSTIFICATION=1 TAG_BEHAVIOR=LISTSTART FONT_SIZE=10 TAGBAR_DELETE=0 [conjecture] ALIAS=Conjecture TAG_TYPE=THEOREMENV TAG_LEADIN=Conjecture BKGROUND_COLOR=R:255 G:255 B:255 PARAGRAPH_INDENT_RIGHT=36 PARAGRAPH_INDENT_LEFT=36 PARAGRAPH_INDENT_FIRST=0 PARAGRAPH_INDENT_REST=0 PARAGRAPH_LEADING_BEFORE=0 PARAGRAPH_LEADING_AFTER=0 LINE_LEADING=0 LINE_JUSTIFICATION=1 TAG_BEHAVIOR=LISTSTART FONT_SIZE=10 TAGBAR_DELETE=0 [example] ALIAS=Example TAG_TYPE=THEOREMENV TAG_LEADIN=Example BKGROUND_COLOR=R:255 G:255 B:255 PARAGRAPH_INDENT_LEFT=18 PARAGRAPH_INDENT_FIRST=-16 PARAGRAPH_INDENT_REST=0 PARAGRAPH_LEADING_BEFORE=0 PARAGRAPH_LEADING_AFTER=0 LINE_LEADING=0 LINE_JUSTIFICATION=1 TAG_BEHAVIOR=LISTSTART LEADIN_TEXT_COLOR= R:255 G:255 B:255 LEADIN_BACKGROUND_COLOR=R:145 G:30 B:80 LEADIN_FONT_WEIGHT=700 LEADIN_FONT_SLANT=0 TAGBAR_DELETE=0 LEADIN_ALIGNLEFT=1 FONT_SIZE=10 [definition] ALIAS=Definition TAG_TYPE=THEOREMENV TAG_LEADIN=Definition BKGROUND_COLOR=R:255 G:255 B:255 PARAGRAPH_INDENT_RIGHT=36 PARAGRAPH_INDENT_LEFT=36 PARAGRAPH_INDENT_FIRST=0 PARAGRAPH_INDENT_REST=0 PARAGRAPH_LEADING_BEFORE=0 PARAGRAPH_LEADING_AFTER=0 LINE_LEADING=0 LINE_JUSTIFICATION=1 TAG_BEHAVIOR=LISTSTART FONT_SIZE=10 TAGBAR_DELETE=0 LEADIN_ALIGNLEFT=1 FONT_SIZE=10 [remark] ALIAS=Remark TAG_TYPE=THEOREMENV TAG_LEADIN=Remark BKGROUND_COLOR=R:255 G:255 B:255 PARAGRAPH_INDENT_RIGHT=36 PARAGRAPH_INDENT_LEFT=36 PARAGRAPH_INDENT_FIRST=0 PARAGRAPH_INDENT_REST=0 PARAGRAPH_LEADING_BEFORE=0 PARAGRAPH_LEADING_AFTER=0 LINE_LEADING=0 LINE_JUSTIFICATION=1 TAG_BEHAVIOR=LISTSTART FONT_SIZE=10 TAGBAR_DELETE=0 [exercise] ALIAS=Exercise TAG_TYPE=THEOREMENV TAG_LEADIN=Exercise BKGROUND_COLOR=R:255 G:255 B:255 PARAGRAPH_INDENT_RIGHT=36 PARAGRAPH_INDENT_LEFT=36 PARAGRAPH_INDENT_FIRST=0 PARAGRAPH_INDENT_REST=0 PARAGRAPH_LEADING_BEFORE=0 PARAGRAPH_LEADING_AFTER=0 LINE_LEADING=0 LINE_JUSTIFICATION=1 TAG_BEHAVIOR=LISTSTART FONT_SIZE=10 TAGBAR_DELETE=0 [axiom] ALIAS=Axiom TAG_TYPE=THEOREMENV TAG_LEADIN=Axiom BKGROUND_COLOR=R:255 G:255 B:255 PARAGRAPH_INDENT_RIGHT=36 PARAGRAPH_INDENT_LEFT=36 PARAGRAPH_INDENT_FIRST=0 PARAGRAPH_INDENT_REST=0 PARAGRAPH_LEADING_BEFORE=0 PARAGRAPH_LEADING_AFTER=0 LINE_LEADING=0 LINE_JUSTIFICATION=1 TAG_BEHAVIOR=LISTSTART FONT_SIZE=10 TAGBAR_DELETE=0 [newenvironment] TAG_TYPE=TLTEMPLATE PARAGRAPH_INDENT_LEFT=36 PARAGRAPH_INDENT_FIRST=0 PARAGRAPH_INDENT_REST=18 TAG_BEHAVIOR=LISTSTART [tiny_big] TAG_TYPE=TEXT FONT_SIZE=7 TEXT_COLOR=R255,G0,B255 TAGBAR_DELETE=1 [tiny] TAG_TYPE=TEXT FONT_SIZE=6 TEXT_COLOR=R255,G0,B255 TAGBAR_DELETE=1 [scriptsize] TAG_TYPE=TEXT FONT_SIZE=8 TAGBAR_DELETE=1 [footnotesize] TAG_TYPE=TEXT FONT_SIZE=9 TAGBAR_DELETE=1 [small] TAG_TYPE=TEXT ALIAS=Smaller FONT_SIZE=9 TAGBAR_DELETE=0 [normalsize] TAG_TYPE=TEXT FONT_SIZE=10 TEXT_COLOR=R:0 G:128 B:128 TAGBAR_DELETE=1 [large] TAG_TYPE=TEXT FONT_SIZE=14 TEXT_COLOR=R255,G0,B255 TAGBAR_DELETE=1 [Large] TAG_TYPE=TEXT ALIAS=Bigger FONT_SIZE=16 [LARGE] TAG_TYPE=TEXT FONT_SIZE=16 TAGBAR_DELETE=1 [huge] TAG_TYPE=TEXT FONT_SIZE=20 TEXT_COLOR=R255,G0,B255 TAGBAR_DELETE=1 [Huge] TAG_TYPE=TEXT FONT_SIZE=22 TEXT_COLOR=R255,G0,B255 TAGBAR_DELETE=1 [thebibliography] ALIAS=Bibliography item TAG_TYPE=BIBENV TAG_LEADIN=bibitem TAG_BEHAVIOR=NOSTRUCTURETAGS|NOMULTIPARA PARAGRAPH_INDENT_LEFT=18 PARAGRAPH_INDENT_FIRST=0 PARAGRAPH_INDENT_REST=18 TAGBAR_DELETE=1 FONT_SIZE=10 PARAGRAPH_LEADING_BEFORE=0 PARAGRAPH_LEADING_AFTER=0 LINE_LEADING=0 LINE_JUSTIFICATION=1 [quotation] TAG_TYPE=PARA ALIAS=Block Quote PARAGRAPH_INDENT_RIGHT=24 PARAGRAPH_INDENT_LEFT=24 BKGROUND_COLOR=R:240 G:240 B:240 PARAGRAPH_INDENT_FIRST=0 PARAGRAPH_INDENT_REST=0 PARAGRAPH_LEADING_BEFORE=0 PARAGRAPH_LEADING_AFTER=0 LINE_LEADING=0 LINE_JUSTIFICATION=1 FONT_FACE=Times New Roman NEXT_TAG=Body Text FONT_SIZE=10 TAGBAR_DELETE=0 [quote] TAG_TYPE=PARA ALIAS=Body Quote PARAGRAPH_INDENT_LEFT=0 NEXT_TAG=Body Quote BKGROUND_COLOR=R:255 G:255 B:255 PARAGRAPH_INDENT_FIRST=-11 PARAGRAPH_INDENT_REST=0 PARAGRAPH_LEADING_BEFORE=7 PARAGRAPH_LEADING_AFTER=4 LINE_LEADING=0 LINE_JUSTIFICATION=1 FONT_FACE=Times New Roman FONT_WEIGHT=700 FONT_SLANT=1 TAGBAR_DELETE=0 TEXT_COLOR=R:0 G:128 B:128 FONT_SIZE=10 PARAGRAPH_INDENT_RIGHT=0 [center] TAG_TYPE=PARA ALIAS=Body Center LINE_JUSTIFICATION=2 FONT_FACE=Times New Roman NEXT_TAG=Body Center FONT_SIZE=10 PARAGRAPH_INDENT_LEFT=18 PARAGRAPH_INDENT_FIRST=0 PARAGRAPH_INDENT_REST=0 PARAGRAPH_LEADING_BEFORE=0 PARAGRAPH_LEADING_AFTER=0 LINE_LEADING=0 TAGBAR_DELETE=0 BKGROUND_COLOR=R:255 G:255 B:255 [verbatim] TAG_TYPE=PARA ALIAS=Preformatted PARAGRAPH_INDENT_RIGHT=36 PARAGRAPH_INDENT_LEFT=36 TAG_BEHAVIOR=FORCESTEXT|NOLINEBREAK|PRESERVESPACES FONT_FACE=Courier New FONT_SIZE=10 NEXT_TAG=Body Verbatim BKGROUND_COLOR=R:255 G:255 B:255 PARAGRAPH_INDENT_FIRST=0 PARAGRAPH_INDENT_REST=0 PARAGRAPH_LEADING_BEFORE=0 PARAGRAPH_LEADING_AFTER=0 LINE_LEADING=0 LINE_JUSTIFICATION=1 [newfield] TAG_TYPE=TLTEMPLATE PARAGRAPH_INDENT_LEFT=70 PARAGRAPH_INDENT_FIRST=0 [f_0] ALIAS=Title TAG_TYPE=FIELD TAG_LEADIN=Title: TAG_BEHAVIOR=FRONTMATTER|NOSTRUCTURETAGS|NOENVTAGS PARAGRAPH_INDENT_LEFT=35 PARAGRAPH_INDENT_FIRST=0 [f_1] ALIAS=Author TAG_TYPE=FIELD TAG_LEADIN=Author: TAG_BEHAVIOR=FRONTMATTER|NOSTRUCTURETAGS|NOENVTAGS PARAGRAPH_INDENT_LEFT=35 PARAGRAPH_INDENT_FIRST=0 [f_2] ALIAS=Address TAG_TYPE=FIELD TAG_LEADIN=Address: TAG_BEHAVIOR=FRONTMATTER|NOSTRUCTURETAGS|NOENVTAGS PARAGRAPH_INDENT_LEFT=35 PARAGRAPH_INDENT_FIRST=0 [f_7] ALIAS=Date TAG_TYPE=FIELD TAG_LEADIN=Date: TAG_BEHAVIOR=FRONTMATTER|NOSTRUCTURETAGS|NOENVTAGS PARAGRAPH_INDENT_LEFT=35 PARAGRAPH_INDENT_FIRST=0 [abstract] TAG_TYPE=FIELD TAG_LEADIN=Abstract: TAG_BEHAVIOR=FRONTMATTER|NOSTRUCTURETAGS PARAGRAPH_INDENT_LEFT=35 PARAGRAPH_INDENT_FIRST=0 [f_11] ALIAS=Make Title TAG_TYPE=FIELD TAG_LEADIN=Make Title TAG_BEHAVIOR=FRONTMATTER|NOSTRUCTURETAGS|NOENVTAGS PARAGRAPH_INDENT_LEFT=35 PARAGRAPH_INDENT_FIRST=0 [f_12] ALIAS=Make LOF TAG_TYPE=FIELD TAG_LEADIN=Make LOF TAG_BEHAVIOR=FRONTMATTER|NOSTRUCTURETAGS|NOENVTAGS PARAGRAPH_INDENT_LEFT=35 PARAGRAPH_INDENT_FIRST=0 [f_13] ALIAS=Make LOT TAG_TYPE=FIELD TAG_LEADIN=Make LOT TAG_BEHAVIOR=FRONTMATTER|NOSTRUCTURETAGS|NOENVTAGS PARAGRAPH_INDENT_LEFT=35 PARAGRAPH_INDENT_FIRST=0 [f_14] ALIAS=Make TOC TAG_TYPE=FIELD TAG_LEADIN=Make TOC TAG_BEHAVIOR=FRONTMATTER|NOSTRUCTURETAGS|NOENVTAGS PARAGRAPH_INDENT_LEFT=35 PARAGRAPH_INDENT_FIRST=0 [frak] ALIAS=Fraktur TAG_TYPE=TEXT TAG_BEHAVIOR=FORCESMATH FONT_SLANT=1 TEXT_COLOR=R:192 G:192 B:192 MATH_COLOR=R:255 G:0 B:255 TAGBAR_DELETE=1 FONT_SIZE=10 FONT_WEIGHT=400 [Bbb] ALIAS=Blackboard bold TAG_TYPE=TEXT TAG_BEHAVIOR=FORCESMATH TEXT_COLOR=R:192 G:192 B:192 TAGBAR_DELETE=1 FONT_SIZE=10 [newlist] TAG_TYPE=TLTEMPLATE PARAGRAPH_INDENT_LEFT=70 PARAGRAPH_INDENT_FIRST=0 [Hyperlink] TAG_TYPE=INTERNAL TEXT_COLOR=R0,G128,B0 MATH_COLOR=R0,G128,B0 [ProgramCall] TAG_TYPE=INTERNAL TEXT_COLOR=R128,G0,B0 MATH_COLOR=R128,G0,B0 [ProgramCall] TAG_TYPE=INTERNAL TEXT_COLOR=R128,G0,B0 MATH_COLOR=R128,G0,B0 [Margin_Hint] ALIAS=Margin Hint TAG_TYPE=NOTE ICON_NAME=MarginHint ICON_SIZE=0.49in,0.53in,0.00in FONT_SIZE=10 PARAGRAPH_INDENT_LEFT=18 PARAGRAPH_INDENT_FIRST=0 PARAGRAPH_INDENT_REST=0 PARAGRAPH_LEADING_BEFORE=0 PARAGRAPH_LEADING_AFTER=0 LINE_LEADING=0 LINE_JUSTIFICATION=1 TAGBAR_DELETE=0 [Solution_Note] ALIAS=Solution Note TAG_TYPE=NOTE ICON_NAME=Solution ICON_SIZE=0.23in,0.26in,0.00in FONT_SIZE=10 PARAGRAPH_INDENT_LEFT=18 PARAGRAPH_INDENT_FIRST=0 PARAGRAPH_INDENT_REST=0 PARAGRAPH_LEADING_BEFORE=0 PARAGRAPH_LEADING_AFTER=0 LINE_LEADING=0 LINE_JUSTIFICATION=1 TAGBAR_DELETE=0 [Proof_Theorem] ALIAS=Proof TAG_TYPE=PROOF ICON_NAME=Proof ICON_SIZE=0.23in,0.26in,0.00in FONT_SIZE=10 PARAGRAPH_INDENT_LEFT=18 PARAGRAPH_INDENT_FIRST=0 PARAGRAPH_INDENT_REST=0 PARAGRAPH_LEADING_BEFORE=0 PARAGRAPH_LEADING_AFTER=0 LINE_LEADING=0 LINE_JUSTIFICATION=1 TAGBAR_DELETE=0 [Prob_Solv_Hint] ALIAS=Problem Solving Hint TAG_TYPE=NOTE ICON_NAME=ProbSolvHint ICON_SIZE=0.51in,0.64in,0.00in FONT_SIZE=10 PARAGRAPH_INDENT_LEFT=18 PARAGRAPH_INDENT_FIRST=0 PARAGRAPH_INDENT_REST=0 PARAGRAPH_LEADING_BEFORE=0 PARAGRAPH_LEADING_AFTER=0 LINE_LEADING=0 LINE_JUSTIFICATION=1 TAGBAR_DELETE=0 [Note] TAG_TYPE=NOTE ICON_NAME=Note ICON_SIZE=0.50in,0.51in,0.00in FONT_SIZE=10 PARAGRAPH_INDENT_LEFT=18 PARAGRAPH_INDENT_FIRST=0 PARAGRAPH_INDENT_REST=0 PARAGRAPH_LEADING_BEFORE=0 PARAGRAPH_LEADING_AFTER=0 LINE_LEADING=0 LINE_JUSTIFICATION=1 TAGBAR_DELETE=0 [Answer] ALIAS=AnswerNote TAG_TYPE=NOTE ICON_NAME=Answer ICON_SIZE=0.26in,0.26in,0.00in FONT_SIZE=10 PARAGRAPH_INDENT_LEFT=18 PARAGRAPH_INDENT_FIRST=0 PARAGRAPH_INDENT_REST=0 PARAGRAPH_LEADING_BEFORE=0 PARAGRAPH_LEADING_AFTER=0 LINE_LEADING=0 LINE_JUSTIFICATION=1 TAGBAR_DELETE=0 [FKeys] 0x0001=(Remove Item Tag) 0x0002=Body Text 0x0003=(Normal) 0x0004=bf 0x0005=em 0x0006=enumerate 0x0007=itemize 0x0008=cal 0x000a=section 0x000b=subsection %%%%%%%%%%%%%%%%%%%%%% End /document/webmath.cst %%%%%%%%%%%%%%%%%%%%%% %%%%%%%%%%%%%%%%% Start /document/graphics/Image10.gif %%%%%%%%%%%%%%%% GedQxdSXBDPs@@H@@@@@@|sB@@@@@BDPs@@`@~sxcin\{OLJgtj}bszMo{`abcdefghijkl mnopqrstuvwxyz{|}~@CJ\HqbFObLireL[z|IthRgRmjukXsj]Kwn^BNlxqdK[~LztjWsnm{w pcK_~K@toSrN~}_tsoF~Ax`UHHaGBaaHnxBJrxcFxHdLJydHVieDbYf@nIg|yygxEZWdJJCfnU jgfpjYyJkQjf]@@khvZUmrFkDxVe_~ZYpEU[qM@Lo}bUqdb\rI~knE^ppPVMNLgQvOC]SFsMKWG sw`wbxfK]yuYTzjCsygPn{cGT{sKC|bX}xws~|wO]|O@Fp[LPBVpZdODvaE^aCrpD~NGFQNPQe UQ^\q~{VFwqF]pG{hv^gH}wzJiFQGJMyJByJGiJ\yKqPHaxLiqGeyMiY|vyOnhN}yCKQPEjE\XQ MzttgRIS[jO`WRaJENwTizDDwUqJFmHW]jWZTCkXYILKkYGJZGKOWK[Uk[SI\ckGgK]ExQo{[ sj^S[DvZ__Pj@\`AI_Kl\KbSl`WLcOfc_Ljb\][LeqkeoLvrlfMha{\NpDaA]k`Liw|dOmg[dj QmBF]kwUicmM\DlimbnMn_MmWMuzmoeMpgaat}nQ`qcm`RnrQ~pk}o_nBcntINukN}B@sO~v[nv qNNNxm^wok|Z\ycksRTz{[sZozOq`ZWo`iOZucMo~x@oOtjk}WvE`Bx@^REhA^`fSBb`KXLu `JxCvRQhDNai@~QZWHhea}aPma^`GVF`HY]SbxFTbbqI^VOBspbmhK~bpXLJcsHMVcvxMbcyhNz bkQJvV{dOZWoTPfW~DQnT@IX\d\qRZ^UhE~dhPSRWSizEe}tURTYiVrdMUUZZFuWfZaYixeeyVN BbY~afiI[~TnimMpy\Rgui]^gxY^jg{I_vg~y_BhAj`NhDZaZhGJbfhJzbrhMjc~hPZdJiSZdV @@@pN %%%%%%%%%%%%%%%%%% End /document/graphics/Image10.gif %%%%%%%%%%%%%%%%% %%%%%%%%%%%%%%%%% Start /document/graphics/Image20.gif %%%%%%%%%%%%%%%% GedQxdSXaIPLA@H@@@@@@|sB@@@@@aIPLA@`@~sxcin\{OLJgtj}bszMo{`abcdefghijkl mnopqrstuvwxyz{|}~@CJ\HqbFObLireL[z|IthRgRmjukXsj]Kwn^BNlxqdK[NF@pin~Lvv ]asUNZuv]~nWzpxgu_^x]E`CjwLDZEbUih`KqxTOJUdgLITU~teMRTfMqIS^FBhJIZRdjYOf^T jFmzFmniagZYl[}ZFvJDnAiklGRkV|vkpCS\qF_Lrwf|rLwlsn~\tROmtAS}uXg}gZs]w^owwaK ~xNYMygc^zPkN{~rvoA_``Wcy|XNxkc~ypON~_C@r@AVOyUhwaPzQCBnPCfHGnHHjBFzNEFGI BILbB~khLxX]RxEAy\laG{cFcRI_A`RIIXYKRiKYQLWqLgh|jyGpyF[GOMI^tYOCjPc@PMI@JzP SZRQjKNjAXjRcJ{`jLO@UeJVgxQ]jHGtVWiUsjD~zUEkC~ZYJ{EPKQCkYW@[_{gxzYT{DvjEZc] CC^e{\i{OQY_W{`E[\q^yPbSPa_pbupcmJ`]|I}QcOKIk|~f\[{ZQnLDbLF^Lhi\A}|nx|]uLI U}XY]}\]g[HjmLiD}ZYuma]Gw}imHbe]Ep]^GRiK^mw\j`IoeykH~pS]hZNM_^fS^lGAtiPQEnr p~BqEsAOZxuvW}}LOuMawal__yyo^biZ{sE}e_koob~OYuzGmq_nw@zH@X@V^Exm\`Je}U MagBNZLXC^N\tDVWQxh`NTtEZF]xP}aHHMeayFYDbOeO}Q@wHvQkwhUQpxHuTBXH}RthLZcqhJV cZuoqci`CRInHwURcwJr^AtP~ZbX|pb~XKR^^`K]bA`~LeJYh}OLy|p^qDUZOAiMEenbWneGDRn OXy@eOjI\qfOYmHdoyQBBxtWJgiIIbYTXIPe|e^jJRezAhhGaZ\XenxgmiabfBjvqhsFdbXFfHh hcidBewtaiM`ejN^JPtJ`JNriwc\mJcZ\bLwYGTVljb}YCikZfODlFkWZgrJnhZYdXI^ktIwE hxj[jkpIpl~{XlA{Rjl~ZtMlwhj~bHqk~kMKib\PkNblSKur[KVu~l^{qvdV{uJmYkvbmB[iPkQ dy^n\[zjnhK{vnCXjFoYYiVl|{\V`WR~Vn~[^j`ALAsXH\A[pbkBKpbsCkokiBcbT\RjgQRDWkV l{^jEjCsq]lGGiHlF[kKk[MKdwyEnSlJGmr{^~rNkBprRdMSn\iKOsk[MgQ`W|uk}\BKIPTP[t LLOp}KJyPtWKStE]N_fL]Sg\yGTgrRmRksQ}VSsU\RCvUy{ruTmWuCMYWv`mYrf]DBVV|[sMt }}zss]HKrx{NWrzM\ga|a][u]fJiBNGOxXcbOnH@cWNE~vZht~GO^gDy{HfJGU~vPNa{DzyqLv ~y{MaGvg]}NzbTjCxTafGzh~kGzkndseWXuZ{FBnOu[|Zothkn[Q@Om_f]xkV|zqqo{e]rkPL_M z|FNNB}R{tKoU_zlyc[vkaW^Jzokqv_w]_NRp`_bO~sHHG~MNz{de_oCrm_UYpAYZl~tk|wgqmV _VwoDgUJ@ucHLI@[G_iSDnyptHVeCDHisRx@y\poEHJ[QDAY]bMuFlhJM@WT{BOx|j^IBiVvO fuXcwiAg_UpVGH\Rq@m\JOVxKTB^BY[iPwwMTFnBs`cP_gwKGrCc]~paXWC@^M|_FqdXMTIrDG_ HqfHTTGFEgbhogH~Wqkhc@KjEK\VqmhWLW|EWFYmbHQ|tIDIcio]XZkGbFMceQUxRLH~FmalQwX ZLNnFOcvq}HJDsUG{czQ@y\TJRIHKFrCIbLQjHWdLrFic|QBIcdRrIIelRZIodXrLif\SrI{d^r OIhLTJJOdydVheqTbJPefjXh\lMJH]etq@IN|OFKGdmR~Xm|NBDscvR^IldPzGee@Saym|UbKCc MbnHr\YvOXfNSHer|YJMaSsjy}IZvDGcGsbYn\[zKI_xqRwnT\RKGfjDUyE[BVNyffSAC]Qo@_ \JnNKxeAkqVgXBSG}D^jOsgnE~iXziLOE[TneGz|b]``Cfo~yOC`vT~[`^zlgLt^IdS]^P\hoK LjsAmQGehVt^yqAL~QshwSHH@E``W`h}QOjvtqUROTcTYhFma~RARjtVWAUUXHq[qTcY_{Vq|fi kQQuNmS]SMNEuZFgaerFt\JPEHPezITIgLdIC@r~eSSZHpNxDUdN{^gRqlJPrerQUYHN\JT}MpU I`MSXWM}oHxHK[DPSRmSHWYLwUBU}kDVJrkPByjBuoNMqfgRB{rZfkr[FvEFb}g~}ziLjE{GkI uReZUVkve]lJcrl~SN[EusvhxV`VNwhMKBY[g`syQi}nmZ[\mvMeu{`~[^mJKZ[idzI[EPvvMxi e]jYGX{VQZWu[RCCnglZ~_yVCgqMTQVUcxisvr}Tv\Cg\owZxXvu[f|lh[j\zb}d]Iw_Hw]UZ] _fRVF{y]xuXSoZirzdkqZZhXbuCG|MqJ_kZQvzk}M~y^K`QekjcebbrndsBLAfgELQpCW_JMt{ bOY_NXbyNIWN^Sg|TjKpzBONCm~rJ|VrCgs@qvkoiHNzu_InYXM|CfCk`KohVS[@~WBaOqLx@jA vE{[_GXivmD[Byq{wMlpMHobGrqXTl~S{nckhwX\lR^@Ga{\OY_\j}HWcSqVyoANK~baYn{Kg kQNKcEYoVXYFIbL{FPslre|UJfsoXFf[fFS_gl@lyL]FZvpnsWSXxHVH_~cedUdCt_yn|cjA KhepRw|`fqfhEpIyjKcVQS^gtPZG}cVFOaE_VZM]enScd[kaPJ]dFJWhAteYP}U~Skj{fpQ]lY FU@cr^ZL]\fTwRxt{yF}ZFbkgutzP}aFVseeujwSMo~XWlSvuIS}m~RcAcuL[jyhVJceaWZS LvFZkmkvccYUkJ|R`NnF{sYxfJjfcvH[GE^vIkW[vfet]Zy|AouTm{E\]n^snowr{n]rjMgoYuR Iqm]^`OCmqz{r}mU`[pAsUyEnaFagpgvM\[]EWaWjC`G|gkFwagkItS|lMGocGqGypvR^HW]wmP y_\VlHOcOlYyW\HtfVfWg@SD|~GHx|MNDWeOnmys\X~Kd[ryvD]]LGcGtEzF}cnKoiws{C~Cj lcYjgseLO\kMRgiWkSzR|aNy~kkuivV`~JUSqyh}|F[w\Wanwx[}\Bn[toGCu]r~lt[\wUC{ ]xcOgYpVQfA~xfPS[LxCxE]|np^[kxWua}mnkNoWiJS|\bBewcgWn|f]u~dGdwuGNuDQ|aiy} zyD]bG^tgxW|aFTUw]_WzNKJU_MWicW]aX^R^kbiD]i}e][o@}CVyOw}GjRpMdJjmyHv[nwiC zIvyXcJtIWug~Myu~Xqw{vcxK_F^igCnWv{ALm\GIw}WtXxs_w]jD{S`W_glzGw~O^L~c~ETt MdTz~KSszmX{BYP~YqqwuwAL|q~EfdV}~}G\@NEIXq_pUPH|@fyh@BwAAB{YAzwL|}PEW LUqfmlA^mm}A|M[Im}XEBr[ilE}I|ESU@b}}}AciBR}aTy]Pue|QBBS}wygmyeCrCFfAvQ|yCr BJDzeu|IiAy}bEyeDnDnBBdaDvDrdq_QDneAVEZE^EbEfEjEnErEvEzE~EBFFFJFNFRFVFZF^F zkAuMc]BZDjkuxiDBEFx|z}FJvAC^D~~`BvC~DNCn@BguFnFNBfvY}y_}GzlIyYGzFroXCr|ICv GrGRHNEVU~yxubUjlHfjaFfIjInIrUfXghgxgHhXhhhxhHiXihixiHjXjX|v\X`H[XdxzWcXeH kG_Vax`HWwlXbheHkhntdH\xcHFsnXVGoHPhmxYVpxKhpHnH`hnX^xOXkhZhah\WmhHxpHtXqh\ XsxPhjxvHwXwhwxwHxXxhxxxHyXyhyxyHzXzhzxzH{X{h{x{H|X|h|x|H}X}h}x}H~X~h~x~HX dSA@@@lC %%%%%%%%%%%%%%%%%% End /document/graphics/Image20.gif %%%%%%%%%%%%%%%%% %%%%%%%%%%%%%%%%% Start /document/graphics/Image30.gif %%%%%%%%%%%%%%%% GedQxdSXkDPs@@H@@@@@@|sB@@@@@kDPs@@`@~sxcin\{OLJgtj}bszMo{`abcdefghijkl mnopqrstuvwxyz{|}~@CJ\HqbFObLireL[z|IthRgRmjukXsj]Kwn^BNlxqdK[~LztjWsnm{w pcK_N}zvocOo~}kAp_O~Ax`xHHaGNcaHnhKJrxcixHdSRbdT^yGVbyfZhIg_Vag`NJDbRziKXJ jkZ`jlbjkoRZlrBJmurymxbin{RYo~BIpAsxpDchqGSXrJCHsMswsPkgtScWuV[GvYSwv\Kgw_C Wxb{FyesvyhkfzkcV{n[F|qSv|tKf}wCV~z{E}seAPmHpAbeBZPk`pDReErPixpGBeHJQgPqJ rdKbQ~WbFwHRzhGMBHCIQFIIEbIOIPRiJ}AK[IO^ILuaLgYalyxpy[jiNiAO{iL~IPaaPGZcLJ{ PZZJjRUA@^JTcjTgJUkjUoJVsjVaZSU@S{zI~JXKaXG{HJkYCAZS[fX[XVk[y@\c[ghkl[WfK^ m`^wKEzk_e@`C\iH|AM|UFLbY`bWlW]\Ca|TZld[UecBek\fe|fa\g]|gW\hQ|hM\iI|iC\j}{j w[kq{km[li{lc[m]{mY[nS{nM[oG{oA[p{zp[ZqUzqQZrMzrGZsAzs{Ymg^wvgugCuRt_lvoNx |^^gxVGOyKoyOOzSozWO{[Uo[wcwe_xqNxCXuiO~mo~I^K[OAgME`st@BSEXK]`jTB^RqvTA \ag`l`uW}Ia{GEz[pwky`ytEz_APDN_^XkhaVveeQehIfD`XJrQhHKJRGpS}bsxK`UtxMbcyhNn ciCWqc`bTa^dPnQDIFYdRDR~PJICqdIdSZP[xONeTYUZeWIVfeZyVre]iW~e`YXJfcIYVffyYb GE@@@lC %%%%%%%%%%%%%%%%%% End /document/graphics/Image30.gif %%%%%%%%%%%%%%%%% %%%%%%%%%%%%%%%%% Start /document/graphics/Image40.gif %%%%%%%%%%%%%%%% GedQxdSXBI@QA@H@@@@@@|sB@@@@@BI@QA@`@~sxcin\{OLJgtj}bszMo{`abcdefghijkl mnopqrstuvwxyz{|}~@CJ\HqbFObLireL[z|IthRgRmjukXsj]Kwn^BNlxqdK[~LztjWsnm{w pcK_N}zvocOo~}|{{O`AJx`DVhaGbXbJnHcMzxcPFidSRYeV^IfYjyf\vig_BZhbNJieZzihfj jkrZkn~JlqJ{ltVkmwb[nznKo}z{o@GlpCS\qF_LrIk|rLwlsOC]tROMuU[}uXgmv[s]w^MxaK ~xdWnygc^zjoN{m{~{pGo|sS_}v_O~yk~|woCPaA`@RPeLpBbpb\pDrP`lpFBq]|pHRQ[LqJ bqX\q~sHGYqF{HHOqGCIIEqHKIJ{pISIKqpJ[ILgpKcIM]pLkINSpMsIOwIOqiOIMCjPaIQKJK OJRQiRWJI[jSAITcJGgJUqhUoJEsjVaHW{JCJXQhXGKR@@ZSkZWK[[k[_[Z]D\gK]k[Yec]sk^ ckYgq]{k~K`yg`Gl~JlaqGbSl}VLcigc_l|bldaGekl{nLfYgfwlzzlgQGhCmyFMiIgiOmxRmj AGk[mw^MlyflgmvjmmqFnsmuvMoifomtBnpaFqKnsNNrYfrWnrZnsQFtcnqfNuIfuon@u^rNw ew^oBoxwEyK_nNOzoezW_mZo{gE|c_lfO}_e}o_kro~~_U|oirA`gb@N`eRAZ`cBBf`arBr `_bC~`]RDVUZdLdZlFF^aVHG~BYXGNDZHHvB_HLTb^xFZbiHJjPcHIzb^DLnBbhJVcvhKh^tHKn c|xKJcipIbckhPvbwHQvc~HRFcFYR^dJiJhT{eTNeTYUZeWIVfeZyVre]iW~e`YXJfcIYVfjUNh cNYSjfKISNcCyZFgoYQ~dPyOBdwIJdfri]fgfPNzgY`VB{Y[rfFJ_vguYOThsybjgCJIthJzaz hUjOLixi[ZiIjf~hbTdNBYjSQaRhfTjfJf`jijeljlJexjRvYrKfF[Q_OTl^Fcv@]PyZBykeaHB lEtEYkxGGN~sBe|UoiSAPoVEFKVTkFPrziN[uEmJpG}lSAoRm[qt^m^tZeEUk^LR\KUdnR{NbVo yzZEEKy^Lh{w^DmEC@oqkWhoj[svoAsIApNq@wm|k}^E[KBc]s{AlPnkJnZqpVpOQ}JqQuC{kT |BGqKPEj\@pjq{[C@ralFhYsE`dmRLDcrfDLONfkL[Ht\MSHwLNKH{zrns{gSjslE{WCMPCgF MAVtImjDdLmRoiOmBFuRmhlkUmacMLVOu\Mpyu_MXO\b}XSvFgY_vh]rivkM[s\n}[CwOg\Owt mtYwwM^W]z}^swXg_wFhYBWru`{VEnawr@nPCxcAcwxaqcCy_XGR~dwEUneCnYno`y[N{zyfGh Gzb^zQzenis^h^jkzoGkwznn|A{qnlW_t^m[{x^nk{{Now{~~oC|AopO|D_q[|GOrg|Jrs|Mos |P_tK}SOuW}Vuc}Yovo}\_w{}_OxG~bxS~eoy_~h_zk~kO{w~n{Cqo|Ot_}[wO~gz~ s}o@p@h@\@R@K`FpCHBLAj@W`LpFhC|ABAc`RpIHElBZAo`XpLhF\CrA{`^pOHHLDJBGadp RhI|DbBSajpUHKlEzB_appXhL\FRCkavp[XBl@@@pN %%%%%%%%%%%%%%%%%% End /document/graphics/Image40.gif %%%%%%%%%%%%%%%%% %%%%%%%%%%%%%%%%% Start /document/graphics/Image50.gif %%%%%%%%%%%%%%%% GedQxdSXBDPs@@H@@@@@@|sB@@@@@BDPs@@`@~sxcin\{OLJgtj}bszMo{`abcdefghijkl mnopqrstuvwxyz{|}~@CJ\HqbFObLireL[z|IthRgRmjukXsj]Kwn^BNlxqdK[~LztjWsnm{w pcK_~K@toSrN~}_tsoF~Ax`UHHaGBaaHnxBJrxcFxHdLJydHVieDbYf@nIg|yygxEZWdJJCfnU jgfpjYyJkQjf]@@khvZUmrFkDxVe_~ZYpEU[qM@Lo}bUqdb\rI~knE^ppPVMNLgQvOC]SFsMKWG sw`wbxfK]yuYTzjCsygPn{cGT{sKC|bX}xws~|wO]|O@Fp[LPBVpZdODvaE^aCrpD~NGFQNPQe UQ^\q~{VFwqF]pG{hv^gH}wzJiFQGJMyJByJGiJ\yKqPHaxLiqGeyMiY|vyOnhN}yCKQPEjE\XQ MzttgRIS[jO`WRaJENwTizDDwUqJFmHW]jWZTCkXYILKkYGJZGKOWK[Uk[SI\ckGgK]ExQo{[ sj^S[DvZ__Pj@\`AI_Kl\KbSl`WLcOfc_Ljb\][LeqkeoLvrlfMha{\NpDaA]k`Liw|dOmg[dj QmBF]kwUicmM\DlimbnMn_MmWMuzmoeMpgaat}nQ`qcm`RnrQ~pk}o_nBcntINukN}B@sO~v[nv qNNNxm^wok|Z\ycksRTz{[sZozOq`ZWo`iOZucMo~x@oOtjk}WvE`Bx@^REhA^`fSBb`KXLu `JxCvRQhDNai@~QZWHhea}aPma^`GVF`HY]SbxFTbbqI^VOBspbmhK~bpXLJcsHMVcvxMbcyhNz bkQJvV{dOZWoTPfW~DQnT@IX\d\qRZ^UhE~dhPSRWSizEe}tURTYiVrdMUUZZFuWfZaYixeeyVN BbY~afiI[~TnimMpy\Rgui]^gxY^jg{I_vg~y_BhAj`NhDZaZhGJbfhJzbrhMjc~hPZdJiSZdV @@@pN %%%%%%%%%%%%%%%%%% End /document/graphics/Image50.gif %%%%%%%%%%%%%%%%% %%%%%%%%%%%%%%%%% Start /document/graphics/Image60.gif %%%%%%%%%%%%%%%% GedQxdSXaIPLA@H@@@@@@|sB@@@@@aIPLA@`@~sxcin\{OLJgtj}bszMo{`abcdefghijkl mnopqrstuvwxyz{|}~@CJ\HqbFObLireL[z|IthRgRmjukXsj]Kwn^BNlxqdK[NF@pin~Lvv ]asUNZuv]~nWzpxgu_^x]E`CjwLDZEbUih`KqxTOJUdgLITU~teMRTfMqIS^FBhJIZRdjYOf^T jFmzFmniagZYl[}ZFvJDnAiklGRkV|vkpCS\qF_Lrwf|rLwlsn~\tROmtAS}uXg}gZs]w^owwaK ~xNYMygc^zPkN{~rvoA_``Wcy|XNxkc~ypON~_C@r@AVOyUhwaPzQCBnPCfHGnHHjBFzNEFGI BILbB~khLxX]RxEAy\laG{cFcRI_QJ]qJ[QJ[IK`yJdyDOylDY{ryLuIO{iO}@MiPGjlBJQOJR IcQSZs@`S]JSupSgJTku\hzSczmfZ~rjV_@WGhW_x`FkBB{W[@ZwUUSkAL[D`{MaT[e`\uz]wT^ }yZoP_wfevk]ELO{}f@aGrRiPbOPdZ|nJW]AKbg\id|cIiE`Lj\deapex|aGugi\ED]~|\huLF P]@Qbk]|VWlZa]ic{lamm}tKypm{\Q~m]}MmOKkHErK^BTNBDNnKXGbmIsesG^tkNkY[jvNvanv AZvAZqGkJxY[hBuyARwm^CEQ{UyObo{AA}]y|qe~WL~WaW}@WWUB|r}USwCAN`FeBb`Asm PNxBT`Tb@RZEcDvHWXCbIpDGBaNPF~F^Xn\JbHh}OghdPb^CHbReX\u[ucKBZrxHpbCcMzYZXEa Ftx`UFxhENF}ximcrwHIFCinYSeSGudVTHqNIIQnd_@CIe[FNeeqttqREYWZd_yGIfxGXre^YXn ehiYnfFHYzdoYZzfcI\rcrYJ@dJsCZg[TIug~s_vgG]Na{d`NgDZ[~^^w]FgJzY^hCXaZhPJIh OmteJi[DJzhqHcrg}SXObJGFj^JePjKzkahQz~dZkZezj`zkVcqzcripwlNiwZzYBAeofkvwhU TWFrDLF[rx~K^eZy|qLLHmnLbhrReCzqhKVks@aR[Xp\ZkZB^xzspmnusqlUCxFlfK{MnuWmFMe GoNM|wzbXAHutJw[vh`sKcunv[U~oZCMum@[}JU_kbE\G|z`ppRtipQVjBqKlAgpU|DggM\FSqW lD[k~@ry[jbHsqXIkLr[LFkqT[G{WdLKspi\K_rV\kyrj[DCsr\Mkds\Ios}lJOelIcsylP{s@ ]PSoCMQssH}Qwt^|MksOMMGuPMRlNMVot[MeZt]MWktcmIRuIMY[ZUmUwTYZTZvlkzXi{KkRph sjIvichNwt]orWj]ZkSz}^Ck}}OUIGnPYMC\xrw~]XWvI^cOxO~op{PeYpmcWySNakyF~caHe myu}[GydK`STo[~fbPNhcRlNjOPpNf\zuvkWtcRmK{w~\yzqN`u{v~o{HzZnO|MWFM{{noG|Lyq K[JW[|Lrw|]RkW}t^t[\OoVKu^_Dv}`ovK|YOys{frQEpXrg~P_{o}o_xo~QTys}pJ_}voz S}o{VuO|G{A|~^So{GBL~y|f_LPAhuC@R@K`\cLqb{`t@G`kKJxhs@b|v[~aOx`CqmAs_Qp hBH|BfAq[cpDexCo`BC@UP`qg[Engv_sK~@N\s]yhaROyWHLABCw`LbXh]SwMwhCHQahQdGVC{` JQgHD|IZC{FDq`H~SQkx|pBFEwbdelHVdKF@y`RPyPUlLbAb]QmHZtLb{P_kQYX|xJrF}bdQxx nr}EkbhqtH\POzGabqvHzkOZUlcyqBY\\KFHoc~ov`^\OvE]dGI{hdDQJFu_XqGif|}mIQdUR CIIDv}ISdIrQiWtBZJUccczHSDBnF^dbDXi[{JRC_e`qUHklUjj|auRWin|M~KwerrwQZXXjPLf qM^IOlXBI{ewr`iXBYNRXfAsTItlIIMcCxidBu|ZjYrfWEpiUQ\vCW_[sWVCZXjNOgSmKyFR]rF XgeKmI|TPFFgdPi|I~\_VNqc|SPie|_RJCews@jAE`bPGh}SGhBMa^J~wajJYSPtth@}avPoh^k IZAUczOOhXToyy`\BRGisRRjI}dnPQijtUZHcezR_ihKXjL]f~whivt[jrDgjSiibt]J@{IyQmh BuHzGmQUQIjCUJZPEivDSj[tbzSuivQehPuiZUeaJUqjMUfjRecjU{jWukCVYlRVKkfusJZMmjV Wkluvj[}mBWckruyJ]mnZWokxu|j^]orW{k~uszOU_nTAkYUEHWmmMXcjGVkZauMfR]l^UCKa]q ~UQlQVH[XEsvX_`YtLKgerJYUlSV_JrbtZZgi}tSkj][mZ[mhtTkkMvnYemgAcim]ZIASlLvmzh Utz[wm}V`keusR|^fsvE~cBVzR[qZvkrJyRekeY`q\OnFObHZYzV\}eFWJGL|yNHAbdo[{QJFRH sdqQl{izLVPenyTjKqBVZKwnZOt[sU}vQYorw_{MA~FEW\cEzK|Ajx]WoAPq[adZ^CD@XNW{uz UbFk`wd[p]{Y]cpmFlBukYj]LLMS|KRRBCVkpvWVhFFyNbqdlvcJVm[U^ieENphMCNeNYwK[n uEcYpaTE~tgACOccn{xQH|MH[NK~XZKjEIkd[epvelSnFdGVKKPkfeJYgpZjiLVnJoeorzwl LPuF_~Tq_Yn|pu}BeumUVaK[vTAbmrcYu\hmMCf_s]yxlXvDKgI]eYoL]vL~_vO{fIsAZ|LqUO ggcraYEMc~ktd[tPzOVd^R{BitVzcrc~Rki_l[zN}DtSCjKiaZm\_vPcfO`Dzu|`^OKhgsjZWMl fOWhQtGZT\jNVbmseZZ}i^|ehOWzzYmmfU[jasLx[M`~UglquE[cJqfW[lGvztLUmW{ufvhzg mt^Tcmyv^[bQf~[Kn[ic{r]|r\cn]Wi{u]sb]{nkVo{x}MR^slavwY{}n^[[m]XdZa}MnVowwd {`mrnUolOxF|\MrNawlswA|hMp^awp}wT[@^EosJpW[]{kmEO[WmaxW\PnGoc{kKSR\|ME_dgqQ [t{sVKegbayr\atLofGsmyx|~wxy|]^{dg{sQx|`^OWhWptvO|{m~VdWrqxJ]N~JGlisoZ\zI ^AgjWoSzH}HnQOdguyzN\oNINjkqIyN}fnKgw~uUyf]inXge_fW{c}g~Hj_WuOv[]u^WGogvWzD }MU_osvA|B^I}`opOtM|H~]Yb_qW|X|N~w~cOrKke|RnvNTgl_vs{|]gme_nohe{hSL@dcuw {e]MiGr[zs|k~@Ohp[ro{[~V_\Gn{vC{b~ZOjgugrg}x~W|d`_uOzt^TFigxw{S}I_^Oewnw |_~\WhotosM}k~vLkOvOke}w~eNnow{C~C}kN_hgIc[A{kZrOlw|SwG~T\mWHhyWo{m}W ^vgw~std}J_y_{Gh{|A]qDY`I|Myy~Qyem{Mzqq|IEY@rpI~MARAVAZA^AbAfAjAnArA vAzA~ABBF~U@^|moLBz@v@j~E|AMYBfBj}EAZuMzpBJ@nuBVt}KICB|yBRBf@BAnCz{e~MCV~Q @N}CVgyCvuC^Bj{izm|yzMD~IAFBJENEREVEZE^EbEfEjEnErEvEzE~EBFFFJFNFRFVFZF^F bFfFjFnFrFvFzF~FBGFGJGNGRGVGZG^GbGfGjGnGrGvGzG~GBHFHJHNHRHVHZH^HbHfHjHBMU@@ @@{@ %%%%%%%%%%%%%%%%%% End /document/graphics/Image60.gif %%%%%%%%%%%%%%%%% %%%%%%%%%%%%%%%%% Start /document/graphics/Image70.gif %%%%%%%%%%%%%%%% GedQxdSXqH`j@@H@@@@@@|sB@@@@@qH`j@@`@~sxcin\{OLJgtj}bszMo{`abcdefghijkl mnopqrstuvwxyz{|}~@CJ\HqbFObLireL[z|IthRgRmjukXsj]Kwn^BNlxqdK[~LztjWsnm{w pcK_N}zvocOo~}|{{O`AJx`DVhaGbXbJnHcMzxcPFidSRYeV^IL@`yf\Vgf]BZhk}iheZZXd^j jk^Ujl~JlMyZltV{PsZ[nzfCn{z{ootKpCS\IBWLrIoqqJwlsQp|sRO}AQS}uM[Mv[Slv\}n^C ~xtJNygojyhonhjs~{[zN|sSi|t_cvc~JjOWHApABHARpCnGCbpEZGErpGFGGBqIrFIRqh @`Ljq~wHG{hGHHCiHGIIKiIOIJSiJWIK[iK_ILciLgIMkiMMyEYbEsIOEsN{IP{rOCJQqrPKJR grQSJS]rR[JTSrScJUIrTkJVqUsJWuqV{JXkqWCKYsTYOkRQkZ_rXWk[GQ[_k\}P\gk]sP]ok^ iP^wk__P_k`UP`GlaCPaOl`Slb}Kc[l^_Ldmkdgl\kle]KfslZwLgOkgLYCmhAKiKMWOMjqjj WMU[mkaJlcMSgMmQjmoMQsmnAJo{MOMpohpG~DKnq_HrS~BWNsOhs_~@cntGuk~~nNvogvw~| znw_Gxc\LB\_M_yeqyU_z[Oj\_p`oKVo|YAzM[}EZ~WA}~Gq_ip}}_~GXt_Cx|q^DHBj`m`An `oGCZ_@X@Nah`Cv`UxIt]wDGva^xGBbahHNbdXIZbghG^CWhDFa|WFZBkhKvbjpxUAqHM~bFSNR BwHENc~HD^`OxPRdlxOjaIhPba~tNBWMiHtcVxSFBQIRrdGiUjdFYQneKIVjcSYxaWbikUcdyxX fgywdfjyvpfmyu|fpytHgmyFeMsYUdJuIBpgF`_Jf{IWdQCZ^REBzVPhIJu\g\ScjEZB`BhIpdb hasGm@VjQaXEzWhhKZuxf}yR@RDv_vi^qgBicJvhiVqgHi^JgJTZdcrjmBGAjzWe^kqYmj_zJhr kaJsEk@{p^BjZhrBm~hYakjlHqdvkPzoBmdZPZmqbtfmaCw^X[kwZK`[xVNd{bYnuxbnlbznnj R{znfB|FomC}RYv{\MoxKgho{Kfto~ke@pA\}Pp]fAcVCLBCIJ|B{HMlCsHP\DwOTlE_qX\Fkq[ LGwq^|GCralHOrd\I[rgLJgrj|Jsrm|HW@@@{@ %%%%%%%%%%%%%%%%%% End /document/graphics/Image70.gif %%%%%%%%%%%%%%%%% %%%%%%%%%%%%%%%%%%%%% Start /document/N4QRHX00.wmf %%%%%%%%%%%%%%%%%%%% WwlqZB@@@@@@@DzBr^@zC@@@@@PYTH@@I@@@CXjF@@`A@pA@@@@@@P@@@@p@A`@@E@@@@l`@@@@ @@T@@@@@CBTqBBAE@@@@[@@@X@@vK@@@PB@A@`@@@@`@@x}_B@@@@~_@@xI@@D@@@@t R@@@@A@@@@pG@@@`@@@@`~B@@@@@@@@@@@@@@A@@@@mD@@@\@@@@@B@@@@@@@@@@@D@@@@tR@A @@G@@@@{K@DC@@@@@@@@Y@@@@@A\@@@@PPreVXlA@@@PAEJ`@CqOA@XE{|wEVls_GH@U]iEaY UR@@@@PKAH@@E@@@@HP@A@@@@T@@@@P@B|CPA@@@@nD@F@@@@E@@@@d`@@@@@@P@@@@@BA@@ @E@@@@D`@@l@@@@`LJ@@@@@@@@H@@@@@@@|KDUlPA@@@@AHpO@K@@@@HcB@@@@@@@@B@ @@@@@@BQEK\@@@@`EDTqBBA@@@@@E@@@@D`@@l@@@@`LJ@@@@@@@@H@@x@@N@\HD]k@G@ @@@{K@XC@@@@@@@@Y@@@@@A\@@@@@UiuVYsAbSe]GHR}V[ayF@XE{|wEVls_GH@U]iEaYUR@@ @@PKAL@@\@@@@lo@P|O@@@@@@@@dA@@@@DpA@@@@TeV[eMGHNUv]`Hu[mEf[@`Uls_WXqN]`@T }weVDfUIA@@@@mD@A@P@@@@PKAL@@H@@@@ho@@@`B@@@@@@@@@P@@@@PKAT@@D@@@@@_@@@@B@@ @@zK@@@h@@@@@@@@@@D@@@@tR@@@PA@@@@TH`xHxG@E@@@@La@bc`PPP@@@@PKAT@@D@@@@@_@@ @@B@@@@zK@@@h@@@@@@@@@@D@@@@tR@@@PA@@@@THpxH@FBE@@@@La@\C@XHP@@@@PKAT@@D@@@ @@_@@@PA@@@@TH`H`M@E@@@@La@Fc@v@T@@@@@EBhQBnFPA@@@@SH`jHxZ@G@@@@DRAA@PK@t\ BlEpA@@@@aTP@@`C@MgPhAT@@@@@EBxOBXCPA@@@@SH`qH`M@E@@@@Pa@Zd`VCT@@@@pDBhJBZM pA@@@@aTP@@tB@Mg@FC\@@@@PHED@@v@PsItt@E@@@@Pa@~c@aBT@@@@pDBXLBDJPA@@@@TH`FI \PAE@@@@La@jbpAE\@@@@PHED@@m@PsITLAG@@@@DRAA@@M@t\BzSPA@@@@TH`HDCAE@@@@La@ FcPLDT@@@@@EBhQBsZPA@@@@SH`jHLkAG@@@@DRAA@PK@t\BqYpA@@@@aTP@@HC@Mg`iFT@@@@@ EBxOB]WPA@@@@SH`qHt]AE@@@@Pa@Zd@XHT@@@@pDBhJB`apA@@@@aTP@@@C@Mg@NHT@@@@@EBx OBI^PA@@@@SH`qHdxAE@@@@Pa@Zd@CJT@@@@pDBhJBLhpA@@@@aTP@@HC@Mg@yIT@@@@@EBxOBv dPA@@@@SH`qHXSBE@@@@Pa@Zd@nKT@@@@pDBhJBxnpA@@@@aTP@@PC@Mg@dKT@@@@@EBxOBbkPA @@@@SH`qHHnBE@@@@Pa@ZdPYMT@@@@pDBhJBeupA@@@@aTP@@XC@MgPOMT@@@@@EBxOBNrPA@@@ @SH`qHxHCE@@@@Pa@ZdPDOT@@@@pDBhJBQ|pA@@@@aTP@@`C@MgPzNT@@@@@EBxOB{xPA@@@@SH `qHlcCE@@@@Pa@~cpyOT@@@@pDBXLBgPA@@@@TH`xH`IBE@@@@La@bc@JHT@@@@@EB\tAXbPA@ @@@SHpQG`BBG@@@@DRAA@@L@xxAh\pA@@@@aTP@@xB@N^@^G\@@@@PHED@@r@`cG@zAE@@@@Pa@ U`@_HT@@@@pDBTABDaPA@@@@TH@kE`IBE@@@@La@lV@JH\@@@@PHED@@p@p|E`rAG@@@@DRAA@` K@L_Ax]pA@@@@aTP@@PC@sW@hGT@@@@@EBhgA|aPA@@@@SH`^FPDBE@@@@Pa@RP@fHT@@@@pDBH AAh`pA@@@@aTP@@@C@YQ@JG\@@@@PHED@@n@PVD`wAG@@@@DRAA@`M@dEA`^PA@@@@THpwDpGBE @@@@La@_S@QHT@@@@@EB\g@XbPA@@@@SHp]B`BBG@@@@DRAA@@L@xk@h\pA@@@@aTP@@xB@~J@^ G\@@@@PHED@@x@`oB@zAE@@@@Pa@DM@_HT@@@@pDBPt@DaPA@@@@TH@w@`IBE@@@@La@\C@JH\@ @@@PHED@@q@pHA`rAG@@@@DRAA@`K@LR@x]pA@@@@aTP@@@C@cD@hGT@@@@@EBdZ@|aPA@@@@SH PjAPDBD@@@@tR@D@pA@@@@aTP@@`G@oi`rO\@@@@PHED@@yAplA`gAD@@@@tR@C@@B@@@@zK@@@ @@@@@@@@@@@D@@@@tR@@@@A@@@@pGPA@PA@@@pF@@@F@`}B@@@d@P@@H@@@@H@@~_g@@@@` G@@~_B@@A@@@@mDPA@P@@@@@|AT@@G@@@@XAAUlpoP@@@@@@B@@@@zK@@@pA@@@@@@@@@D@@ @@tR@E@@A@@@@pG@@@T@@@@@EBpM@`aPA@@@@SH@w@\FBE@@@@Pa@\CpYHT@@@@pDBpM@naPA@@ @@TH@w@xFBE@@@@La@\CP]HT@@@@@EBpM@uaPA@@@@SH@w@pGBE@@@@Pa@\C@_HT@@@@pDBpM@C bPA@@@@TH@w@LHBE@@@@La@\C`bHT@@@@@EBpM@JbPA@@@@SH@w@DIBE@@@@Pa@\CPdHT@@@@pD BpM@XbPA@@@@TH@w@`IBE@@@@La@\CpgHT@@@@@EBpM@_bPA@@@@SH@w@XJBE@@@@Pa@\C`iHT@ @@@pDBpM@mbPA@@@@TH@w@tJBE@@@@La@\C@mHT@@@@@EBpM@tbPA@@@@SH@w@lKBE@@@@Pa@\C pnHT@@@@pDBpM@BcPA@@@@TH@w@HLBE@@@@La@\CPrHT@@@@@EBpM@IcPA@@@@SH@w@@MBE@@@@ Pa@\C@tHT@@@@pDBpM@WcPA@@@@TH@w@\MBE@@@@La@\C`wHT@@@@@EBpM@^cPA@@@@SH@w@TNB E@@@@Pa@\CPyHT@@@@pDBpM@lcPA@@@@TH@w@pNBE@@@@La@\Cp|HT@@@@@EBpM@scPA@@@@SH@ w@hOBE@@@@Pa@\C`~HT@@@@pDBpM@AdPA@@@@TH@w@DPBE@@@@La@\C@BIT@@@@@EBpM@HdPA@@ @@SH@w@|PBE@@@@Pa@\CpCIT@@@@pDBpM@VdPA@@@@TH@w@XQBE@@@@La@\CPGIT@@@@@EBpM@] dPA@@@@SH@w@PRBE@@@@Pa@\C@IIT@@@@pDBpM@kdPA@@@@TH@w@lRBE@@@@La@\C`LIT@@@@@E BpM@rdPA@@@@SH@w@dSBE@@@@Pa@\CPNIT@@@@pDBpM@@ePA@@@@TH@w@@TBE@@@@La@\CpQIT@ @@@@EBpM@GePA@@@@SH@w@xTBE@@@@Pa@\C`SIT@@@@pDBpM@UePA@@@@TH@w@TUBE@@@@La@\C @WIT@@@@@EBpM@\ePA@@@@SH@w@LVBE@@@@Pa@\CpXIT@@@@pDBpM@jePA@@@@TH@w@hVBE@@@@ La@\CP\IT@@@@@EBpM@qePA@@@@SH@w@`WBE@@@@Pa@\C@^IT@@@@pDBpM@ePA@@@@TH@w@|WB E@@@@La@\C`aIT@@@@@EBpM@FfPA@@@@SH@w@tXBE@@@@Pa@\CPcIT@@@@pDBpM@TfPA@@@@TH@ w@PYBE@@@@La@\CpfIT@@@@@EBpM@[fPA@@@@SH@w@HZBE@@@@Pa@\C`hIT@@@@pDBpM@ifPA@@ @@TH@w@dZBE@@@@La@\C@lI`@@@@`~B@@@@@@@@@@@@@@A@@@@mD@@@P@@@@@|AT@@T@@@@|oA@ @`A@Xo@@@@I@D@@B@@@@B@`wI@@@@xA@`g@@P@@@@PKAT@@D@@@@@_@E@pA@@@@VPPE K|KD@@@@@PA@@@pF@@@F@`}B@@@d@P@@H@@@@H@@~_g@@@@`G@@~_B@@A@@@@mDPA@P@ @@@@|AT@@G@@@@XAAUlpoP@@@@@@B@@@@zK@@@pA@@@@@@@@@D@@@@tR@E@@A@@@@pG@@@T@@@@ @EBHNBpfPA@@@@SH`xH\[BE@@@@Pa@bcpmIT@@@@pDBHNB~fPA@@@@TH`xHx[BE@@@@La@bcPqI T@@@@@EBHNBEgPA@@@@SH`xHp\BE@@@@Pa@bc@sIT@@@@pDBHNBSgPA@@@@TH`xHL]BE@@@@La@ bc`vIT@@@@@EBHNBZgPA@@@@SH`xHD^BE@@@@Pa@bcPxIT@@@@pDBHNBhgPA@@@@TH`xH`^BE@@ @@La@bcp{IT@@@@@EBHNBogPA@@@@SH`xHX_BE@@@@Pa@bc`}IT@@@@pDBHNB}gPA@@@@TH`xHt _BE@@@@La@bc@AJT@@@@@EBHNBDhPA@@@@SH`xHl`BE@@@@Pa@bcpBJT@@@@pDBHNBRhPA@@@@T H`xHHaBE@@@@La@bcPFJT@@@@@EBHNBYhPA@@@@SH`xH@bBE@@@@Pa@bc@HJT@@@@pDBHNBghPA @@@@TH`xH\bBE@@@@La@bc`KJT@@@@@EBHNBnhPA@@@@SH`xHTcBE@@@@Pa@bcPMJT@@@@pDBHN B|hPA@@@@TH`xHpcBE@@@@La@bcpPJT@@@@@EBHNBCiPA@@@@SH`xHhdBE@@@@Pa@bc`RJT@@@@ pDBHNBQiPA@@@@TH`xHDeBE@@@@La@bc@VJT@@@@@EBHNBXiPA@@@@SH`xH|eBE@@@@Pa@bcpWJ T@@@@pDBHNBfiPA@@@@TH`xHXfBE@@@@La@bcP[JT@@@@@EBHNBmiPA@@@@SH`xHPgBE@@@@Pa@ bc@]JT@@@@pDBHNB{iPA@@@@TH`xHlgBE@@@@La@bc``JT@@@@@EBHNBBjPA@@@@SH`xHdhBE@@ @@Pa@bcPbJT@@@@pDBHNBPjPA@@@@TH`xH@iBE@@@@La@bcpeJT@@@@@EBHNBWjPA@@@@SH`xHx iBE@@@@Pa@bc`gJT@@@@pDBHNBejPA@@@@TH`xHTjBE@@@@La@bc@kJT@@@@@EBHNBljPA@@@@S H`xHLkBE@@@@Pa@bcplJT@@@@pDBHNBzjPA@@@@TH`xHhkBE@@@@La@bcPpJT@@@@@EBHNBAkPA @@@@SH`xH`lBE@@@@Pa@bc@rJT@@@@pDBHNBOkPA@@@@TH`xH|lBE@@@@La@bc`uJT@@@@@EBHN BVkPA@@@@SH`xHtmBE@@@@Pa@bcPwJT@@@@pDBHNBdkPA@@@@TH`xHPnBE@@@@La@bcpzJT@@@@ @EBHNBkkPA@@@@SH`xHHoBE@@@@Pa@bc`|JT@@@@pDBHNBykPA@@@@TH`xHdoBE@@@@La@bc@@K `@@@@`~B@@@@@@@@@@@@@@A@@@@mD@@@P@@@@@|AT@@T@@@@|oA@@`A@Xo@@@@I@D@@B@@@@B@` wI@@@@xA@`g@@P@@@@PKAT@@D@@@@@_@E@pA@@@@VPPEK|KD@@@@@PA@@@pF@@@F@` }B@@@d@P@@H@@@@H@@~_g@@@@`G@@~_B@@A@@@@mDPA@P@@@@@|AT@@G@@@@XAAUlpoP@ @@@@@B@@@@zK@@@pA@@@@@@@@@D@@@@tR@E@@A@@@@pG@@@T@@@@@EBpM@O\PA@@@@SH@w@XqAE @@@@Pa@\C`EGT@@@@pDBpM@]\PA@@@@TH@w@tqAE@@@@La@\C@IGT@@@@@EBpM@d\PA@@@@SH@w @lrAE@@@@Pa@\CpJGT@@@@pDBpM@r\PA@@@@TH@w@HsAE@@@@La@\CPNGT@@@@@EBpM@y\PA@@@ @SH@w@@tAE@@@@Pa@\C@PGT@@@@pDBpM@G]PA@@@@TH@w@\tAE@@@@La@\C`SGT@@@@@EBpM@N] PA@@@@SH@w@TuAE@@@@Pa@\CPUGT@@@@pDBpM@\]PA@@@@TH@w@puAE@@@@La@\CpXGT@@@@@EB pM@c]PA@@@@SH@w@hvAE@@@@Pa@\C`ZGT@@@@pDBpM@q]PA@@@@TH@w@DwAE@@@@La@\C@^GT@@ @@@EBpM@x]PA@@@@SH@w@|wAE@@@@Pa@\Cp_GT@@@@pDBpM@F^PA@@@@TH@w@XxAE@@@@La@\CP cGT@@@@@EBpM@M^PA@@@@SH@w@PyAE@@@@Pa@\C@eGT@@@@pDBpM@[^PA@@@@TH@w@lyAE@@@@L a@\C`hGT@@@@@EBpM@b^PA@@@@SH@w@dzAE@@@@Pa@\CPjGT@@@@pDBpM@p^PA@@@@TH@w@@{AE @@@@La@\CpmGT@@@@@EBpM@w^PA@@@@SH@w@x{AE@@@@Pa@\C`oGT@@@@pDBpM@E_PA@@@@TH@w @T|AE@@@@La@\C@sGT@@@@@EBpM@L_PA@@@@SH@w@L}AE@@@@Pa@\CptGT@@@@pDBpM@Z_PA@@@ @TH@w@h}AE@@@@La@\CPxGT@@@@@EBpM@a_PA@@@@SH@w@`~AE@@@@Pa@\C@zGT@@@@pDBpM@o_ PA@@@@TH@w@|~AE@@@@La@\C`}GT@@@@@EBpM@v_PA@@@@SH@w@tAE@@@@Pa@\CPGT@@@@pDB pM@D`PA@@@@TH@w@P@BE@@@@La@\CpBHT@@@@@EBpM@K`PA@@@@SH@w@HABE@@@@Pa@\C`DHT@@ @@pDBpM@Y`PA@@@@TH@w@dABE@@@@La@\C@HHT@@@@@EBpM@``PA@@@@SH@w@\BBE@@@@Pa@\Cp IHT@@@@pDBpM@n`PA@@@@TH@w@xBBE@@@@La@\CPMHT@@@@@EBpM@u`PA@@@@SH@w@pCBE@@@@P a@\C@OHT@@@@pDBpM@CaPA@@@@TH@w@LDBE@@@@La@\C`RHT@@@@@EBpM@JaPA@@@@SH@w@DEBE @@@@Pa@\CPTHT@@@@pDBpM@XaPA@@@@TH@w@`EBE@@@@La@\C@XH`@@@@`~B@@@@@@@@@@@@@@A @@@@mD@@@P@@@@@|AT@@T@@@@|oA@@`A@Xo@@@@I@D@@B@@@@B@`wI@@@@xA@`g@@P@ @@@PKAT@@D@@@@@_@E@pA@@@@VPPEK|KD@@@@@PA@@@pF@@@F@`}B@@@d@P@@H@@@@H@@~_ g@@@@`G@@~_B@@A@@@@mDPA@P@@@@@|AT@@G@@@@XAAUlpoP@@@@@@B@@@@zK@@@pA@@@@@@ @@@D@@@@tR@E@@A@@@@pG@@@T@@@@@EBHNBVPA@@@@SH`xHX\AE@@@@Pa@bc`qET@@@@pDBHNB MWPA@@@@TH`xHt\AE@@@@La@bc@uET@@@@@EBHNBTWPA@@@@SH`xHl]AE@@@@Pa@bcpvET@@@@p DBHNBbWPA@@@@TH`xHH^AE@@@@La@bcPzET@@@@@EBHNBiWPA@@@@SH`xH@_AE@@@@Pa@bc@|ET @@@@pDBHNBwWPA@@@@TH`xH\_AE@@@@La@bc`ET@@@@@EBHNB~WPA@@@@SH`xHT`AE@@@@Pa@b cPAFT@@@@pDBHNBLXPA@@@@TH`xHp`AE@@@@La@bcpDFT@@@@@EBHNBSXPA@@@@SH`xHhaAE@@@ @Pa@bc`FFT@@@@pDBHNBaXPA@@@@TH`xHDbAE@@@@La@bc@JFT@@@@@EBHNBhXPA@@@@SH`xH|b AE@@@@Pa@bcpKFT@@@@pDBHNBvXPA@@@@TH`xHXcAE@@@@La@bcPOFT@@@@@EBHNB}XPA@@@@SH `xHPdAE@@@@Pa@bc@QFT@@@@pDBHNBKYPA@@@@TH`xHldAE@@@@La@bc`TFT@@@@@EBHNBRYPA@ @@@SH`xHdeAE@@@@Pa@bcPVFT@@@@pDBHNB`YPA@@@@TH`xH@fAE@@@@La@bcpYFT@@@@@EBHNB gYPA@@@@SH`xHxfAE@@@@Pa@bc`[FT@@@@pDBHNBuYPA@@@@TH`xHTgAE@@@@La@bc@_FT@@@@@ EBHNB|YPA@@@@SH`xHLhAE@@@@Pa@bcp`FT@@@@pDBHNBJZPA@@@@TH`xHhhAE@@@@La@bcPdFT @@@@@EBHNBQZPA@@@@SH`xH`iAE@@@@Pa@bc@fFT@@@@pDBHNB_ZPA@@@@TH`xH|iAE@@@@La@b c`iFT@@@@@EBHNBfZPA@@@@SH`xHtjAE@@@@Pa@bcPkFT@@@@pDBHNBtZPA@@@@TH`xHPkAE@@@ @La@bcpnFT@@@@@EBHNB{ZPA@@@@SH`xHHlAE@@@@Pa@bc`pFT@@@@pDBHNBI[PA@@@@TH`xHdl AE@@@@La@bc@tFT@@@@@EBHNBP[PA@@@@SH`xH\mAE@@@@Pa@bcpuFT@@@@pDBHNB^[PA@@@@TH `xHxmAE@@@@La@bcPyFT@@@@@EBHNBe[PA@@@@SH`xHpnAE@@@@Pa@bc@{FT@@@@pDBHNBs[PA@ @@@TH`xHLoAE@@@@La@bc`~FT@@@@@EBHNBz[PA@@@@SH`xHDpAE@@@@Pa@bcP@GT@@@@pDBHNB H\PA@@@@TH`xH`pAE@@@@La@bcpCG`@@@@`~B@@@@@@@@@@@@@@A@@@@mD@@@P@@@@@|AT@@T@@ @@|oA@@`A@Xo@@@@I@D@@B@@@@B@`wI@@@@xA@`g@@P@@@@PKAT@@D@@@@@_@E@pA@@ @@VPPEK|KD@@@@@PA@@@pF@@@F@`}B@@@d@P@@H@@@@H@@~_g@@@@`G@@~_B@@A@@@@m DPA@P@@@@@|AT@@G@@@@XAAUlpoP@@@@@@B@@@@zK@@@pA@@@@@@@@@D@@@@tR@E@@A@@@@pG@@ @T@@@@@EBHNB@lPA@@@@SH`xH\pBE@@@@Pa@bcpAKT@@@@pDBHNBNlPA@@@@TH`xHxpBE@@@@La @bcPEKT@@@@@EBHNBUlPA@@@@SH`xHpqBE@@@@Pa@bc@GKT@@@@pDBHNBclPA@@@@TH`xHLrBE@ @@@La@bc`JKT@@@@@EBHNBjlPA@@@@SH`xHDsBE@@@@Pa@bcPLKT@@@@pDBHNBxlPA@@@@TH`xH `sBE@@@@La@bcpOKT@@@@@EBHNBlPA@@@@SH`xHXtBE@@@@Pa@bc`QKT@@@@pDBHNBMmPA@@@@ TH`xHttBE@@@@La@bc@UKT@@@@@EBHNBTmPA@@@@SH`xHluBE@@@@Pa@bcpVKT@@@@pDBHNBbmP A@@@@TH`xHHvBE@@@@La@bcPZKT@@@@@EBHNBimPA@@@@SH`xH@wBE@@@@Pa@bc@\KT@@@@pDBH NBwmPA@@@@TH`xH\wBE@@@@La@bc`_KT@@@@@EBHNB~mPA@@@@SH`xHTxBE@@@@Pa@bcPaKT@@@ @pDBHNBLnPA@@@@TH`xHpxBE@@@@La@bcpdKT@@@@@EBHNBSnPA@@@@SH`xHhyBE@@@@Pa@bc`f KT@@@@pDBHNBanPA@@@@TH`xHDzBE@@@@La@bc@jKT@@@@@EBHNBhnPA@@@@SH`xH|zBE@@@@Pa @bcpkKT@@@@pDBHNBvnPA@@@@TH`xHX{BE@@@@La@bcPoKT@@@@@EBHNB}nPA@@@@SH`xHP|BE@ @@@Pa@bc@qKT@@@@pDBHNBKoPA@@@@TH`xHl|BE@@@@La@bc`tKT@@@@@EBHNBRoPA@@@@SH`xH d}BE@@@@Pa@bcPvKT@@@@pDBHNB`oPA@@@@TH`xH@~BE@@@@La@bcpyKT@@@@@EBHNBgoPA@@@@ SH`xHx~BE@@@@Pa@bc`{KT@@@@pDBHNBuoPA@@@@TH`xHTBE@@@@La@bc@KT@@@@@EBHNB|oP A@@@@SH`xHL@CE@@@@Pa@bcp@LT@@@@pDBHNBJpPA@@@@TH`xHh@CE@@@@La@bcPDLT@@@@@EBH NBQpPA@@@@SH`xH`ACE@@@@Pa@bc@FLT@@@@pDBHNB_pPA@@@@TH`xH|ACE@@@@La@bc`ILT@@@ @@EBHNBfpPA@@@@SH`xHtBCE@@@@Pa@bcPKLT@@@@pDBHNBtpPA@@@@TH`xHPCCE@@@@La@bcpN LT@@@@@EBHNB{pPA@@@@SH`xHHDCE@@@@Pa@bc`PLT@@@@pDBHNBIqPA@@@@TH`xHdDCE@@@@La @bc@TL`@@@@`~B@@@@@@@@@@@@@@A@@@@mD@@@P@@@@@|AT@@T@@@@|oA@@`A@Xo@@@@I@D@@B@ @@@B@`wI@@@@xA@`g@@P@@@@PKAT@@D@@@@@_@E@pA@@@@VPPEK|KD@@@@@PA@@@pF @@@F@`}B@@@d@P@@H@@@@H@@~_g@@@@`G@@~_B@@A@@@@mDPA@P@@@@@|AT@@G@@@@XAA UlpoP@@@@@@B@@@@zK@@@pA@@@@@@@@@D@@@@tR@E@@A@@@@pG@@@T@@@@@EBHNBoQPA@@@@SH` xHXGAE@@@@Pa@bc`]DT@@@@pDBHNB}QPA@@@@TH`xHtGAE@@@@La@bc@aDT@@@@@EBHNBDRPA@@ @@SH`xHlHAE@@@@Pa@bcpbDT@@@@pDBHNBRRPA@@@@TH`xHHIAE@@@@La@bcPfDT@@@@@EBHNBY RPA@@@@SH`xH@JAE@@@@Pa@bc@hDT@@@@pDBHNBgRPA@@@@TH`xH\JAE@@@@La@bc`kDT@@@@@E BHNBnRPA@@@@SH`xHTKAE@@@@Pa@bcPmDT@@@@pDBHNB|RPA@@@@TH`xHpKAE@@@@La@bcppDT@ @@@@EBHNBCSPA@@@@SH`xHhLAE@@@@Pa@bc`rDT@@@@pDBHNBQSPA@@@@TH`xHDMAE@@@@La@bc @vDT@@@@@EBHNBXSPA@@@@SH`xH|MAE@@@@Pa@bcpwDT@@@@pDBHNBfSPA@@@@TH`xHXNAE@@@@ La@bcP{DT@@@@@EBHNBmSPA@@@@SH`xHPOAE@@@@Pa@bc@}DT@@@@pDBHNB{SPA@@@@TH`xHlOA E@@@@La@bc`@ET@@@@@EBHNBBTPA@@@@SH`xHdPAE@@@@Pa@bcPBET@@@@pDBHNBPTPA@@@@TH` xH@QAE@@@@La@bcpEET@@@@@EBHNBWTPA@@@@SH`xHxQAE@@@@Pa@bc`GET@@@@pDBHNBeTPA@@ @@TH`xHTRAE@@@@La@bc@KET@@@@@EBHNBlTPA@@@@SH`xHLSAE@@@@Pa@bcpLET@@@@pDBHNBz TPA@@@@TH`xHhSAE@@@@La@bcPPET@@@@@EBHNBAUPA@@@@SH`xH`TAE@@@@Pa@bc@RET@@@@pD BHNBOUPA@@@@TH`xH|TAE@@@@La@bc`UET@@@@@EBHNBVUPA@@@@SH`xHtUAE@@@@Pa@bcPWET@ @@@pDBHNBdUPA@@@@TH`xHPVAE@@@@La@bcpZET@@@@@EBHNBkUPA@@@@SH`xHHWAE@@@@Pa@bc `\ET@@@@pDBHNByUPA@@@@TH`xHdWAE@@@@La@bc@`ET@@@@@EBHNB@VPA@@@@SH`xH\XAE@@@@ Pa@bcpaET@@@@pDBHNBNVPA@@@@TH`xHxXAE@@@@La@bcPeET@@@@@EBHNBUVPA@@@@SH`xHpYA E@@@@Pa@bc@gET@@@@pDBHNBcVPA@@@@TH`xHLZAE@@@@La@bc`jET@@@@@EBHNBjVPA@@@@SH` xHD[AE@@@@Pa@bcPlET@@@@pDBHNBxVPA@@@@TH`xH`[AE@@@@La@bcpoE`@@@@`~B@@@@@@@@@ @@@@@A@@@@mD@@@P@@@@@|AT@@T@@@@|oA@@`A@Xo@@@@I@D@@B@@@@B@`wI@@@@xA@` g@@P@@@@PKAT@@D@@@@@_@E@pA@@@@VPPEK|KD@@@@@PA@@@pF@@@F@`}B@@@d@P@@H@@@@H@ @~_g@@@@`G@@~_B@@A@@@@mDPA@P@@@@@|AT@@G@@@@XAAUlpoP@@@@@@B@@@@zK@@@pA @@@@@@@@@D@@@@tR@E@@A@@@@pG@@@T@@@@@EBpM@PqPA@@@@SH@w@xECE@@@@Pa@\C`WLT@@@@ pDBpM@lqPA@@@@TH@w@pFCE@@@@La@\C`^LT@@@@@EBpM@zqPA@@@@SH@w@`HCE@@@@Pa@\C@bL T@@@@pDBpM@VrPA@@@@TH@w@XICE@@@@La@\C@iLT@@@@@EBpM@drPA@@@@SH@w@HKCE@@@@Pa@ \C`lLT@@@@pDBpM@@sPA@@@@TH@w@@LCE@@@@La@\C`sLT@@@@@EBpM@NsPA@@@@SH@w@pMCE@@ @@Pa@\C@wLT@@@@pDBpM@jsPA@@@@TH@w@hNCE@@@@La@\C@~LT@@@@@EBpM@xsPA@@@@SH@w@X PCE@@@@Pa@\C`AMT@@@@pDBpM@TtPA@@@@TH@w@PQCE@@@@La@\C`HMT@@@@@EBpM@btPA@@@@S H@w@@SCE@@@@Pa@\C@LMT@@@@pDBpM@~tPA@@@@TH@w@xSCE@@@@La@\C@SMT@@@@@EBpM@LuPA @@@@SH@w@lUCE@@@@Pa@\CpVMT@@@@pDBpM@iuPA@@@@TH@w@dVCE@@@@La@\Cp]MT@@@@@EBpM @wuPA@@@@SH@w@TXCE@@@@Pa@\CPaMT@@@@pDBpM@SvPA@@@@TH@w@LYCE@@@@La@\CPhMT@@@@ @EBpM@avPA@@@@SH@w@|ZCE@@@@Pa@\CpkMT@@@@pDBpM@}vPA@@@@TH@w@t[CE@@@@La@\CprM T@@@@@EBpM@KwPA@@@@SH@w@d]CE@@@@Pa@\CPvMT@@@@pDBpM@gwPA@@@@TH@w@\^CE@@@@La@ \CP}MT@@@@@EBpM@uwPA@@@@SH@w@L`CE@@@@Pa@\Cp@NT@@@@pDBpM@QxPA@@@@TH@w@DaCE@@ @@La@\CpGNT@@@@@EBpM@_xPA@@@@SH@w@tbCE@@@@Pa@\CPKNT@@@@pDBpM@{xPA@@@@TH@w@l cCE@@@@La@\CPRNT@@@@@EBpM@IyPA@@@@SH@w@\eCE@@@@Pa@\CpUNT@@@@pDBpM@eyPA@@@@T H@w@TfCE@@@@La@\Cp\NT@@@@@EBpM@syPA@@@@SH@w@DhCE@@@@Pa@\CP`NT@@@@pDBpM@OzPA @@@@TH@w@|hCE@@@@La@\CPgNT@@@@@EBpM@]zPA@@@@SH@w@ljCE@@@@Pa@\CpjNT@@@@pDBpM @yzPA@@@@TH@w@dkCE@@@@La@\CpqNT@@@@@EBpM@G{PA@@@@SH@w@TmCE@@@@Pa@\CPuNT@@@@ pDBpM@c{PA@@@@TH@w@LnCE@@@@La@\CP|N`@@@@`~B@@@@@@@@@@@@@@A@@@@mD@@@P@@@@@|A T@@T@@@@|oA@@`A@Xo@@@@I@D@@B@@@@B@`wI@@@@xA@`g@@P@@@@PKAT@@D@@@@@_@ E@pA@@@@VPPEK|KD@@@@@PA@@@pF@@@F@`}B@@@d@P@@H@@@@H@@~_g@@@@`G@@~_B@@ A@@@@mDPA@P@@@@@|AT@@G@@@@XAAUlpoP@@@@@@B@@@@zK@@@pA@@@@@@@@@D@@@@tR@E@@A@@ @@pG@@@T@@@@@EBHNBq{PA@@@@SH`xH`oCE@@@@Pa@bc@~NT@@@@pDBHNB{PA@@@@TH`xH|oCE @@@@La@bc`AOT@@@@@EBHNBF|PA@@@@SH`xHtpCE@@@@Pa@bcPCOT@@@@pDBHNBT|PA@@@@TH`x HPqCE@@@@La@bcpFOT@@@@@EBHNB[|PA@@@@SH`xHHrCE@@@@Pa@bc`HOT@@@@pDBHNBi|PA@@@ @TH`xHdrCE@@@@La@bc@LOT@@@@@EBHNBp|PA@@@@SH`xH\sCE@@@@Pa@bcpMOT@@@@pDBHNB~| PA@@@@TH`xHxsCE@@@@La@bcPQOT@@@@@EBHNBE}PA@@@@SH`xHptCE@@@@Pa@bc@SOT@@@@pDB HNBS}PA@@@@TH`xHLuCE@@@@La@bc`VOT@@@@@EBHNBZ}PA@@@@SH`xHDvCE@@@@Pa@bcPXOT@@ @@pDBHNBh}PA@@@@TH`xH`vCE@@@@La@bcp[OT@@@@@EBHNBo}PA@@@@SH`xHXwCE@@@@Pa@bc` ]OT@@@@pDBHNB}}PA@@@@TH`xHtwCE@@@@La@bc@aOT@@@@@EBHNBD~PA@@@@SH`xHlxCE@@@@P a@bcpbOT@@@@pDBHNBR~PA@@@@TH`xHHyCE@@@@La@bcPfOT@@@@@EBHNBY~PA@@@@SH`xH@zCE @@@@Pa@bc@hOT@@@@pDBHNBg~PA@@@@TH`xH\zCE@@@@La@bc`kOT@@@@@EBHNBn~PA@@@@SH`x HT{CE@@@@Pa@bcPmOT@@@@pDBHNB|~PA@@@@TH`xHp{CE@@@@La@bcppOT@@@@@EBHNBCPA@@@ @SH`xHh|CE@@@@Pa@bc`rOT@@@@pDBHNBQPA@@@@TH`xHD}CE@@@@La@bc@vOT@@@@@EBHNBX PA@@@@SH`xH|}CE@@@@Pa@bcpwOT@@@@pDBHNBfPA@@@@TH`xHX~CE@@@@La@bcP{OT@@@@@EB HNBmPA@@@@SH`xHPCE@@@@Pa@bc@}OT@@@@pDBHNB{PA@@@@TH`xHlCE@@@@La@bc`@PT@@ @@@EBHNBB@QA@@@@SH`xHd@DE@@@@Pa@bcPBPT@@@@pDBHNBP@QA@@@@TH`xH@ADE@@@@La@bcp EPT@@@@@EBHNBW@QA@@@@SH`xHxADE@@@@Pa@bc`GPT@@@@pDBHNBe@QA@@@@TH`xHTBDE@@@@L a@bc@KPT@@@@@EBHNBl@QA@@@@SH`xHLCDE@@@@Pa@bcpLPT@@@@pDBHNBz@QA@@@@TH`xHhCDE @@@@La@bcPPP`@@@@`~B@@@@@@@@@@@@@@A@@@@mD@@@P@@@@@|AT@@T@@@@|oA@@`A@Xo@@@@I @D@@B@@@@B@`wI@@@@xA@`g@@P@@@@PKAT@@D@@@@@_@E@pA@@@@VPPEK|KD@@@@@PA @@@pF@@@F@`}B@@@d@P@@H@@@@H@@~_g@@@@`G@@~_B@@A@@@@mDPA@P@@@@@|AT@@G@ @@@XAAUlpoP@@@@@@B@@@@zK@@@pA@@@@@@@@@D@@@@tR@E@@A@@@@pG@@@T@@@@@EBpM@NGPA@ @@@SH@w@p]@E@@@@Pa@\C@wAT@@@@pDBpM@jGPA@@@@TH@w@h^@E@@@@La@\C@~AT@@@@@EBpM@ xGPA@@@@SH@w@X`@E@@@@Pa@\C`ABT@@@@pDBpM@THPA@@@@TH@w@Pa@E@@@@La@\C`HBT@@@@@ EBpM@bHPA@@@@SH@w@@c@E@@@@Pa@\C@LBT@@@@pDBpM@~HPA@@@@TH@w@xc@E@@@@La@\C@SBT @@@@@EBpM@LIPA@@@@SH@w@he@E@@@@Pa@\C`VBT@@@@pDBpM@hIPA@@@@TH@w@`f@E@@@@La@\ C`]BT@@@@@EBpM@vIPA@@@@SH@w@Ph@E@@@@Pa@\C@aBT@@@@pDBpM@RJPA@@@@TH@w@Hi@E@@@ @La@\C@hBT@@@@@EBpM@`JPA@@@@SH@w@xj@E@@@@Pa@\C`kBT@@@@pDBpM@|JPA@@@@TH@w@pk @E@@@@La@\C`rBT@@@@@EBpM@JKPA@@@@SH@w@`m@E@@@@Pa@\C@vBT@@@@pDBpM@fKPA@@@@TH @w@Xn@E@@@@La@\C@}BT@@@@@EBpM@tKPA@@@@SH@w@Hp@E@@@@Pa@\C`@CT@@@@pDBpM@PLPA@ @@@TH@w@@q@E@@@@La@\C`GCT@@@@@EBpM@^LPA@@@@SH@w@pr@E@@@@Pa@\C@KCT@@@@pDBpM@ zLPA@@@@TH@w@hs@E@@@@La@\C@RCT@@@@@EBpM@HMPA@@@@SH@w@Xu@E@@@@Pa@\C`UCT@@@@p DBpM@dMPA@@@@TH@w@Pv@E@@@@La@\Cp\CT@@@@@EBpM@sMPA@@@@SH@w@Dx@E@@@@Pa@\CP`CT @@@@pDBpM@ONPA@@@@TH@w@|x@E@@@@La@\CPgCT@@@@@EBpM@]NPA@@@@SH@w@lz@E@@@@Pa@\ CpjCT@@@@pDBpM@yNPA@@@@TH@w@d{@E@@@@La@\CpqCT@@@@@EBpM@GOPA@@@@SH@w@T}@E@@@ @Pa@\CPuCT@@@@pDBpM@cOPA@@@@TH@w@L~@E@@@@La@\CP|CT@@@@@EBpM@qOPA@@@@SH@w@| @E@@@@Pa@\CpCT@@@@pDBpM@MPPA@@@@TH@w@t@AE@@@@La@\CpFDT@@@@@EBpM@[PPA@@@@SH @w@dBAE@@@@Pa@\CPJDT@@@@pDBpM@wPPA@@@@TH@w@\CAE@@@@La@\CPQDT@@@@@EBpM@EQPA@ @@@SH@w@LEAE@@@@Pa@\CpTDT@@@@pDBpM@aQPA@@@@TH@w@DFAE@@@@La@\Cp[D`@@@@`~B@@@ @@@@@@@@@@@A@@@@mD@@@P@@@@@|AT@@T@@@@|oA@@`A@Xo@@@@I@D@@B@@@@B@`wI@@@@x A@`g@@P@@@@PKAT@@D@@@@@_@E@pA@@@@VPPEK|KD@@@@@PA@@@pF@@@F@`}B@@@d@P@@H @@@@H@@~_g@@@@`G@@~_B@@A@@@@mDPA@P@@@@@|AT@@G@@@@XAAUlpoP@@@@@@B@@@@z K@@@pA@@@@@@@@@D@@@@tR@E@@A@@@@pG@@@T@@@@@EBHNB~APA@@@@SH`xHTH@E@@@@Pa@bcPa @T@@@@pDBHNBLBPA@@@@TH`xHpH@E@@@@La@bcpd@T@@@@@EBHNBSBPA@@@@SH`xHhI@E@@@@Pa @bc`f@T@@@@pDBHNBaBPA@@@@TH`xHDJ@E@@@@La@bc@j@T@@@@@EBHNBhBPA@@@@SH`xH|J@E@ @@@Pa@bcpk@T@@@@pDBHNBvBPA@@@@TH`xHXK@E@@@@La@bcPo@T@@@@@EBHNB}BPA@@@@SH`xH PL@E@@@@Pa@bc@q@T@@@@pDBHNBKCPA@@@@TH`xHlL@E@@@@La@bc`t@T@@@@@EBHNBRCPA@@@@ SH`xHdM@E@@@@Pa@bcPv@T@@@@pDBHNB`CPA@@@@TH`xH@N@E@@@@La@bcpy@T@@@@@EBHNBgCP A@@@@SH`xHxN@E@@@@Pa@bc`{@T@@@@pDBHNBuCPA@@@@TH`xHTO@E@@@@La@bc@@T@@@@@EBH NB|CPA@@@@SH`xHLP@E@@@@Pa@bcp@AT@@@@pDBHNBJDPA@@@@TH`xHhP@E@@@@La@bcPDAT@@@ @@EBHNBQDPA@@@@SH`xH`Q@E@@@@Pa@bc@FAT@@@@pDBHNB_DPA@@@@TH`xH|Q@E@@@@La@bc`I AT@@@@@EBHNBfDPA@@@@SH`xHtR@E@@@@Pa@bcPKAT@@@@pDBHNBtDPA@@@@TH`xHPS@E@@@@La @bcpNAT@@@@@EBHNB{DPA@@@@SH`xHHT@E@@@@Pa@bc`PAT@@@@pDBHNBIEPA@@@@TH`xHdT@E@ @@@La@bc@TAT@@@@@EBHNBPEPA@@@@SH`xH\U@E@@@@Pa@bcpUAT@@@@pDBHNB^EPA@@@@TH`xH xU@E@@@@La@bcPYAT@@@@@EBHNBeEPA@@@@SH`xHpV@E@@@@Pa@bc@[AT@@@@pDBHNBsEPA@@@@ TH`xHLW@E@@@@La@bc`^AT@@@@@EBHNBzEPA@@@@SH`xHDX@E@@@@Pa@bcP`AT@@@@pDBHNBHFP A@@@@TH`xH`X@E@@@@La@bcpcAT@@@@@EBHNBOFPA@@@@SH`xHXY@E@@@@Pa@bc`eAT@@@@pDBH NB]FPA@@@@TH`xHtY@E@@@@La@bc@iAT@@@@@EBHNBdFPA@@@@SH`xHlZ@E@@@@Pa@bcpjAT@@@ @pDBHNBrFPA@@@@TH`xHH[@E@@@@La@bcPnAT@@@@@EBHNByFPA@@@@SH`xH@\@E@@@@Pa@bc@p AT@@@@pDBHNBGGPA@@@@TH`xH\\@E@@@@La@bc`sA`@@@@`~B@@@@@@@@@@@@@@A@@@@mD@@@P@ @@@@|AT@@T@@@@|oA@@`A@Xo@@@@I@D@@B@@@@B@`wI@@@@xA@`g@@P@@@@PKAT@@D@ @@@@_@E@pA@@@@VPPEK|KD@@@@@PA@@@pF@@@F@`}B@@@d@P@@H@@@@H@@~_g@@@@`G@@ ~_B@@A@@@@mDPA@P@@@@@|AT@@G@@@@XAAUlpoP@@@@@@E@@@@[@@@X@@vK@@@PB@A@`@@@@` @@x}_B@@@@~_@@xI@@D@@@@tR@E@@A@@@@pGPA@`@@@@`~B@@@@@@@@@@@@@@A@@@@mDP A@P@@@@@|A@@@G@@@@po@@@pO@@@@A@@@@mD@@@P@@@@@|AD@@\@@@@lo@P@pA@@@@@@@oB@ @@@@P@BH`HSew\tUV[@TYZQXVe@@`B@DB@JF@@@@@@A@@@@p^|S@@A@@@@mDP@@L@@@@@@@@ %%%%%%%%%%%%%%%%%%%%%% End /document/N4QRHX00.wmf %%%%%%%%%%%%%%%%%%%%% %%%%%%%%%%%%%%%%%%%%% Start /document/N4QRHX0F.xvz %%%%%%%%%%%%%%%%%%%% ||C^mqFHvUf\sev[nucHqxBLb|cOJpSHD}tPTeETEArPayf]aMGHSeuTTUTS`HR[uAG[oQW[lyB YtQfH~h@OCEf[vEv\`dFY}HB\l}F]OIfZeMF]qXcH`Lt[nMWZsQWYnQWObDcH`pTXy}V]tucHTE fXuqVXrIBHF}v[tUf\F}f[tucHfDF\oMwNsEf[sur\eIWZfYRXp}v\{@RLrHBHOUG]pUG]UyVZt MWObtTSb@BReevYhQWOb\SMnTSLtHcH`Xt[oQWYrED[i]f[mUf[tucHCUf[tUf\b@bPaMvZgIw[ uyFYC}F[oIWObLbQFYdQFYdH`Ht[rQVYrMt[l}f\}HrHx@CNp`CLb@bPoIGYeIwUiQF]hucHpHB HLUfYtuTXr]VZnucHqHBHAUG]oAE[aeWOb@cH`HUZgaF]MEf\gef[}HRLb@rUiQF]hucHqDsLnT CNx`cH`Dd[iuVXtev[nME]yqVY}HbTuyvSnMVYb@rTpEvXiyvY}HRLb@rPoqV]myv\}HBLb@bTo ]w\}HBLb@BReEFYeIgQoyF]}HbIaAw[sms\ayv\mLWYrefYfDF\oMwN`DcLb@BReEFYeIWPlevY nuVYnQWObLTYnQWYrIBHB}F]t}V[MEf\gef[}HRLb@BUoAWSaIwYiyVObDcH~h@OSMVYnUfLdME ]yqVY`HTXcmvYr}V]nQFUrEf[sAWXrUf[tucHpHBHF}v[tUf\F}f[tucHfDF\oMwNsEf[sur\eI WZfYRXp}v\{@RLrHBHHUVZgaF]}HBNpHBHLUvYeyFYPqVXcUV[eyF]}HbPoQG]oufH`HTXcmvYr }V]nQvPoqv[rucHcXdQFYdQFIBHF}v[tUf\AqVZgyV[eyF]}HrPeyF]eIgH`pTYgUf[dYt[nQWO bXRXp}v\{LWXnMWKsUf\iYfIaAw[smCHxHBHB}f\dUf\C}F[oIWObLBNp`CLx@cH`Ht[rQVYr]U ZdQGZ}HBLb@BSe]VYnQVPlevYnuVYnQWObLTYnQWYrIBHLUfYtuTXr]VZnucHqHBHRevYhQWSaI wYiyVObDcH`\UZdQGZ}HRLr@cH`pTYgUf[dYUZsefXlUVOb@cH``TYaQVYrYt[nQWObXRXp}v\{ LWXnMWKsUf\iYfIaAw[smCHqHcH``TYaQVYrED[i]f[mUf[tucHCUf[tUf\b@bPoQG]ouVSaIwY iyVObDcH`Pu[puTXr]VZnucHqHrK~h@OC}v[rQVZnEF]eMU^sQWYmICYSQW^lUFHSMVXlef[guc HUyvXoyv\tIWXiyVYdIBHXQUZcmv\NUW[bUf\}HbSoIW[aqfH``UPxev\TeF]lUVPlevYnuVYnQ WObTd[dIBHYQUZcmv\Vev\iIF[eucHqHBHSUgXgIWZdqTZnUvPoqv[rucHcLtPCMtPCIBHSUgXg IWZdqTZnUvUiQF]hucHpxRLb@RVTevXkMGSaIVYlMgUiMWZbqVY}HRLb@RPxUv\Lef[eMt[l}f\ }HrHp@CLp@CLb@RPxUv\Lef[e]UZdQGZ}HBLnDCNb@BUiMvZsqTXbUF[F}f[tucHfDF\oMwNTeV [eMGHNUv]`Hu[mEf[fDF\oMwN``CHc@CLp@CLpHBHYED^iMGUiQG[eED[i]f[mUf[tucHEyFYb@ RVTevXkMgSuufXeIWObxt[ruVXlIBHC}v[rQVZnEF]eQU^pUVObpTZnqTZnIBHGIWZdqTZnUvTt eG[eucHS}F[iQfH``uQreFYVev\iIF[eucHpHBHGIWZded[FIw[nQWOb@cH``UPxev\Vev\iIF[ eucHqHBHSUgXgIWZdqTZnUvTteG[eucHS}F[iQfH``uTuIvYreFYVev\iIF[eucHpHBHAaWYsQU ZtqVYF}f[tucHfDF\oMwNTeV[eMGHNUv]`Hu[mEf[fDF\oMwN`DcL`LBLp@CLp@cH``EUiMvZsq TXbUF[SQW^lUVOb`t[ref^oyF]aqfH``EUiMvZsITYt]WYeyVObDcH`duQreFYVev\iIF[eucHp HBHYED^iMGUiQG[e}d\iUf[tEF]i}f[}HBRoIWZz}f[tEF[b@RPxUv\}HRPuQw[mEF]iMfH`DD^ eMWRnYd\oyF]}HBLb@RVAaWZsYUZsefXlUVObDcH``EUiMvZsYUZsefXlUVObDcH`PUZcmv\LUf [gQGZ}HbLb@rQreFYLef[eMt[l}f\}HrHydSNydSNb@RVSUgXgIWZdYUZsefXlUVOb@cH``EUiM vZsqTXbUF[sYUZsefXlUVObDcH`\d\iQFSiyVYWeFYtaVOb@cKqHBHYQUZcmv\LEfXeqvTteG[e ucHH}f\iiw[nQWXlIBHYQUZcmv\BUF]wUVYnucHqHBHAaWYsQUZpMWOb@cHoxcB||dXjME]yqVY `PU^pUVObPTYfEV]lQgH`XTZlqvPoqv[rucHcXdQp@CLpHBHLef[eMt[l}f\rtcHcXdQqPSNsHB HVUf\tevXaqVPseW[pQw[tUv\SQW^lUVObPTXsaVYdIBHFeF[lAUXtQWYryVObPTZa]v[nEF[Le f[eMgH`PUZmUfPe]VZnucHpHBHXuTYsaVObDSLb@BUuIV]lEf\}HBLb@BUiAGSeyvYtaVObPcH` XUYrQWZcEF[AMW^mAG]oQWYsYUZsefXlUVObDcH`PUZtqVYF}f[tucHfDF\oMwNsEf[sur\eIWZ fYRXp}v\{@RLqHBHLef[eME]yqVY}HrToqVZdIBHLef[eMt[l}f\Def\eMF]i}f[XucHpHBHOIG ]h}vYoyVXlAe\oiVYcQWZoyVOb@cH`DdYfUvXtYUZe]WZn]fPoaWObDcH`pTZnUvPoqv[rQTZrU vXtev[neUObDcH`PUZtqVYAqVZgyV[eyF]}HrPeyF]eIgH`pTZnUvPoqv[rQTZrUvXtev[niUOb DcH`Xd\auVYsucHu@cH`@u[iyF]Sef^eucHqxRMb@RPrIw[wqTYn]F]hucHPIw[p}f\tev[nEF[ b@RUMUv\hucHrTcH`XTZlqvPoqv[rQU^pUVObPTZcaf\ouVXtevXb@BVLef[eMgUiMWZbqVY}HR Lb@bUiMWZbqVYAYF]eIWQnQVObDcH`pTZgaF]C}F[oIWObLbQFYdQFYdH`DDYaAG]iYWYMUv\hu cHpHBHXMU]buVYsaVOb@cH`TD^tUf[sev[nucHFef[iQWYb@BSiyVYsYUZsefXlUVObDcH`dUSe MGZ}HRLqHBHFeF[lMt[l}f\rtcHcXCMyTSQDIBHSaVXdef[gucHSuv[oQGZb@BToef[tME]yqVY }HbQiqF[eQvPiIwXlUv\b@bQiqF[C}F[oIGQiIWYcQWZoyFV}HBLb@BUiuVYEyFY}HRLpHBHCEV [eIWXC}v[rQVZnEF]eMWOb@cH`TESiyVYsYUZsefXlUVObDcH`XUZsefXlUfPeYv[rUfPe]VZnu cHqHBHFeF[lME]yqVY}HRQvUf[OQFYb@bQiqF[C}F[oIGQiIWYcQWZoyVV}HBLb@bUeIG]iMVXl Et\yuF\t}F]eMwPoqv[rucHc`CLx@CNpHBHUMU]buVYsaVOb@cH`XTZlqvPoqv[rQTZrUvXtev[ niUObDcH`XUYrQWZcEF[AMW^mAG]oQWYs]UZdQGZ}HBLnHcH`XUSeMGZ}HbLuHBHYqTZnUv\Vev \iIF[eucHqHBHYMU]buVYsaVOb@cH`PU]bUFQiEV[eQWYrucHrxRMb@bQiqF[eQVObDcH`hUSeM GZ}HRLqHBHP}VZnQw\Vev\iIF[eucHpHBHLef[eMt[l}f\}HrHp@CLpXdQb@BSiyVYWeFYtaVOb @cKsTcH`pTYgUf[dUd[tIW^}HBLb@bULef[eMgUiMWZbqVY}HRLb@bUSUgXmUv\hucHpHBHTeF\ AyvYlUVOb@cKtDCNx\SNpHCLt\SNb@BUiAwTteG[eucHFeF[lUFYb@rReUF\UAgUeMF]oIWObDc H`PTZsMv[nQWZnUWZtewTeEf\caVObDcH`tTYsafUiMWZbqVY}HBLb@rPl}v\eQVOb@cH`@u[iy F]C}F[oIWObLRLyDSNw@cH`pTZnUvPoqv[rQU^pUVObXD[aQgHoxcB||dXjME]yqVY`PU^pUVOb LT]rYWYrPfH`pTYgUf[dMt[l}f\}HBSiyVYC}F[oIgH`pTYgUf[dUd[tIW^}HRLb@RUMUv\hucH qHSLb|bOJpsTcUf[eICY`dFY}HB\l}F]OIfZeMF]q\cH`Lt[nMWZsQWYnQWObDcH`pTYfQWOb@c H`Ht[tQw[mucHpHbO|Lt[oIGYiyVXtUvTyMG]eufLdME]yqVY`DD^eMGUiQG[eYt[nQWObXRXp} v\{PUZmUv\`xTYwAbTouVXnYRXp}v\{@RLr@rHp@CLp@CLb@BUiMvZsqTXbUF[F}f[tucHfDF\o MwNTeV[eMGHNUv]`Hu[mEf[fDF\oMwN``CHc@CLp@CLpHBHSMVXlef[gucHUyvXoyv\tIWXiyVY dIBHAaWYsucHAUG]ouVXtevXb@RPxUv\TeF\sucHpHrK~h@OOIfZSQW^lUFHTeG\eucHCUg\vUf LdIBHLUvYeyFYC}F[oIWObpTZnUvPoqv[rIrK~h@OC}v[rQVZnEF]eMU^sQWYmICY`dFY}HB\l} F]OIfZeMF]q`cH`Lt[nMWZsQWYnQWObDcH`DD^eMGUiQG[eYt[nQWObXRXp}v\{PUZmUv\`xTYw AbTouVXnYRXp}v\{@RLr@rHp@CLp@CLb@BUiMvZsqTXbUF[F}f[tucHfDF\oMwNTeV[eMGHNUv] `Hu[mEf[fDF\oMwN``CHc@CLp@CLpHBHSMVXlef[gucHUyvXoyv\tIWXiyVYdIBHAaWYsucHAUG ]ouVXtevXb@RPxUv\TeF\sucHpHBHAUG]oYUZe]WZn]fPoaWObDSMbxCOAaWYs}d\i]VZnaEHVE F[}HBLbxCL||RPxUv\OIWZgef[XycB|DD^eMwSrevYiyVV`XUXlucHpHbOppsKAaWYs}d\i]VZn eeOJpCVAaWZsQUZtqVY~`GOo`UPxev\TeF]lUfOJpSVAaWZsQUZtqVY~dGOodUPxev\TeF]lUfO JpCVTevXkMWPnMFZoIGHVEF[}HBLbxCL||BVTevXkMWPnMFZoIgOJpCVTevXkMGQiMG]ayvXeAb UaqVOb@cH~@COo`EUiMvZsQTZsQWXnMVY~h@OYQUZcmv\AyvXh}f\`XUXlucHpHbOppsKYQUZcm v\AyvXh}f\~h@OYQUZcmv\Dev\tEf[cUFHVEF[}HBLbxCL||RVTevXkMGQiMG]ayvXeycB||dXj ME]yqVY`PU^pUVObLT]rYWYrPfH`pTYgUf[dMt[l}f\}HBSiyVYC}F[oIgHoxcB||dXjICY`PU^ pUVObLT]rYWYrPfH`dFY}HB\l}F]OIfZeMF]q@cH`Lt[nMWZsQWYnQWObDcH`XUZsefXlUVObDc H`@u[iyF]SQW^lUVObXTZlqVYdMTZrMF[eMgH`pTZnUvUiQF]hucHpxRMb@BSiyVYSQW^lUVObL u[leFYb@RUMUv\hucHq@CLb@BSiyVYC}F[oIGUyAWY}HbQlEF]b@BSiyVYC}F[oIWObLBLp@CLp @cH`pTZnUv\Vev\iIF[eucHqHBHDev\c}f[tef[ueF]yMUYaIwXhucHqHbO|TD^pIGHOAG]}HRU NEV[eIbOqQC^||RQxAg\~h@OEaG\rArSpQWObTUSiyfH~@COoTD^pIgOJpSQxAg\`|D\tucHUuT XxIbOqpsKEaG\rycB|TD^pIGHOAG]}HBVFUg[cQWZoyfH~DGMxqsKEaG\rycB|TD^pIGHOAG]}H RVFUg[cQWZoyfH~DGMxqsKEaG\rycB|dT[gICY~pCToqV^rPFHFeF[lUFY}HBLb@rPl}v\eQVOb @cH~pCTrxCL`@COo@eL~h@OPIcOpxBLq@SLpDCLq@SLpDCHpxBLq@SLpDCLq@SLpDCOo@eL~h@O PIcOpxBLr@cLpHCLr@cLpHCHpxBLr@cLpHCLr@cLpHCOo@eL~h@OPIcOpxBLs@sLpLCLs@sLpLC HpxBLs@sLpLCLs@sLpLCOo@eL~h@OPIcOpxBLt@CMpPCLt@CMpPCHpxBLt@CMpPCLt@CMpPCOo@ eL~h@OPIcOpxBLu@SMpTCLu@SMpTSL`@cKpTCLu@SMpTCLu@SMqpsKPIcOJpCTrxCLn@cMpXCLv @cMpXCLvDCHpxBLv@cMpXCLv@cMpXSL||BTrxcB|@eL~@cKp\CLw@sMp\CLw@sMq@BLn@sMp\CL w@sMp\CLwDCOo@eL~h@OPIcOpxBLx@CNp`CLx@CNp`SL`@cKp`CLx@CNp`CLx@CNqpsKPIcOJpC TrxCLn@SNpdCLy@SNpdCLyDCHpxBLy@SNpdCLy@SNpdSL||BTrxcB|@eL~@cKq@SLpDCLq@SLpD CHpxRLpDCLq@SLpDCLqpsKPIcOJpCTrxCLnDSLqDSLqDSLqDSLq@BLnDSLqDSLqDSLqDSLqpsKP IcOJpCTrxCLnDcLqHSLrDcLqHSLr@BLnDcLqHSLrDcLqHSLrpsKPIcOJpCTrxCLnDsLqLSLsDsL qLSLs@BLnDsLqLSLsDsLqLSLspsKPIcOJpCTrxCLnDCMqPSLtDCMqPSLt@BLnDCMqPSLtDCMqPS LtpsKPIcOJpCTrxCLnDSMqTSLuDSMqTSLu@BLnDSMqTSLuDSMqTSLupsKPIcOJpCTrxCLnDcMqX SLvDcMqXSLv@BLnDcMqXSLvDcMqXSLvpsKPIcOJpCTrxCLnDsMq\SLwDsMq\SLw@BLnDsMq\SLw DsMq\SLwpsKPIcOJpCTrxCLnDCNq`SLxDCNq`SLx@BLnDCNq`SLxDCNq`SLxpsKPIcOJpCTrxCL nDSNqdSLyDSNqdSLy@BLnDSNqdSLyDSNqdSLypsKPIcOJpCTrxCLnHCLr@cLpHCLr@cL`@cKr@c LpHCLr@cLpHCOo@eL~h@OPIcOpxbLqHSLrDcLqHSLrDCHpxbLqHSLrDcLqHSLrDCOo@eL~h@OPI cOpxbLrHcLrHcLrHcLrHCHpxbLrHcLrHcLrHcLrHCOo@eL~h@OPIcOpxbLsHsLrLcLsHsLrLCHp xbLsHsLrLcLsHsLrLCOo@eL~h@OPIcOpxbLtHCMrPcLtHCMrPCHpxbLtHCMrPcLtHCMrPCOo@eL ~h@OPIcOpxbLuHSMrTcLuHSMrTCHpxbLuHSMrTcLuHSMrTCOo@eL~h@OPIcOpxbLvHcMrXcLvHc MrXCHpxbLvHcMrXcLvHcMrXCOo@eL~h@OPIcOpxbLwHsMr\cLwHsMr\CHpxbLwHsMr\cLwHsMr\ COo@eL~h@OPIcOpxbLxHCNr`cLxHCNr`CHpxbLxHCNr`cLxHCNr`COo@eL~h@OPIcOpxbLyHSNr dcLyHSNrdCHpxbLyHSNrdcLyHSNrdCOo@eL~h@OPIcOpxrLpLCLs@sLpLCLs@BLnLCLs@sLpLCL s@sL||BTrxcB|@eL~@cKsDsLqLSLsDsLqLSL`@cKsDsLqLSLsDsLqLSL||BTrxcB|@eL~@cKsHs LrLcLsHsLrLcL`@cKsHsLrLcLsHsLrLcL||BTrxcB|@eL~@cKsLsLsLsLsLsLsLsL`@cKsLsLsL sLsLsLsLsL||BTrxcB|@eL~@cKsPsLtLCMsPsLtLCM`@cKsPsLtLCMsPsLtLCM||BTrxcB|@eL~ @cKsTsLuLSMsTsLuLSM`@cKsTsLuLSMsTsLuLSM||BTrxcB|@eL~@cKsXsLvLcMsXsLvLcM`@cK sXsLvLcMsXsLvLcM||BTrxcB|@eL~@cKs\sLwLsMs\sLwLsM`@cKs\sLwLsMs\sLwLsM||BTrxc B|@eL~@cKs`sLxLCNs`sLxLCN`@cKs`sLxLCNs`sLxLCN||BTrxcB|@eL~@cKsdsLyLSNsdsLyL SN`@cKsdsLyLSNsdsLyLSN||BTrxcB|@eL~@cKt@CMpPCLt@CMpPCHpxBMpPCLt@CMpPCLtpsKP IcOJpCTrxCLnPSLtDCMqPSLtDCMq@BLnPSLtDCMqPSLtDCMqpsKPIcOJpCTrxCLnPcLtHCMrPcL tHCMr@BLnPcLtHCMrPcLtHCMrpsKPIcOJpCTrxCLnPsLtLCMsPsLtLCMs@BLnPsLtLCMsPsLtLC MspsKPIcOJpCTrxCLnPCMtPCMtPCMtPCMt@BLnPCMtPCMtPCMtPCMtpsKPIcOJpCTrxCLnPSMtT CMuPSMtTCMu@BLnPSMtTCMuPSMtTCMupsKPIcOJpCTrxCLnPcMtXCMvPcMtXCMv@BLnPcMtXCMv PcMtXCMvpsKPIcOJpCTrxCLnPsMt\CMwPsMt\CMw@BLnPsMt\CMwPsMt\CMwpsKPIcOJpCTrxCL nPCNt`CMxPCNt`CMx@BLnPCNt`CMxPCNt`CMxpsKPIcOJpCTrxCLnPSNtdCMyPSNtdCMy@BLnPS NtdCMyPSNtdCMypsKPIcOJpCTrxCLnTCLu@SMpTCLu@SMq@BLnTCLu@SMpTCLu@SMqpsKPIcOJp CTrxCLnTSLuDSMqTSLuDSMr@BLnTSLuDSMqTSLuDSMrpsKPIcOJpCTrxCLnTcLuHSMrTcLuHSMs @BLnTcLuHSMrTcLuHSMspsKPIcOJpCTrxCLnTsLuLSMsTsLuLSMt@BLnTsLuLSMsTsLuLSMtpsK PIcOJpCTrxCLnTCMuPSMtTCMuPSMu@BLnTCMuPSMtTCMuPSMupsKPIcOJpCTrxCLnTSMuTSMuTS MuTSMv@BLnTSMuTSMuTSMuTSMvpsKPIcOJpCTrxCLnTcMuXSMvTcMuXSMw@BLnTcMuXSMvTcMuX SMwpsKPIcOJpCTrxCLnTsMu\SMwTsMu\SMx@BLnTsMu\SMwTsMu\SMxpsKPIcOJpCTrxCLnTCNu `SMxTCNu`SMy@BLnTCNu`SMxTCNu`SMypsKPIcOJpCTrxCLnTSNudSMyTSNudcM`@cKudSMyTSN udSMyXCOo@eL~h@OPIcOpxbMpXCLv@cMpXCLvDCHpxbMpXCLv@cMpXCLvDCOo@eL~h@OPIcOpxb MqXSLvDcMqXSLvHCHpxbMqXSLvDcMqXSLvHCOo@eL~h@OPIcOpxbMrXcLvHcMrXcLvLCHpxbMrX cLvHcMrXcLvLCOo@eL~h@OPIcOpxbMsXsLvLcMsXsLvPCHpxbMsXsLvLcMsXsLvPCOo@eL~h@OP IcOpxbMtXCMvPcMtXCMvTCHpxbMtXCMvPcMtXCMvTCOo@eL~h@OPIcOpxbMuXSMvTcMuXSMvXCH pxbMuXSMvTcMuXSMvXCOo@eL~h@OPIcOpxbMvXcMvXcMvXcMv\CHpxbMvXcMvXcMvXcMv\COo@e L~h@OPIcOpxbMwXsMv\cMwXsMv`CHpxbMwXsMv\cMwXsMv`COo@eL~h@OPIcOpxbMxXCNv`cMxX CNvdCHpxbMxXCNv`cMxXCNvdCOo@eL~h@OPIcOpxbMyXSNvdcMyXSNw@BLnXSNvdcMyXSNvdsM| |BTrxcB|@eL~@cKw@sMp\CLw@sMp\SL`@cKw@sMp\CLw@sMp\SL||BTrxcB|@eL~@cKwDsMq\SL wDsMq\cL`@cKwDsMq\SLwDsMq\cL||BTrxcB|@eL~@cKwHsMr\cLwHsMr\sL`@cKwHsMr\cLwHs Mr\sL||BTrxcB|@eL~@cKwLsMs\sLwLsMs\CM`@cKwLsMs\sLwLsMs\CM||BTrxcB|@eL~@cKwP sMt\CMwPsMt\SM`@cKwPsMt\CMwPsMt\SM||BTrxcB|@eL~@cKwTsMu\SMwTsMu\cM`@cKwTsMu \SMwTsMu\cM||BTrxcB|@eL~@cKwXsMv\cMwXsMv\sM`@cKwXsMv\cMwXsMv\sM||BTrxcB|@eL ~@cKw\sMw\sMw\sMw\CN`@cKw\sMw\sMw\sMw\CN||BTrxcB|@eL~@cKw`sMx\CNw`sMx\SN`@c Kw`sMx\CNw`sMx\SN||BTrxcB|@eL~@cKwdsMy\SNwdsMy`CHpxrMy\SNwdsMy\SNxpsKPIcOJp CTrxCLn`CLx@CNp`CLx@CNq@BLn`CLx@CNp`CLx@CNqpsKPIcOJpCTrxCLn`SLxDCNq`SLxDCNr @BLn`SLxDCNq`SLxDCNrpsKPIcOJpCTrxCLn`cLxHCNr`cLxHCNs@BLn`cLxHCNr`cLxHCNspsK PIcOJpCTrxCLn`sLxLCNs`sLxLCNt@BLn`sLxLCNs`sLxLCNtpsKPIcOJpCTrxCLn`CMxPCNt`C MxPCNu@BLn`CMxPCNt`CMxPCNupsKPIcOJpCTrxCLn`SMxTCNu`SMxTCNv@BLn`SMxTCNu`SMxT CNvpsKPIcOJpCTrxCLn`cMxXCNv`cMxXCNw@BLn`cMxXCNv`cMxXCNwpsKPIcOJpCTrxCLn`sMx \CNw`sMx\CNx@BLn`sMx\CNw`sMx\CNxpsKPIcOJpCTrxCLn`CNx`CNx`CNx`CNy@BLn`CNx`CN x`CNx`CNypsKPIcOJpCTrxCLn`SNxdCNy`SNxdSN`@cKxdCNy`SNxdCNydCOo@eL~h@OPIcOpxR NpdCLy@SNpdCLyDCHpxRNpdCLy@SNpdCLyDCOo@eL~h@OPIcOpxRNqdSLyDSNqdSLyHCHpxRNqd SLyDSNqdSLyHCOo@eL~h@OPIcOpxRNrdcLyHSNrdcLyLCHpxRNrdcLyHSNrdcLyLCOo@eL~h@OP IcOpxRNsdsLyLSNsdsLyPCHpxRNsdsLyLSNsdsLyPCOo@eL~h@OPIcOpxRNtdCMyPSNtdCMyTCH pxRNtdCMyPSNtdCMyTCOo@eL~h@OPIcOpxRNudSMyTSNudSMyXCHpxRNudSMyTSNudSMyXCOo@e L~h@OPIcOpxRNvdcMyXSNvdcMy\CHpxRNvdcMyXSNvdcMy\COo@eL~h@OPIcOpxRNwdsMy\SNwd sMy`CHpxRNwdsMy\SNwdsMy`COo@eL~h@OPIcOpxRNxdCNy`SNxdCNydCHpxRNxdCNy`SNxdCNy dCOo@eL~h@OPIcOq@RL||BTrxcB||BToqV^rPfOJpsKIuvYrPfOJpsKOIfZrPfOJpsSbifLdABU yAWY}HrPuIg]eICYb@RZducHpqv[t}dXjUvXtESLb@rPoyv\iMG]eyF]}HRLb@bUiMWZbqVY}HR Lb@BToef[tME]yqVY}HbQiqF[eQvPiIwXlUv\b@BSiyVYWeFYtaVOb@cKuHBHLef[eME]yqVY}H rToqVZdIBHUuTYsaVObDCLpHBHLef[eMt[l}f\TeG\eucHFqVXtIBHLef[eMt[l}f\}HrHp@CLp @CLb@BSiyVYsYUZsefXlUVObDcH`PTZsMv[nQWZnUWZtewTeEf\caVObDcH~pSQxAg\`|D\tucH UyTXmUfH~DGMxqsKEaG\rycB|TD^pIGHOAG]}HRUMef[bxSKqpsKEaG\rycB|TD^pIGHOAG]}HR UMEF^bxCL||RQxAg\~h@OEaG\rArSpQWOb`eQuyvXtev[nIbOqQC^||RQxAg\~h@OEaG\rArSpQ WObdeQuyvXtev[nIbOqQC^||RQxAg\~h@OIuvYrPfO|@u[legLdAbQiqF[eQVOb@cH`LD[oMWYd ucHpHbO|@eL~tRL`tRL||BTrxcB|@eL~tBLndCNy`SNxdCNy`SNy@RKpxRNxdCNy`SNxdCNydCO o@eL~h@OPIcOm@cKy\SNwdsMy\SNwdCN`tBLndsMy\SNwdsMy\SNxpsKPIcOJpCTrxSKpxRNvdc MyXSNvdcMy\CHm@cKyXSNvdcMyXSNvdsM||BTrxcB|@eL~tBLndSMyTSNudSMyTSNv@RKpxRNud SMyTSNudSMyXCOo@eL~h@OPIcOm@cKyPSNtdCMyPSNtdSM`tBLndCMyPSNtdCMyPSNupsKPIcOJ pCTrxSKpxRNsdsLyLSNsdsLyPCHm@cKyLSNsdsLyLSNsdCM||BTrxcB|@eL~tBLndcLyHSNrdcL yHSNs@RKpxRNrdcLyHSNrdcLyLCOo@eL~h@OPIcOm@cKyDSNqdSLyDSNqdcL`tBLndSLyDSNqdS LyDSNrpsKPIcOJpCTrxSKpxRNpdCLy@SNpdCLyDCHm@cKy@SNpdCLy@SNpdSL||BTrxcB|@eL~t BLn`SNxdCNy`SNxdSN`tBLn`SNxdCNy`SNxdSN||BTrxcB|@eL~tBLn`CNx`CNx`CNx`CNy@RKp xBNx`CNx`CNx`CNxdCOo@eL~h@OPIcOm@cKx\CNw`sMx\CNw`CN`tBLn`sMx\CNw`sMx\CNxpsK PIcOJpCTrxSKpxBNv`cMxXCNv`cMx\CHm@cKxXCNv`cMxXCNv`sM||BTrxcB|@eL~tBLn`SMxTC Nu`SMxTCNv@RKpxBNu`SMxTCNu`SMxXCOo@eL~h@OPIcOm@cKxPCNt`CMxPCNt`SM`tBLn`CMxP CNt`CMxPCNupsKPIcOJpCTrxSKpxBNs`sLxLCNs`sLxPCHm@cKxLCNs`sLxLCNs`CM||BTrxcB| @eL~tBLn`cLxHCNr`cLxHCNs@RKpxBNr`cLxHCNr`cLxLCOo@eL~h@OPIcOm@cKxDCNq`SLxDCN q`cL`tBLn`SLxDCNq`SLxDCNrpsKPIcOJpCTrxSKpxBNp`CLx@CNp`CLxDCHm@cKx@CNp`CLx@C Np`SL||BTrxcB|@eL~tBLn\SNwdsMy\SNwdCN`tBLn\SNwdsMy\SNwdCN||BTrxcB|@eL~tBLn\ CNw`sMx\CNw`sMy@RKpxrMx\CNw`sMx\CNwdCOo@eL~h@OPIcOm@cKw\sMw\sMw\sMw\CN`tBLn \sMw\sMw\sMw\sMxpsKPIcOJpCTrxSKpxrMv\cMwXsMv\cMw\CHm@cKwXsMv\cMwXsMv\sM||BT rxcB|@eL~tBLn\SMwTsMu\SMwTsMv@RKpxrMu\SMwTsMu\SMwXCOo@eL~h@OPIcOm@cKwPsMt\C MwPsMt\SM`tBLn\CMwPsMt\CMwPsMupsKPIcOJpCTrxSKpxrMs\sLwLsMs\sLwPCHm@cKwLsMs\ sLwLsMs\CM||BTrxcB|@eL~tBLn\cLwHsMr\cLwHsMs@RKpxrMr\cLwHsMr\cLwLCOo@eL~h@OP IcOm@cKwDsMq\SLwDsMq\cL`tBLn\SLwDsMq\SLwDsMrpsKPIcOJpCTrxSKpxrMp\CLw@sMp\CL wDCHm@cKw@sMp\CLw@sMp\SL||BTrxcB|@eL~tBLnXSNvdcMyXSNvdsM`tBLnXSNvdcMyXSNvds M||BTrxcB|@eL~tBLnXCNv`cMxXCNv`cMy@RKpxbMxXCNv`cMxXCNvdCOo@eL~h@OPIcOm@cKv\ cMwXsMv\cMwXCN`tBLnXsMv\cMwXsMv\cMxpsKPIcOJpCTrxSKpxbMvXcMvXcMvXcMv\CHm@cKv XcMvXcMvXcMvXsM||BTrxcB|@eL~tBLnXSMvTcMuXSMvTcMv@RKpxbMuXSMvTcMuXSMvXCOo@eL ~h@OPIcOm@cKvPcMtXCMvPcMtXSM`tBLnXCMvPcMtXCMvPcMupsKPIcOJpCTrxSKpxbMsXsLvLc MsXsLvPCHm@cKvLcMsXsLvLcMsXCM||BTrxcB|@eL~tBLnXcLvHcMrXcLvHcMs@RKpxbMrXcLvH cMrXcLvLCOo@eL~h@OPIcOm@cKvDcMqXSLvDcMqXcL`tBLnXSLvDcMqXSLvDcMrpsKPIcOJpCTr xSKpxbMpXCLv@cMpXCLvDCHm@cKv@cMpXCLv@cMpXSL||BTrxcB|@eL~tBLnTSNudSMyTSNudcM `tBLnTSNudSMyTSNudcM||BTrxcB|@eL~tBLnTCNu`SMxTCNu`SMy@RKpxRMxTCNu`SMxTCNudC Oo@eL~h@OPIcOm@cKu\SMwTsMu\SMwTCN`tBLnTsMu\SMwTsMu\SMxpsKPIcOJpCTrxSKpxRMvT cMuXSMvTcMu\CHm@cKuXSMvTcMuXSMvTsM||BTrxcB|@eL~tBLnTSMuTSMuTSMuTSMv@RKpxRMu TSMuTSMuTSMuXCOo@eL~h@OPIcOm@cKuPSMtTCMuPSMtTSM`tBLnTCMuPSMtTCMuPSMupsKPIcO JpCTrxSKpxRMsTsLuLSMsTsLuPCHm@cKuLSMsTsLuLSMsTCM||BTrxcB|@eL~tBLnTcLuHSMrTc LuHSMs@RKpxRMrTcLuHSMrTcLuLCOo@eL~h@OPIcOm@cKuDSMqTSLuDSMqTcL`tBLnTSLuDSMqT SLuDSMrpsKPIcOJpCTrxSKpxRMpTCLu@SMpTCLuDCHm@cKu@SMpTCLu@SMpTSL||BTrxcB|@eL~ tBLnPSNtdCMyPSNtdCMy@RKpxBMyPSNtdCMyPSNtdCOo@eL~h@OPIcOm@cKt`CMxPCNt`CMxPCN `tBLnPCNt`CMxPCNt`CMxpsKPIcOJpCTrxSKpxBMwPsMt\CMwPsMt\CHm@cKt\CMwPsMt\CMwPs M||BTrxcB|@eL~tBLnPcMtXCMvPcMtXCMv@RKpxBMvPcMtXCMvPcMtXCOo@eL~h@OPIcOm@cKtT CMuPSMtTCMuPSM`tBLnPSMtTCMuPSMtTCMupsKPIcOJpCTrxSKpxBMtPCMtPCMtPCMtPCHm@cKt PCMtPCMtPCMtPCM||BTrxcB|@eL~tBLnPsLtLCMsPsLtLCMs@RKpxBMsPsLtLCMsPsLtLCOo@eL ~h@OPIcOm@cKtHCMrPcLtHCMrPcL`tBLnPcLtHCMrPcLtHCMrpsKPIcOJpCTrxSKpxBMqPSLtDC MqPSLtDCHm@cKtDCMqPSLtDCMqPSL||BTrxcB|@eL~tBLnPCLt@CMpPCLt@CM`tBLnPCLt@CMpP CLt@CM||BTrxcB|@eL~tBLnLSNsdsLyLSNsdsLy@RKpxrLyLSNsdsLyLSNsdCOo@eL~h@OPIcOm @cKs`sLxLCNs`sLxLCN`tBLnLCNs`sLxLCNs`sLxpsKPIcOJpCTrxSKpxrLwLsMs\sLwLsMs\CH m@cKs\sLwLsMs\sLwLsM||BTrxcB|@eL~tBLnLcMsXsLvLcMsXsLv@RKpxrLvLcMsXsLvLcMsXC Oo@eL~h@OPIcOm@cKsTsLuLSMsTsLuLSM`tBLnLSMsTsLuLSMsTsLupsKPIcOJpCTrxSKpxrLtL CMsPsLtLCMsPCHm@cKsPsLtLCMsPsLtLCM||BTrxcB|@eL~tBLnLsLsLsLsLsLsLsLs@RKpxrLs LsLsLsLsLsLsLCOo@eL~h@OPIcOm@cKsHsLrLcLsHsLrLcL`tBLnLcLsHsLrLcLsHsLrpsKPIcO JpCTrxSKpxrLqLSLsDsLqLSLsDCHm@cKsDsLqLSLsDsLqLSL||BTrxcB|@eL~tBLnLCLs@sLpLC Ls@sL`tBLnLCLs@sLpLCLs@sL||BTrxcB|@eL~tBLnHSNrdcLyHSNrdcLy@RKpxbLyHSNrdcLyH SNrdCOo@eL~h@OPIcOm@cKr`cLxHCNr`cLxHCN`tBLnHCNr`cLxHCNr`cLxpsKPIcOJpCTrxSKp xbLwHsMr\cLwHsMr\CHm@cKr\cLwHsMr\cLwHsM||BTrxcB|@eL~tBLnHcMrXcLvHcMrXcLv@RK pxbLvHcMrXcLvHcMrXCOo@eL~h@OPIcOm@cKrTcLuHSMrTcLuHSM`tBLnHSMrTcLuHSMrTcLups KPIcOJpCTrxSKpxbLtHCMrPcLtHCMrPCHm@cKrPcLtHCMrPcLtHCM||BTrxcB|@eL~tBLnHsLrL cLsHsLrLcLs@RKpxbLsHsLrLcLsHsLrLCOo@eL~h@OPIcOm@cKrHcLrHcLrHcLrHcL`tBLnHcLr HcLrHcLrHcLrpsKPIcOJpCTrxSKpxbLqHSLrDcLqHSLrDCHm@cKrDcLqHSLrDcLqHSL||BTrxcB |@eL~tBLnHCLr@cLpHCLr@cL`tBLnHCLr@cLpHCLr@cL||BTrxcB|@eL~tBLnDSNqdSLyDSNqdS Ly@RKpxRLyDSNqdSLyDSNqdCOo@eL~h@OPIcOm@cKq`SLxDCNq`SLxDCN`tBLnDCNq`SLxDCNq` SLxpsKPIcOJpCTrxSKpxRLwDsMq\SLwDsMq\CHm@cKq\SLwDsMq\SLwDsM||BTrxcB|@eL~tBLn DcMqXSLvDcMqXSLv@RKpxRLvDcMqXSLvDcMqXCOo@eL~h@OPIcOm@cKqTSLuDSMqTSLuDSM`tBL nDSMqTSLuDSMqTSLupsKPIcOJpCTrxSKpxRLtDCMqPSLtDCMqPCHm@cKqPSLtDCMqPSLtDCM||B TrxcB|@eL~tBLnDsLqLSLsDsLqLSLs@RKpxRLsDsLqLSLsDsLqLCOo@eL~h@OPIcOm@cKqHSLrD cLqHSLrDcL`tBLnDcLqHSLrDcLqHSLrpsKPIcOJpCTrxSKpxRLqDSLqDSLqDSLqDCHm@cKqDSLq DSLqDSLqDSL||BTrxcB|@eL~tBLnDCLq@SLpDCLq@SL`tBLnDCLq@SLpDCLq@SL||BTrxcB|@eL ~tBLn@SNpdCLy@SNpdCLyDCHm@cKpdCLy@SNpdCLy@SNqpsKPIcOJpCTrxSKpxBLx@CNp`CLx@C Np`SL`tBLn@CNp`CLx@CNp`CLxDCOo@eL~h@OPIcOm@cKp\CLw@sMp\CLw@sMq@RKpxBLw@sMp\ CLw@sMp\SL||BTrxcB|@eL~tBLn@cMpXCLv@cMpXCLvDCHm@cKpXCLv@cMpXCLv@cMqpsKPIcOJ pCTrxSKpxBLu@SMpTCLu@SMpTSL`tBLn@SMpTCLu@SMpTCLuDCOo@eL~h@OPIcOm@cKpPCLt@CM pPCLt@CM`tBLn@CMpPCLt@CMpPCLtpsKPIcOJpCTrxSKpxBLs@sLpLCLs@sLpLCHm@cKpLCLs@s LpLCLs@sL||BTrxcB|@eL~tBLn@cLpHCLr@cLpHCLr@RKpxBLr@cLpHCLr@cLpHCOo@eL~h@OPI cOm@cKpDCLq@SLpDCLq@SL`tBLn@SLpDCLq@SLpDCLqpsKPIcOJpCTrxCL`@COo@eL~h@Oo@u[l egLdycB||RRm]fLdycB||rSbifLdycB||dXjICY`PU^pUVObLT]rYWYrPfH`dFY}HB\l}F]OIfZ eMF]qHcH`Lt[nMWZsQWYnQWObDcH`XUZsefXlUVObDcH`@u[iyF]SQW^lUVObXTZlqVYdMTZrMF [eMgH`pTZnUvUiQF]hucHpxRMb@BSiyVYSQW^lUVObLu[leFYb@RUMUv\hucHq@CLb@BSiyVYC} F[oIGUyAWY}HbQlEF]b@BSiyVYC}F[oIWObLBLp@CLp@cH`pTZnUv\Vev\iIF[eucHqHBHDev\c }f[tef[ueF]yMUYaIwXhucHqHbO|TD^pIGHOAG]}HRUNEV[eIbOqQC^||RQxAg\~h@OEaG\rArS pQWObTUSiyfH~DCOoTD^pIgOJpSQxAg\`|D\tucHUuTXxIbOrpsKEaG\rycB|TD^pIGHOAG]}HB VFUg[cQWZoyfH~DGMxqsKEaG\rycB|TD^pIGHOAG]}HRVFUg[cQWZoyfH~DGMxARK`HCOoTD^pI gOJpSRm]fLdyCOP}F[yICY`XTZlqVYducHpHBHCqv[sUFY}HBLbxCOPIcOq@RKqpsKPIcOJpCTr xSLn@SLpDCLq@SLpDCHm@cKy`SNxdCNy`SNxdSN||BTrxcB|@eL~DcKpHCLr@cLpHCLr@RKpxRN wdsMy\SNwdsMy`COo@eL~h@OPIcOqxBLs@sLpLCLs@sL`tBLndcMyXSNvdcMyXSNwpsKPIcOJpC TrxSLn@CMpPCLt@CMpPCHm@cKyTSNudSMyTSNudcM||BTrxcB|@eL~DcKpTCLu@SMpTCLuDCHm@ cKyPSNtdCMyPSNtdSM||BTrxcB|@eL~DcKpXCLv@cMpXCLvDCHm@cKyLSNsdsLyLSNsdCM||BTr xcB|@eL~DcKp\CLw@sMp\CLwDCHm@cKyHSNrdcLyHSNrdsL||BTrxcB|@eL~DcKp`CLx@CNp`CL xDCHm@cKyDSNqdSLyDSNqdcL||BTrxcB|@eL~DcKpdCLy@SNpdCLyDCHm@cKy@SNpdCLy@SNpdS L||BTrxcB|@eL~DcKq@SLpDCLq@SLpDCHm@cKxdCNy`SNxdCNydCOo@eL~h@OPIcOqxRLqDSLqD SLqDSLq@RKpxBNx`CNx`CNx`CNxdCOo@eL~h@OPIcOqxRLrDcLqHSLrDcLq@RKpxBNw`sMx\CNw `sMx`COo@eL~h@OPIcOqxRLsDsLqLSLsDsLq@RKpxBNv`cMxXCNv`cMx\COo@eL~h@OPIcOqxRL tDCMqPSLtDCMq@RKpxBNu`SMxTCNu`SMxXCOo@eL~h@OPIcOqxRLuDSMqTSLuDSMr@RKpxBNt`C MxPCNt`CMxTCOo@eL~h@OPIcOqxRLvDcMqXSLvDcMr@RKpxBNs`sLxLCNs`sLxPCOo@eL~h@OPI cOqxRLwDsMq\SLwDsMr@RKpxBNr`cLxHCNr`cLxLCOo@eL~h@OPIcOqxRLxDCNq`SLxDCNr@RKp xBNq`SLxDCNq`SLxHCOo@eL~h@OPIcOqxRLyDSNqdSLyDSNr@RKpxBNp`CLx@CNp`CLxDCOo@eL ~h@OPIcOqxbLpHCLr@cLpHCLr@RKpxrMy\SNwdsMy\SNxpsKPIcOJpCTrxSLnHSLrDcLqHSLrDc L`tBLn\CNw`sMx\CNw`sMypsKPIcOJpCTrxSLnHcLrHcLrHcLrHcL`tBLn\sMw\sMw\sMw\sMxp sKPIcOJpCTrxSLnHsLrLcLsHsLrLcL`tBLn\cMwXsMv\cMwXsMwpsKPIcOJpCTrxSLnHCMrPcLt HCMrPcL`tBLn\SMwTsMu\SMwTsMvpsKPIcOJpCTrxSLnHSMrTcLuHSMrTsL`tBLn\CMwPsMt\CM wPsMupsKPIcOJpCTrxSLnHcMrXcLvHcMrXsL`tBLn\sLwLsMs\sLwLsMtpsKPIcOJpCTrxSLnHs Mr\cLwHsMr\sL`tBLn\cLwHsMr\cLwHsMspsKPIcOJpCTrxSLnHCNr`cLxHCNr`sL`tBLn\SLwD sMq\SLwDsMrpsKPIcOJpCTrxSLnHSNrdcLyHSNrdsL`tBLn\CLw@sMp\CLw@sMqpsKPIcOJpCTr xSLnLCLs@sLpLCLs@sL`tBLnXSNvdcMyXSNvdsM||BTrxcB|@eL~DcKsDsLqLSLsDsLqLCHm@cK v`cMxXCNv`cMxXSN||BTrxcB|@eL~DcKsHsLrLcLsHsLrLCHm@cKv\cMwXsMv\cMwXCN||BTrxc B|@eL~DcKsLsLsLsLsLsLsLCHm@cKvXcMvXcMvXcMvXsM||BTrxcB|@eL~DcKsPsLtLCMsPsLtL CHm@cKvTcMuXSMvTcMuXcM||BTrxcB|@eL~DcKsTsLuLSMsTsLuPCHm@cKvPcMtXCMvPcMtXSM| |BTrxcB|@eL~DcKsXsLvLcMsXsLvPCHm@cKvLcMsXsLvLcMsXCM||BTrxcB|@eL~DcKs\sLwLsM s\sLwPCHm@cKvHcMrXcLvHcMrXsL||BTrxcB|@eL~DcKs`sLxLCNs`sLxPCHm@cKvDcMqXSLvDc MqXcL||BTrxcB|@eL~DcKsdsLyLSNsdsLyPCHm@cKv@cMpXCLv@cMpXSL||BTrxcB|@eL~DcKt@ CMpPCLt@CMpPCHm@cKudSMyTSNudSMyXCOo@eL~h@OPIcOqxBMqPSLtDCMqPSLt@RKpxRMxTCNu `SMxTCNudCOo@eL~h@OPIcOqxBMrPcLtHCMrPcLt@RKpxRMwTsMu\SMwTsMu`COo@eL~h@OPIcO qxBMsPsLtLCMsPsLt@RKpxRMvTcMuXSMvTcMu\COo@eL~h@OPIcOqxBMtPCMtPCMtPCMt@RKpxR MuTSMuTSMuTSMuXCOo@eL~h@OPIcOqxBMuPSMtTCMuPSMu@RKpxRMtTCMuPSMtTCMuTCOo@eL~h @OPIcOqxBMvPcMtXCMvPcMu@RKpxRMsTsLuLSMsTsLuPCOo@eL~h@OPIcOqxBMwPsMt\CMwPsMu @RKpxRMrTcLuHSMrTcLuLCOo@eL~h@OPIcOqxBMxPCNt`CMxPCNu@RKpxRMqTSLuDSMqTSLuHCO o@eL~h@OPIcOqxBMyPSNtdCMyPSNu@RKpxRMpTCLu@SMpTCLuDCOo@eL~h@OPIcOqxRMpTCLu@S MpTCLu@RKpxBMyPSNtdCMyPSNtdCOo@eL~h@OPIcOqxRMqTSLuDSMqTSLu@RKpxBMxPCNt`CMxP CNt`COo@eL~h@OPIcOqxRMrTcLuHSMrTcLu@RKpxBMwPsMt\CMwPsMt\COo@eL~h@OPIcOqxRMs TsLuLSMsTsLu@RKpxBMvPcMtXCMvPcMtXCOo@eL~h@OPIcOqxRMtTCMuPSMtTCMu@RKpxBMuPSM tTCMuPSMtTCOo@eL~h@OPIcOqxRMuTSMuTSMuTSMv@RKpxBMtPCMtPCMtPCMtPCOo@eL~h@OPIc OqxRMvTcMuXSMvTcMv@RKpxBMsPsLtLCMsPsLtLCOo@eL~h@OPIcOqxRMwTsMu\SMwTsMv@RKpx BMrPcLtHCMrPcLtHCOo@eL~h@OPIcOqxRMxTCNu`SMxTCNv@RKpxBMqPSLtDCMqPSLtDCOo@eL~ h@OPIcOqxRMyTSNudSMyTSNv@RKpxBMpPCLt@CMpPCLtpsKPIcOJpCTrxSLnXCLv@cMpXCLv@cM `tBLnLSNsdsLyLSNsdsLypsKPIcOJpCTrxSLnXSLvDcMqXSLvDcM`tBLnLCNs`sLxLCNs`sLxps KPIcOJpCTrxSLnXcLvHcMrXcLvHcM`tBLnLsMs\sLwLsMs\sLwpsKPIcOJpCTrxSLnXsLvLcMsX sLvLcM`tBLnLcMsXsLvLcMsXsLvpsKPIcOJpCTrxSLnXCMvPcMtXCMvPcM`tBLnLSMsTsLuLSMs TsLupsKPIcOJpCTrxSLnXSMvTcMuXSMvTsM`tBLnLCMsPsLtLCMsPsLtpsKPIcOJpCTrxSLnXcM vXcMvXcMvXsM`tBLnLsLsLsLsLsLsLsLspsKPIcOJpCTrxSLnXsMv\cMwXsMv\sM`tBLnLcLsHs LrLcLsHsLrpsKPIcOJpCTrxSLnXCNv`cMxXCNv`sM`tBLnLSLsDsLqLSLsDsLqpsKPIcOJpCTrx SLnXSNvdcMyXSNvdsM`tBLnLCLs@sLpLCLs@sL||BTrxcB|@eL~DcKw@sMp\CLw@sMp\CHm@cKr dcLyHSNrdcLyHSN||BTrxcB|@eL~DcKwDsMq\SLwDsMq\CHm@cKr`cLxHCNr`cLxHCN||BTrxcB |@eL~DcKwHsMr\cLwHsMr\CHm@cKr\cLwHsMr\cLwHsM||BTrxcB|@eL~DcKwLsMs\sLwLsMs\C Hm@cKrXcLvHcMrXcLvHcM||BTrxcB|@eL~DcKwPsMt\CMwPsMt\CHm@cKrTcLuHSMrTcLuHSM|| BTrxcB|@eL~DcKwTsMu\SMwTsMu`CHm@cKrPcLtHCMrPcLtHCM||BTrxcB|@eL~DcKwXsMv\cMw XsMv`CHm@cKrLcLsHsLrLcLsHsL||BTrxcB|@eL~DcKw\sMw\sMw\sMw`CHm@cKrHcLrHcLrHcL rHcL||BTrxcB|@eL~DcKw`sMx\CNw`sMx`CHm@cKrDcLqHSLrDcLqHSL||BTrxcB|@eL~DcKwds My\SNwdsMy`CHm@cKr@cLpHCLr@cLpHCOo@eL~h@OPIcOqxBNp`CLx@CNp`CLx@RKpxRLyDSNqd SLyDSNqdCOo@eL~h@OPIcOqxBNq`SLxDCNq`SLx@RKpxRLxDCNq`SLxDCNq`COo@eL~h@OPIcOq xBNr`cLxHCNr`cLx@RKpxRLwDsMq\SLwDsMq\COo@eL~h@OPIcOqxBNs`sLxLCNs`sLx@RKpxRL vDcMqXSLvDcMqXCOo@eL~h@OPIcOqxBNt`CMxPCNt`CMx@RKpxRLuDSMqTSLuDSMqTCOo@eL~h@ OPIcOqxBNu`SMxTCNu`SMy@RKpxRLtDCMqPSLtDCMqPCOo@eL~h@OPIcOqxBNv`cMxXCNv`cMy@ RKpxRLsDsLqLSLsDsLqLCOo@eL~h@OPIcOqxBNw`sMx\CNw`sMy@RKpxRLrDcLqHSLrDcLqHCOo @eL~h@OPIcOqxBNx`CNx`CNx`CNy@RKpxRLqDSLqDSLqDSLqDCOo@eL~h@OPIcOqxBNy`SNxdCN y`SNy@RKpxRLpDCLq@SLpDCLqpsKPIcOJpCTrxSLndCLy@SNpdCLy@SN`tBLn@SNpdCLy@SNpdC LyDCOo@eL~h@OPIcOqxRNqdSLyDSNqdSLy@RKpxBLx@CNp`CLx@CNp`SL||BTrxcB|@eL~DcKyH SNrdcLyHSNrdCHm@cKp\CLw@sMp\CLw@sMqpsKPIcOJpCTrxSLndsLyLSNsdsLyLSN`tBLn@cMp XCLv@cMpXCLvDCOo@eL~h@OPIcOqxRNtdCMyPSNtdCMy@RKpxBLu@SMpTCLu@SMpTSL||BTrxcB |@eL~DcKyTSNudSMyTSNv@RKpxBLt@CMpPCLt@CMpPCOo@eL~h@OPIcOqxRNvdcMyXSNvdsM`tB Ln@sLpLCLs@sLpLCLspsKPIcOJpCTrxSLndsMy\SNwdsMy`CHm@cKpHCLr@cLpHCLr@cL||BTrx cB|@eL~DcKy`SNxdCNy`SNy@RKpxBLq@SLpDCLq@SLpDCOo@eL~h@OPIcOr@BL||BTrxcB||BTo qV^rPfOJpsKIuvYrPfOJpsKOIfZrPfOJpsSbifLdABUyAWY}HrPuIg]eICYb@RZducHpqv[t}dX jUvXtEsLb@rPoyv\iMG]eyF]}HRLb@bUiMWZbqVY}HRLb@BToef[tME]yqVY}HbQiqF[eQvPiIw XlUv\b@BSiyVYWeFYtaVOb@cKuHBHLef[eME]yqVY}HrToqVZdIBHUuTYsaVObDCLpHBHLef[eM t[l}f\TeG\eucHFqVXtIBHLef[eMt[l}f\}HrHp@CLp@CLb@BSiyVYsYUZsefXlUVObDcH`PTZs Mv[nQWZnUWZtewTeEf\caVObDcH~pSQxAg\`|D\tucHUyTXmUfH~DGMxqsKEaG\rycB|TD^pIGH OAG]}HRUMef[bxcL||RQxAg\~h@OEaG\rArSpQWObTUSaagH~LCOoTD^pIgOJpSQxAg\`|D\tuc HXYT]nMF]i}f[bxS\t`GOoTD^pIgOJpSQxAg\`|D\tucHYYT]nMF]i}f[bxS\t`GHm@bL||RQxA g\~h@OIuvYrPfO|@u[legLdAbQiqF[eQVOb@cH`LD[oMWYducHpHbO|@eL~HCHppsKPIcOJpCTr xcLn@SLpDCLq@SLpDCHpxBLq@SLpDCLq@SLpDCOo@eL~h@OPIcOrxBLr@cLpHCLr@cL`@cKpHCL r@cLpHCLr@cL||BTrxcB|@eL~HcKpLCLs@sLpLCLs@BLn@sLpLCLs@sLpLCLspsKPIcOJpCTrxc Ln@CMpPCLt@CMpPCHpxBLt@CMpPCLt@CMpPCOo@eL~h@OPIcOrxBLu@SMpTCLu@SMq@BLn@SMpT CLu@SMpTCLuDCOo@eL~h@OPIcOrxBLv@cMpXCLv@cMq@BLn@cMpXCLv@cMpXCLvDCOo@eL~h@OP IcOrxBLw@sMp\CLw@sMq@BLn@sMp\CLw@sMp\CLwDCOo@eL~h@OPIcOrxBLx@CNp`CLx@CNq@BL n@CNp`CLx@CNp`CLxDCOo@eL~h@OPIcOrxBLy@SNpdCLy@SNq@BLn@SNpdCLy@SNpdCLyDCOo@e L~h@OPIcOrxRLpDCLq@SLpDCLq@BLnDCLq@SLpDCLq@SL||BTrxcB|@eL~HcKqDSLqDSLqDSLqD CHpxRLqDSLqDSLqDSLqDCOo@eL~h@OPIcOrxRLrDcLqHSLrDcLq@BLnDcLqHSLrDcLqHSLrpsKP IcOJpCTrxcLnDsLqLSLsDsLqLSL`@cKqLSLsDsLqLSLsDsL||BTrxcB|@eL~HcKqPSLtDCMqPSL tDCHpxRLtDCMqPSLtDCMqPCOo@eL~h@OPIcOrxRLuDSMqTSLuDSMr@BLnDSMqTSLuDSMqTSLups KPIcOJpCTrxcLnDcMqXSLvDcMqXcL`@cKqXSLvDcMqXSLvDcM||BTrxcB|@eL~HcKq\SLwDsMq\ SLwHCHpxRLwDsMq\SLwDsMq\COo@eL~h@OPIcOrxRLxDCNq`SLxDCNr@BLnDCNq`SLxDCNq`SLx psKPIcOJpCTrxcLnDSNqdSLyDSNqdcL`@cKqdSLyDSNqdSLyDSN||BTrxcB|@eL~HcKr@cLpHCL r@cLpHCHpxbLpHCLr@cLpHCLrpsKPIcOJpCTrxcLnHSLrDcLqHSLrDcL`@cKrDcLqHSLrDcLqHS L||BTrxcB|@eL~HcKrHcLrHcLrHcLrHCHpxbLrHcLrHcLrHcLrHCOo@eL~h@OPIcOrxbLsHsLrL cLsHsLr@BLnHsLrLcLsHsLrLcLspsKPIcOJpCTrxcLnHCMrPcLtHCMrPcL`@cKrPcLtHCMrPcLt HCM||BTrxcB|@eL~HcKrTcLuHSMrTcLuLCHpxbLuHSMrTcLuHSMrTCOo@eL~h@OPIcOrxbLvHcM rXcLvHcMs@BLnHcMrXcLvHcMrXcLvpsKPIcOJpCTrxcLnHsMr\cLwHsMr\sL`@cKr\cLwHsMr\c LwHsM||BTrxcB|@eL~HcKr`cLxHCNr`cLxLCHpxbLxHCNr`cLxHCNr`COo@eL~h@OPIcOrxbLyH SNrdcLyHSNs@BLnHSNrdcLyHSNrdcLypsKPIcOJpCTrxcLnLCLs@sLpLCLs@sL`@cKs@sLpLCLs @sLpLCOo@eL~h@OPIcOrxrLqLSLsDsLqLSLs@BLnLSLsDsLqLSLsDsLqpsKPIcOJpCTrxcLnLcL sHsLrLcLsHsL`@cKsHsLrLcLsHsLrLcL||BTrxcB|@eL~HcKsLsLsLsLsLsLsLCHpxrLsLsLsLs LsLsLsLCOo@eL~h@OPIcOrxrLtLCMsPsLtLCMs@BLnLCMsPsLtLCMsPsLtpsKPIcOJpCTrxcLnL SMsTsLuLSMsTCM`@cKsTsLuLSMsTsLuLSM||BTrxcB|@eL~HcKsXsLvLcMsXsLvPCHpxrLvLcMs XsLvLcMsXCOo@eL~h@OPIcOrxrLwLsMs\sLwLsMt@BLnLsMs\sLwLsMs\sLwpsKPIcOJpCTrxcL nLCNs`sLxLCNs`CM`@cKs`sLxLCNs`sLxLCN||BTrxcB|@eL~HcKsdsLyLSNsdsLyPCHpxrLyLS NsdsLyLSNsdCOo@eL~h@OPIcOrxBMpPCLt@CMpPCLt@BLnPCLt@CMpPCLt@CM||BTrxcB|@eL~H cKtDCMqPSLtDCMqPCHpxBMqPSLtDCMqPSLtDCOo@eL~h@OPIcOrxBMrPcLtHCMrPcLt@BLnPcLt HCMrPcLtHCMrpsKPIcOJpCTrxcLnPsLtLCMsPsLtLCM`@cKtLCMsPsLtLCMsPsL||BTrxcB|@eL ~HcKtPCMtPCMtPCMtPCHpxBMtPCMtPCMtPCMtPCOo@eL~h@OPIcOrxBMuPSMtTCMuPSMu@BLnPS MtTCMuPSMtTCMupsKPIcOJpCTrxcLnPcMtXCMvPcMtXSM`@cKtXCMvPcMtXCMvPcM||BTrxcB|@ eL~HcKt\CMwPsMt\CMwTCHpxBMwPsMt\CMwPsMt\COo@eL~h@OPIcOrxBMxPCNt`CMxPCNu@BLn PCNt`CMxPCNt`CMxpsKPIcOJpCTrxcLnPSNtdCMyPSNtdSM`@cKtdCMyPSNtdCMyPSN||BTrxcB |@eL~HcKu@SMpTCLu@SMpTCHpxRMpTCLu@SMpTCLuDCOo@eL~h@OPIcOrxRMqTSLuDSMqTSLu@B LnTSLuDSMqTSLuDSMrpsKPIcOJpCTrxcLnTcLuHSMrTcLuHSM`@cKuHSMrTcLuHSMrTsL||BTrx cB|@eL~HcKuLSMsTsLuLSMsTCHpxRMsTsLuLSMsTsLuPCOo@eL~h@OPIcOrxRMtTCMuPSMtTCMu @BLnTCMuPSMtTCMuPSMupsKPIcOJpCTrxcLnTSMuTSMuTSMuTcM`@cKuTSMuTSMuTSMuTcM||BT rxcB|@eL~HcKuXSMvTcMuXSMvXCHpxRMvTcMuXSMvTcMu\COo@eL~h@OPIcOrxRMwTsMu\SMwTs Mv@BLnTsMu\SMwTsMu\SMxpsKPIcOJpCTrxcLnTCNu`SMxTCNu`cM`@cKu`SMxTCNu`SMxTSN|| BTrxcB|@eL~HcKudSMyTSNudSMyXCHpxRMyTSNudSMyTSNvpsKPIcOJpCTrxcLnXCLv@cMpXCLv @cM`@cKv@cMpXCLv@cMpXSL||BTrxcB|@eL~HcKvDcMqXSLvDcMqXCHpxbMqXSLvDcMqXSLvHCO o@eL~h@OPIcOrxbMrXcLvHcMrXcLv@BLnXcLvHcMrXcLvHcMspsKPIcOJpCTrxcLnXsLvLcMsXs LvLcM`@cKvLcMsXsLvLcMsXCM||BTrxcB|@eL~HcKvPcMtXCMvPcMtXCHpxbMtXCMvPcMtXCMvT COo@eL~h@OPIcOrxbMuXSMvTcMuXSMw@BLnXSMvTcMuXSMvTcMvpsKPIcOJpCTrxcLnXcMvXcMv XcMvXsM`@cKvXcMvXcMvXcMvXsM||BTrxcB|@eL~HcKv\cMwXsMv\cMw\CHpxbMwXsMv\cMwXsM v`COo@eL~h@OPIcOrxbMxXCNv`cMxXCNw@BLnXCNv`cMxXCNv`cMypsKPIcOJpCTrxcLnXSNvdc MyXSNvdsM`@cKvdcMyXSNvdcMy\COo@eL~h@OPIcOrxrMp\CLw@sMp\CLw@BLn\CLw@sMp\CLw@ sMqpsKPIcOJpCTrxcLn\SLwDsMq\SLwDsM`@cKwDsMq\SLwDsMq\cL||BTrxcB|@eL~HcKwHsMr \cLwHsMr\CHpxrMr\cLwHsMr\cLwLCOo@eL~h@OPIcOrxrMs\sLwLsMs\sLw@BLn\sLwLsMs\sL wLsMtpsKPIcOJpCTrxcLn\CMwPsMt\CMwPsM`@cKwPsMt\CMwPsMt\SM||BTrxcB|@eL~HcKwTs Mu\SMwTsMu`CHpxrMu\SMwTsMu\SMwXCOo@eL~h@OPIcOrxrMv\cMwXsMv\cMx@BLn\cMwXsMv\ cMwXsMwpsKPIcOJpCTrxcLn\sMw\sMw\sMw\CN`@cKw\sMw\sMw\sMw\CN||BTrxcB|@eL~HcKw `sMx\CNw`sMx`CHpxrMx\CNw`sMx\CNwdCOo@eL~h@OPIcOrxrMy\SNwdsMy\SNx@BLn\SNwdsM y\SNwdCN||BTrxcB|@eL~HcKx@CNp`CLx@CNp`CHpxBNp`CLx@CNp`CLxDCOo@eL~h@OPIcOrxB Nq`SLxDCNq`SLx@BLn`SLxDCNq`SLxDCNrpsKPIcOJpCTrxcLn`cLxHCNr`cLxHCN`@cKxHCNr` cLxHCNr`sL||BTrxcB|@eL~HcKxLCNs`sLxLCNs`CHpxBNs`sLxLCNs`sLxPCOo@eL~h@OPIcOr xBNt`CMxPCNt`CMx@BLn`CMxPCNt`CMxPCNupsKPIcOJpCTrxcLn`SMxTCNu`SMxTSN`@cKxTCN u`SMxTCNu`cM||BTrxcB|@eL~HcKxXCNv`cMxXCNvdCHpxBNv`cMxXCNv`cMx\COo@eL~h@OPIc OrxBNw`sMx\CNw`sMy@BLn`sMx\CNw`sMx\CNxpsKPIcOJpCTrxcLn`CNx`CNx`CNx`SN`@cKx` CNx`CNx`CNx`SN||BTrxcB|@eL~HcKxdCNy`SNxdCNydCHpxBNy`SNxdCNy`SNypsKPIcOJpCTr xcLndCLy@SNpdCLy@SN`@cKy@SNpdCLy@SNpdSL||BTrxcB|@eL~HcKyDSNqdSLyDSNqdCHpxRN qdSLyDSNqdSLyHCOo@eL~h@OPIcOrxRNrdcLyHSNrdcLy@BLndcLyHSNrdcLyHSNspsKPIcOJpC TrxcLndsLyLSNsdsLyLSN`@cKyLSNsdsLyLSNsdCM||BTrxcB|@eL~HcKyPSNtdCMyPSNtdCHpx RNtdCMyPSNtdCMyTCOo@eL~h@OPIcOrxRNudSMyTSNudcM`@cKyTSNudSMyTSNudcM||BTrxcB| @eL~HcKyXSNvdcMyXSNw@BLndcMyXSNvdcMyXSNwpsKPIcOJpCTrxcLndsMy\SNwdsMy`CHpxRN wdsMy\SNwdsMy`COo@eL~h@OPIcOrxRNxdCNy`SNxdSN`@cKy`SNxdCNy`SNxdSN||BTrxcB|@e L~LCHqpsKPIcOJpsKP}F[yICY~h@OodT[gICY~h@Oo|dXjICY~h@OOIfZrPFHTeG\eucHCUg\vU fLdIBHiQVOb@G[oQwSbiVYcQWLtHBHC}f[sev\tUf[tucHqHBHVev\iIF[eucHqHBHP}VZnQwTt eG[eucHFeF[lUFYCef\cqVYsIBHLef[e]UZdQGZ}HBLnTcH`pTZnUvTteG[eucHS}F[iQfH`TUS eMGZ}HRLp@cH`pTZnUvPoqv[rQU^pUVObXD[aQgH`pTZnUvPoqv[rucHc@CLp@CLpHBHLef[eMg UiMWZbqVY}HRLb@BQiMwXoyF]iyV]iQW^SUVXrMFZ}HRLbxCOEaG\rArSpQWObTeSauVYbxS\t` GOoTD^pIgOJpSQxAg\`|D\tucHUuTZnIbOmHCOoTD^pIgOJpSQxAg\`|D\tucHUuTXxIbOmDCOo TD^pIgOJpSQxAg\`|D\tucHXYT]nMF]i}f[bxS\t`GOoTD^pIgOJpSQxAg\`|D\tucHYYT]nMF] i}f[bxS\t`GHk@bL||RQxAg\~h@OIuvYrPfO|@u[legLdAbQiqF[eQVOb@cH`LD[oMWYducHpHb O|@eL~tbL`@COo@eL~h@OPIcOmDcKy`SNxdCNy`SNy@BLn@SLpDCLq@SLpDCLqpsKPIcOJpCTrx SKqxRNwdsMy\SNwdCN`@cKpHCLr@cLpHCLr@cL||BTrxcB|@eL~tRLndcMyXSNvdcMy\CHpxBLs @sLpLCLs@sLpLCOo@eL~h@OPIcOmDcKyTSNudSMyTSNv@BLn@CMpPCLt@CMpPCLtpsKPIcOJpCT rxSKqxRNtdCMyPSNtdCMy@BLn@SMpTCLu@SMpTCLuDCOo@eL~h@OPIcOmDcKyLSNsdsLyLSNsdC HpxBLv@cMpXCLv@cMpXSL||BTrxcB|@eL~tRLndcLyHSNrdcLyHSN`@cKp\CLw@sMp\CLw@sMqp sKPIcOJpCTrxSKqxRNqdSLyDSNqdSLy@BLn@CNp`CLx@CNp`CLxDCOo@eL~h@OPIcOmDcKy@SNp dCLy@SNpdCHpxBLy@SNpdCLy@SNpdSL||BTrxcB|@eL~tRLn`SNxdCNy`SNxdSN`@cKq@SLpDCL q@SLpDCOo@eL~h@OPIcOmDcKx`CNx`CNx`CNxdCHpxRLqDSLqDSLqDSLqDCOo@eL~h@OPIcOmDc Kx\CNw`sMx\CNwdCHpxRLrDcLqHSLrDcLqHCOo@eL~h@OPIcOmDcKxXCNv`cMxXCNvdCHpxRLsD sLqLSLsDsLqLCOo@eL~h@OPIcOmDcKxTCNu`SMxTCNudCHpxRLtDCMqPSLtDCMqPCOo@eL~h@OP IcOmDcKxPCNt`CMxPCNt`CHpxRLuDSMqTSLuDSMqTCOo@eL~h@OPIcOmDcKxLCNs`sLxLCNs`CH pxRLvDcMqXSLvDcMqXCOo@eL~h@OPIcOmDcKxHCNr`cLxHCNr`CHpxRLwDsMq\SLwDsMq\COo@e L~h@OPIcOmDcKxDCNq`SLxDCNq`CHpxRLxDCNq`SLxDCNq`COo@eL~h@OPIcOmDcKx@CNp`CLx@ CNp`CHpxRLyDSNqdSLyDSNqdCOo@eL~h@OPIcOmDcKwdsMy\SNwdsMy`CHpxbLpHCLr@cLpHCLr psKPIcOJpCTrxSKqxrMx\CNw`sMx\CNx@BLnHSLrDcLqHSLrDcLqpsKPIcOJpCTrxSKqxrMw\sM w\sMw\sMx@BLnHcLrHcLrHcLrHcLrpsKPIcOJpCTrxSKqxrMv\cMwXsMv\cMx@BLnHsLrLcLsHs LrLcLspsKPIcOJpCTrxSKqxrMu\SMwTsMu\SMx@BLnHCMrPcLtHCMrPcLtpsKPIcOJpCTrxSKqx rMt\CMwPsMt\CMw@BLnHSMrTcLuHSMrTcLupsKPIcOJpCTrxSKqxrMs\sLwLsMs\sLw@BLnHcMr XcLvHcMrXcLvpsKPIcOJpCTrxSKqxrMr\cLwHsMr\cLw@BLnHsMr\cLwHsMr\cLwpsKPIcOJpCT rxSKqxrMq\SLwDsMq\SLw@BLnHCNr`cLxHCNr`cLxpsKPIcOJpCTrxSKqxrMp\CLw@sMp\CLw@B LnHSNrdcLyHSNrdcLypsKPIcOJpCTrxSKqxbMyXSNvdcMyXSNw@BLnLCLs@sLpLCLs@sL||BTrx cB|@eL~tRLnXCNv`cMxXCNv`sM`@cKsDsLqLSLsDsLqLSL||BTrxcB|@eL~tRLnXsMv\cMwXsMv \sM`@cKsHsLrLcLsHsLrLcL||BTrxcB|@eL~tRLnXcMvXcMvXcMvXsM`@cKsLsLsLsLsLsLsLsL ||BTrxcB|@eL~tRLnXSMvTcMuXSMvTsM`@cKsPsLtLCMsPsLtLCM||BTrxcB|@eL~tRLnXCMvPc MtXCMvPcM`@cKsTsLuLSMsTsLuLSM||BTrxcB|@eL~tRLnXsLvLcMsXsLvLcM`@cKsXsLvLcMsX sLvLcM||BTrxcB|@eL~tRLnXcLvHcMrXcLvHcM`@cKs\sLwLsMs\sLwLsM||BTrxcB|@eL~tRLn XSLvDcMqXSLvDcM`@cKs`sLxLCNs`sLxLCN||BTrxcB|@eL~tRLnXCLv@cMpXCLv@cM`@cKsdsL yLSNsdsLyLSN||BTrxcB|@eL~tRLnTSNudSMyTSNudcM`@cKt@CMpPCLt@CMpPCOo@eL~h@OPIc OmDcKu`SMxTCNu`SMxXCHpxBMqPSLtDCMqPSLtDCOo@eL~h@OPIcOmDcKu\SMwTsMu\SMwXCHpx BMrPcLtHCMrPcLtHCOo@eL~h@OPIcOmDcKuXSMvTcMuXSMvXCHpxBMsPsLtLCMsPsLtLCOo@eL~ h@OPIcOmDcKuTSMuTSMuTSMuXCHpxBMtPCMtPCMtPCMtPCOo@eL~h@OPIcOmDcKuPSMtTCMuPSM tTCHpxBMuPSMtTCMuPSMtTCOo@eL~h@OPIcOmDcKuLSMsTsLuLSMsTCHpxBMvPcMtXCMvPcMtXC Oo@eL~h@OPIcOmDcKuHSMrTcLuHSMrTCHpxBMwPsMt\CMwPsMt\COo@eL~h@OPIcOmDcKuDSMqT SLuDSMqTCHpxBMxPCNt`CMxPCNt`COo@eL~h@OPIcOmDcKu@SMpTCLu@SMpTCHpxBMyPSNtdCMy PSNtdCOo@eL~h@OPIcOmDcKtdCMyPSNtdCMyTCHpxRMpTCLu@SMpTCLuDCOo@eL~h@OPIcOmDcK t`CMxPCNt`CMxTCHpxRMqTSLuDSMqTSLuHCOo@eL~h@OPIcOmDcKt\CMwPsMt\CMwTCHpxRMrTc LuHSMrTcLuLCOo@eL~h@OPIcOmDcKtXCMvPcMtXCMvTCHpxRMsTsLuLSMsTsLuPCOo@eL~h@OPI cOmDcKtTCMuPSMtTCMuTCHpxRMtTCMuPSMtTCMuTCOo@eL~h@OPIcOmDcKtPCMtPCMtPCMtPCHp xRMuTSMuTSMuTSMuXCOo@eL~h@OPIcOmDcKtLCMsPsLtLCMsPCHpxRMvTcMuXSMvTcMu\COo@eL ~h@OPIcOmDcKtHCMrPcLtHCMrPCHpxRMwTsMu\SMwTsMu`COo@eL~h@OPIcOmDcKtDCMqPSLtDC MqPCHpxRMxTCNu`SMxTCNudCOo@eL~h@OPIcOmDcKt@CMpPCLt@CMpPCHpxRMyTSNudSMyTSNvp sKPIcOJpCTrxSKqxrLyLSNsdsLyLSNt@BLnXCLv@cMpXCLv@cMqpsKPIcOJpCTrxSKqxrLxLCNs `sLxLCNt@BLnXSLvDcMqXSLvDcMrpsKPIcOJpCTrxSKqxrLwLsMs\sLwLsMt@BLnXcLvHcMrXcL vHcMspsKPIcOJpCTrxSKqxrLvLcMsXsLvLcMt@BLnXsLvLcMsXsLvLcMtpsKPIcOJpCTrxSKqxr LuLSMsTsLuLSMt@BLnXCMvPcMtXCMvPcMupsKPIcOJpCTrxSKqxrLtLCMsPsLtLCMs@BLnXSMvT cMuXSMvTcMvpsKPIcOJpCTrxSKqxrLsLsLsLsLsLsLs@BLnXcMvXcMvXcMvXcMwpsKPIcOJpCTr xSKqxrLrLcLsHsLrLcLs@BLnXsMv\cMwXsMv\cMxpsKPIcOJpCTrxSKqxrLqLSLsDsLqLSLs@BL nXCNv`cMxXCNv`cMypsKPIcOJpCTrxSKqxrLpLCLs@sLpLCLs@BLnXSNvdcMyXSNvdsM||BTrxc B|@eL~tRLnHSNrdcLyHSNrdsL`@cKw@sMp\CLw@sMp\SL||BTrxcB|@eL~tRLnHCNr`cLxHCNr` sL`@cKwDsMq\SLwDsMq\cL||BTrxcB|@eL~tRLnHsMr\cLwHsMr\sL`@cKwHsMr\cLwHsMr\sL| |BTrxcB|@eL~tRLnHcMrXcLvHcMrXsL`@cKwLsMs\sLwLsMs\CM||BTrxcB|@eL~tRLnHSMrTcL uHSMrTsL`@cKwPsMt\CMwPsMt\SM||BTrxcB|@eL~tRLnHCMrPcLtHCMrPcL`@cKwTsMu\SMwTs Mu\cM||BTrxcB|@eL~tRLnHsLrLcLsHsLrLcL`@cKwXsMv\cMwXsMv\sM||BTrxcB|@eL~tRLnH cLrHcLrHcLrHcL`@cKw\sMw\sMw\sMw\CN||BTrxcB|@eL~tRLnHSLrDcLqHSLrDcL`@cKw`sMx \CNw`sMx\SN||BTrxcB|@eL~tRLnHCLr@cLpHCLr@cL`@cKwdsMy\SNwdsMy`COo@eL~h@OPIcO mDcKqdSLyDSNqdSLyHCHpxBNp`CLx@CNp`CLxDCOo@eL~h@OPIcOmDcKq`SLxDCNq`SLxHCHpxB Nq`SLxDCNq`SLxHCOo@eL~h@OPIcOmDcKq\SLwDsMq\SLwHCHpxBNr`cLxHCNr`cLxLCOo@eL~h @OPIcOmDcKqXSLvDcMqXSLvHCHpxBNs`sLxLCNs`sLxPCOo@eL~h@OPIcOmDcKqTSLuDSMqTSLu HCHpxBNt`CMxPCNt`CMxTCOo@eL~h@OPIcOmDcKqPSLtDCMqPSLtDCHpxBNu`SMxTCNu`SMxXCO o@eL~h@OPIcOmDcKqLSLsDsLqLSLsDCHpxBNv`cMxXCNv`cMx\COo@eL~h@OPIcOmDcKqHSLrDc LqHSLrDCHpxBNw`sMx\CNw`sMx`COo@eL~h@OPIcOmDcKqDSLqDSLqDSLqDCHpxBNx`CNx`CNx` CNxdCOo@eL~h@OPIcOmDcKq@SLpDCLq@SLpDCHpxBNy`SNxdCNy`SNypsKPIcOJpCTrxSKqxBLy @SNpdCLy@SNq@BLndCLy@SNpdCLy@SNqpsKPIcOJpCTrxSKqxBLx@CNp`CLx@CNq@BLndSLyDSN qdSLyDSNrpsKPIcOJpCTrxSKqxBLw@sMp\CLw@sMq@BLndcLyHSNrdcLyHSNspsKPIcOJpCTrxS KqxBLv@cMpXCLv@cMq@BLndsLyLSNsdsLyLSNtpsKPIcOJpCTrxSKqxBLu@SMpTCLu@SMq@BLnd CMyPSNtdCMyPSNupsKPIcOJpCTrxSKqxBLt@CMpPCLt@CM`@cKyTSNudSMyTSNudcM||BTrxcB| @eL~tRLn@sLpLCLs@sLpLCHpxRNvdcMyXSNvdcMy\COo@eL~h@OPIcOmDcKpHCLr@cLpHCLr@BL ndsMy\SNwdsMy\SNxpsKPIcOJpCTrxSKqxBLq@SLpDCLq@SL`@cKy`SNxdCNy`SNxdSN||BTrxc B|@eL~tRL`DCOo@eL~h@Oo@u[legLdycB||RRm]fLdycB||rSbifLdycB||dXjICY`PU^pUVObL T]rYWYrPfH`dFY}HB\l}F]OIfZeMF]qTcH`Lt[nMWZsQWYnQWObDcH`XUZsefXlUVObDcH`@u[i yF]SQW^lUVObXTZlqVYdMTZrMF[eMgH`pTZnUvUiQF]hucHpxRMb@BSiyVYSQW^lUVObLu[leFY b@RUMUv\hucHq@CLb@BSiyVYC}F[oIGUyAWY}HbQlEF]b@BSiyVYC}F[oIWObLBLp@CLp@cH`pT ZnUv\Vev\iIF[eucHqHBHDev\c}f[tef[ueF]yMUYaIwXhucHqHbO|TD^pIGHOAG]}HRUNEV[eI bOqQC^||RQxAg\~h@OEaG\rArSpQWObTUSiyfH~trL||RQxAg\~h@OEaG\rArSpQWObTUSaagH~ tbL||RQxAg\~h@OEaG\rArSpQWOb`eQuyvXtev[nIbOqQC^||RQxAg\~h@OEaG\rArSpQWObdeQ uyvXtev[nIbOqQC^`lBHrpsKEaG\rycB|dT[gICY~pCToqV^rPFHFeF[lUFY}HBLb@rPl}v\eQV Ob@cH~pCTrxSKs@RKqpsKPIcOJpCTrxSKrxRNxdCNy`SNxdSN`tBLndCNy`SNxdCNy`SNypsKPI cOJpCTrxSKrxRNwdsMy\SNwdCN`tBLndsMy\SNwdsMy\SNxpsKPIcOJpCTrxSKrxRNvdcMyXSNv dsM`tBLndcMyXSNvdcMyXSNwpsKPIcOJpCTrxSKrxRNudSMyTSNudcM`tBLndSMyTSNudSMyTSN vpsKPIcOJpCTrxSKrxRNtdCMyPSNtdCMy@RKpxRNtdCMyPSNtdCMyTCOo@eL~h@OPIcOmHcKyLS NsdsLyLSNsdCHm@cKyLSNsdsLyLSNsdCM||BTrxcB|@eL~tbLndcLyHSNrdcLyHSN`tBLndcLyH SNrdcLyHSNspsKPIcOJpCTrxSKrxRNqdSLyDSNqdSLy@RKpxRNqdSLyDSNqdSLyHCOo@eL~h@OP IcOmHcKy@SNpdCLy@SNpdCHm@cKy@SNpdCLy@SNpdSL||BTrxcB|@eL~tbLn`SNxdCNy`SNxdSN `tBLn`SNxdCNy`SNxdSN||BTrxcB|@eL~tbLn`CNx`CNx`CNx`SN`tBLn`CNx`CNx`CNx`CNyps KPIcOJpCTrxSKrxBNw`sMx\CNw`sMy@RKpxBNw`sMx\CNw`sMx`COo@eL~h@OPIcOmHcKxXCNv` cMxXCNvdCHm@cKxXCNv`cMxXCNv`sM||BTrxcB|@eL~tbLn`SMxTCNu`SMxTSN`tBLn`SMxTCNu `SMxTCNvpsKPIcOJpCTrxSKrxBNt`CMxPCNt`CMx@RKpxBNt`CMxPCNt`CMxTCOo@eL~h@OPIcO mHcKxLCNs`sLxLCNs`CHm@cKxLCNs`sLxLCNs`CM||BTrxcB|@eL~tbLn`cLxHCNr`cLxHCN`tB Ln`cLxHCNr`cLxHCNspsKPIcOJpCTrxSKrxBNq`SLxDCNq`SLx@RKpxBNq`SLxDCNq`SLxHCOo@ eL~h@OPIcOmHcKx@CNp`CLx@CNp`CHm@cKx@CNp`CLx@CNp`SL||BTrxcB|@eL~tbLn\SNwdsMy \SNwdCN`tBLn\SNwdsMy\SNwdCN||BTrxcB|@eL~tbLn\CNw`sMx\CNw`CN`tBLn\CNw`sMx\CN w`sMypsKPIcOJpCTrxSKrxrMw\sMw\sMw\sMx@RKpxrMw\sMw\sMw\sMw`COo@eL~h@OPIcOmHc KwXsMv\cMwXsMv`CHm@cKwXsMv\cMwXsMv\sM||BTrxcB|@eL~tbLn\SMwTsMu\SMwTCN`tBLn\ SMwTsMu\SMwTsMvpsKPIcOJpCTrxSKrxrMt\CMwPsMt\CMw@RKpxrMt\CMwPsMt\CMwTCOo@eL~ h@OPIcOmHcKwLsMs\sLwLsMs\CHm@cKwLsMs\sLwLsMs\CM||BTrxcB|@eL~tbLn\cLwHsMr\cL wHsM`tBLn\cLwHsMr\cLwHsMspsKPIcOJpCTrxSKrxrMq\SLwDsMq\SLw@RKpxrMq\SLwDsMq\S LwHCOo@eL~h@OPIcOmHcKw@sMp\CLw@sMp\CHm@cKw@sMp\CLw@sMp\SL||BTrxcB|@eL~tbLnX SNvdcMyXSNvdsM`tBLnXSNvdcMyXSNvdsM||BTrxcB|@eL~tbLnXCNv`cMxXCNv`sM`tBLnXCNv `cMxXCNv`cMypsKPIcOJpCTrxSKrxbMwXsMv\cMwXsMw@RKpxbMwXsMv\cMwXsMv`COo@eL~h@O PIcOmHcKvXcMvXcMvXcMv\CHm@cKvXcMvXcMvXcMvXsM||BTrxcB|@eL~tbLnXSMvTcMuXSMvTs M`tBLnXSMvTcMuXSMvTcMvpsKPIcOJpCTrxSKrxbMtXCMvPcMtXCMv@RKpxbMtXCMvPcMtXCMvT COo@eL~h@OPIcOmHcKvLcMsXsLvLcMsXCHm@cKvLcMsXsLvLcMsXCM||BTrxcB|@eL~tbLnXcLv HcMrXcLvHcM`tBLnXcLvHcMrXcLvHcMspsKPIcOJpCTrxSKrxbMqXSLvDcMqXSLv@RKpxbMqXSL vDcMqXSLvHCOo@eL~h@OPIcOmHcKv@cMpXCLv@cMpXCHm@cKv@cMpXCLv@cMpXSL||BTrxcB|@e L~tbLnTSNudSMyTSNudcM`tBLnTSNudSMyTSNudcM||BTrxcB|@eL~tbLnTCNu`SMxTCNu`cM`t BLnTCNu`SMxTCNu`SMypsKPIcOJpCTrxSKrxRMwTsMu\SMwTsMv@RKpxRMwTsMu\SMwTsMu`COo @eL~h@OPIcOmHcKuXSMvTcMuXSMvXCHm@cKuXSMvTcMuXSMvTsM||BTrxcB|@eL~tbLnTSMuTSM uTSMuTcM`tBLnTSMuTSMuTSMuTSMvpsKPIcOJpCTrxSKrxRMtTCMuPSMtTCMu@RKpxRMtTCMuPS MtTCMuTCOo@eL~h@OPIcOmHcKuLSMsTsLuLSMsTCHm@cKuLSMsTsLuLSMsTCM||BTrxcB|@eL~t bLnTcLuHSMrTcLuHSM`tBLnTcLuHSMrTcLuHSMspsKPIcOJpCTrxSKrxRMqTSLuDSMqTSLu@RKp xRMqTSLuDSMqTSLuHCOo@eL~h@OPIcOmHcKu@SMpTCLu@SMpTCHm@cKu@SMpTCLu@SMpTSL||BT rxcB|@eL~tbLnPSNtdCMyPSNtdSM`tBLnPSNtdCMyPSNtdCMypsKPIcOJpCTrxSKrxBMxPCNt`C MxPCNu@RKpxBMxPCNt`CMxPCNt`COo@eL~h@OPIcOmHcKt\CMwPsMt\CMwTCHm@cKt\CMwPsMt\ CMwPsM||BTrxcB|@eL~tbLnPcMtXCMvPcMtXSM`tBLnPcMtXCMvPcMtXCMvpsKPIcOJpCTrxSKr xBMuPSMtTCMuPSMu@RKpxBMuPSMtTCMuPSMtTCOo@eL~h@OPIcOmHcKtPCMtPCMtPCMtPCHm@cK tPCMtPCMtPCMtPCM||BTrxcB|@eL~tbLnPsLtLCMsPsLtLCM`tBLnPsLtLCMsPsLtLCMspsKPIc OJpCTrxSKrxBMrPcLtHCMrPcLt@RKpxBMrPcLtHCMrPcLtHCOo@eL~h@OPIcOmHcKtDCMqPSLtD CMqPCHm@cKtDCMqPSLtDCMqPSL||BTrxcB|@eL~tbLnPCLt@CMpPCLt@CM`tBLnPCLt@CMpPCLt @CM||BTrxcB|@eL~tbLnLSNsdsLyLSNsdCM`tBLnLSNsdsLyLSNsdsLypsKPIcOJpCTrxSKrxrL xLCNs`sLxLCNt@RKpxrLxLCNs`sLxLCNs`COo@eL~h@OPIcOmHcKs\sLwLsMs\sLwPCHm@cKs\s LwLsMs\sLwLsM||BTrxcB|@eL~tbLnLcMsXsLvLcMsXCM`tBLnLcMsXsLvLcMsXsLvpsKPIcOJp CTrxSKrxrLuLSMsTsLuLSMt@RKpxrLuLSMsTsLuLSMsTCOo@eL~h@OPIcOmHcKsPsLtLCMsPsLt LCHm@cKsPsLtLCMsPsLtLCM||BTrxcB|@eL~tbLnLsLsLsLsLsLsLsL`tBLnLsLsLsLsLsLsLsL spsKPIcOJpCTrxSKrxrLrLcLsHsLrLcLs@RKpxrLrLcLsHsLrLcLsHCOo@eL~h@OPIcOmHcKsDs LqLSLsDsLqLCHm@cKsDsLqLSLsDsLqLSL||BTrxcB|@eL~tbLnLCLs@sLpLCLs@sL`tBLnLCLs@ sLpLCLs@sL||BTrxcB|@eL~tbLnHSNrdcLyHSNrdsL`tBLnHSNrdcLyHSNrdcLypsKPIcOJpCTr xSKrxbLxHCNr`cLxHCNs@RKpxbLxHCNr`cLxHCNr`COo@eL~h@OPIcOmHcKr\cLwHsMr\cLwLCH m@cKr\cLwHsMr\cLwHsM||BTrxcB|@eL~tbLnHcMrXcLvHcMrXsL`tBLnHcMrXcLvHcMrXcLvps KPIcOJpCTrxSKrxbLuHSMrTcLuHSMs@RKpxbLuHSMrTcLuHSMrTCOo@eL~h@OPIcOmHcKrPcLtH CMrPcLtHCHm@cKrPcLtHCMrPcLtHCM||BTrxcB|@eL~tbLnHsLrLcLsHsLrLcL`tBLnHsLrLcLs HsLrLcLspsKPIcOJpCTrxSKrxbLrHcLrHcLrHcLr@RKpxbLrHcLrHcLrHcLrHCOo@eL~h@OPIcO mHcKrDcLqHSLrDcLqHCHm@cKrDcLqHSLrDcLqHSL||BTrxcB|@eL~tbLnHCLr@cLpHCLr@cL`tB LnHCLr@cLpHCLr@cL||BTrxcB|@eL~tbLnDSNqdSLyDSNqdcL`tBLnDSNqdSLyDSNqdSLypsKPI cOJpCTrxSKrxRLxDCNq`SLxDCNr@RKpxRLxDCNq`SLxDCNq`COo@eL~h@OPIcOmHcKq\SLwDsMq \SLwHCHm@cKq\SLwDsMq\SLwDsM||BTrxcB|@eL~tbLnDcMqXSLvDcMqXcL`tBLnDcMqXSLvDcM qXSLvpsKPIcOJpCTrxSKrxRLuDSMqTSLuDSMr@RKpxRLuDSMqTSLuDSMqTCOo@eL~h@OPIcOmHc KqPSLtDCMqPSLtDCHm@cKqPSLtDCMqPSLtDCM||BTrxcB|@eL~tbLnDsLqLSLsDsLqLSL`tBLnD sLqLSLsDsLqLSLspsKPIcOJpCTrxSKrxRLrDcLqHSLrDcLq@RKpxRLrDcLqHSLrDcLqHCOo@eL~ h@OPIcOmHcKqDSLqDSLqDSLqDCHm@cKqDSLqDSLqDSLqDSL||BTrxcB|@eL~tbLnDCLq@SLpDCL q@SL`tBLnDCLq@SLpDCLq@SL||BTrxcB|@eL~tbLn@SNpdCLy@SNpdSL`tBLn@SNpdCLy@SNpdC LyDCOo@eL~h@OPIcOmHcKp`CLx@CNp`CLxDCHm@cKp`CLx@CNp`CLx@CNqpsKPIcOJpCTrxSKrx BLw@sMp\CLw@sMq@RKpxBLw@sMp\CLw@sMp\SL||BTrxcB|@eL~tbLn@cMpXCLv@cMpXSL`tBLn @cMpXCLv@cMpXCLvDCOo@eL~h@OPIcOmHcKpTCLu@SMpTCLuDCHm@cKpTCLu@SMpTCLu@SMqpsK PIcOJpCTrxSKrxBLt@CMpPCLt@CM`tBLn@CMpPCLt@CMpPCLtpsKPIcOJpCTrxSKrxBLs@sLpLC Ls@sL`tBLn@sLpLCLs@sLpLCLspsKPIcOJpCTrxSKrxBLr@cLpHCLr@cL`tBLn@cLpHCLr@cLpH CLrpsKPIcOJpCTrxSKrxBLq@SLpDCLq@SL`tBLn@SLpDCLq@SLpDCLqpsKPIcOJpCTrxSKr@BL| |BTrxcB||BToqV^rPfOJpsKIuvYrPfOJpsKOIfZrPfOJpcUiUv]iyvYB}F^XuTZnAbUaqVObtrL n@CLp@CLp@CLpDcH~trLn@COoXUZe]WZn]fPoaGVMef[~h@OVeVYwef[gIt[xaUSaaGHVEF[}Hr Ln@CLp@CLp@CLpDcH~LcKppsKVeVYwef[gIt[xaUSaagOJpcUiUv]iyvYB}F^YuTZnAbUaqVObt RLn@CLp@CLp@CLpDcH~tRLn@COoXUZe]WZn]fPoaWVMef[~h@OVeVYwef[gIt[xeUSaaGHVEF[} HRLn@CLp@CLp@CLpDcH~DcKppsKVeVYwef[gIt[xeUSaagOJpCRiyF]sArTcEF[iyvY}HrPoyv\ tIWXiyVYdIrK~psKC}v[rQVZnEF]eMU^sQWYmICY~h@OoLuXeyVYrPfOJpsKCEf[vEv\~h@ %%%%%%%%%%%%%%%%%%%%%% End /document/N4QRHX0F.xvz %%%%%%%%%%%%%%%%%%%%% %%%%%%%%%%%%%%%%% Start /document/graphics/Image80.gif %%%%%%%%%%%%%%%% GedQxdSXZI@~@\O@@@@@@|             sB@@@@@ZI @~@@@B~Cp@HpARpBZpCbpDjpErpFzpGBqHJqIRqJZqKbqLjqMrqNzqOBrPJrQRrRZrSbrTjrUrr VzrWBsXJsYRsZZs[bs\js]rs^zs_Bt`JtaRtbZtcbtdjtertfztgBuhJuiRujZukbuljumrunzu oBvpJvqRvrZvsbvtjvurvvzvwBwxJwyRwzZw{bw|jw}rw~zwBx@KxASxB[xCcxDkxEsxF{xGCy HKyISyJ[yKcyLkyMsyN{yOCzPKzQSzR[zSczTkzUszV{zWC{XK{YS{Z[{[c{\k{]s{^{{_C|`K| aS|b[|cc|dk|es|f{|gC}hK}iS}j[}kc}lk}ms}n{}oC~p~oxGOyKoyOOzSozWO{[o{_O|co|gO }ko}oO~so~wO{oO@F`ED@@@BhANUDxAj`OUBn`NhRu`OhDVTQxDZa@UE^aZxNea[hGZS]xGJ bqTHNbfHKUbghJ^RixJzbbTK~brXGEcshMbQuxMjcSTNnc~hCuchPfPAyPZdDTQ^dGYRjdCISv dxSBe{hTNewXUZesHVfeoxVrekhW~egXXJfcHYVf_xYbf[hZnfWX[zfSH\FgOx\RgKh]^gGX^j gBH_vg@x_Bh~g`Nh|WaZhzGbfhxwbrhvgc~htWdJirGeVipwebingfnilWgzijGhFjhwhRjfgi^ jdWjjjbGkvj~C^oJlz]rzlr]ujmj]xZnb]{JoZ]~zoR]AkpJ]D[qB]GKrz\J{rr\Mksj\P[tb\S {`ucV{ifm@VQnm^jwFWDhx^PLIxVifk[ubRIzrhlK[inEHOFoTKJNomfxboYXHfoxkAynCzjVi x@W`eX@wgGLZQpDYRnvK}bRI|YmpL|CcdVLDWRRlYQqHtxzqX\FOR[\YuqN\HG@dl[jrcE|Bri LFsrkiLGVp[LWpwlHkRt\WqoF|CwoMl@qseIQnXkQKfI}Zatz\}ztwRCuffTOudVU[ubFVgu` vVsu^fWu\VXKvZFYWvXvYcvVfZovTV[{vRF\GwPv\SwNf]c`~|~{M_ww~}_CxA^~vjymLIxGNb gxJ~bsxMNvdENGyxSnPoUT~eKbQnB]y}MXsy{]`brVNgKmd^|mtMNgrxhjhOTJn\fx[Gx^xNi` [lmNLEx@L`g\rz_gk^pui]~TS|yFbZ|w]rC[yIh_wPds_oxNtxOFysjvVV}VoFOxW}t{oUNw Wo_|gEd^xWShoYc{b_zgSn[crq{sStcmoZnpV~rMPkKKCMoB`uoeBA\qt`~_blCHbIAi @uYOPXDCl@r`b`_DKHT{~IXp`EOpgTysABCabpQHIlDZBOahpThJ\ErBEaXeVHLLFJCgatpZhM| FbCmaWKOHEL{pA{a`[W~a@QzbP\HJWJbFQxBRLIfnxaLQaszsa}DiJ[oyrxZBBsfbUeexP[@Jzx bYLnh~Z~yDubnnrXIsqpLczm{rX\MrFcJbQuxKKNBUfcCHrh]|NbFIKoqzx^li@FcPnl@i`dN~ VnbERCiADQfHu`YQFYRqomA{cRRhhe|RjIwd\rNig|SBJCebrQIilTZJOehrTij\UrJ[enrWIlL VJKgetrZim|VbKsezr]IolWzKe@s`ip\XRLKfFscIrLYjLWfLsfis|YBMcfRsiIulZZMofXsli v\[rM{f^soyPI`@@@lC %%%%%%%%%%%%%%%%%% End /document/graphics/Image80.gif %%%%%%%%%%%%%%%%% %%%%%%%%%%%%%%%%%%%%% Start /document/N4QRHX0D.xvz %%%%%%%%%%%%%%%%%%%% ||C^mqFHvUf\sev[nucHqxBLb|cOJpSHD}tPTeETEArPayf]aMGHSeuTTUTS`HR[uAG[oQW[lyB YtQfH~h@OCEf[vEv\`dFY}HB\l}F]OIfZeMF]vHBHC}f[sev\tUf[tucHqHBHLEV^oUG]}HBUaI V]lEf\b@bQo}F]eIgQoyF]}HbIaAw[sms\ayv\mLWYrefYfDF\oMwN`DcLb@rSuQG\uQWUneF]s ucHMudH``TYi]FZtucHu@cKpLCNb@bQo}F]eIWPlevYnuVYnQWObLTYnQWYrIBHBEvXk]f\oUg[ dMt[l}f\}HrHFYdQFYdQb@bPoIGYeIwPoqv[rucHc`CLx@CNpHBHB}f\dUf\WeFYtaVOb@cH`pT YfQWSaIwYiyVObDcH`DT]t}FTlEV^}HBLb@bTi]FZtuTXr]VZnucHqHBHWeFYtaVOb\SMnXSLu` cH`Dd[iuVXtev[nME]yqVY}HbTuyvSnMVYb@rTpEvXiyvY}HRLb@rPoqV]myv\}HBLb@bTo]w\} HBLb@BReEFYeIgQoyF]}HbIaAw[sms\ayv\mLWYrefYfDF\oMwN`DcLb@BReEFYeIWPlevYnuVY nQWObLTYnQWYrIBHB}F]t}V[MEf\gef[}HRLb@BUoAWSaIwYiyVObDcH~h@OSMVYnUfLdME]yqV Y`HTXcmvYr}V]nQFUrEf[sAWXrUf[tucHpHBHF}v[tUf\F}f[tucHfDF\oMwNsEf[sur\eIWZfY RXp}v\{@RLrHBHHUVZgaF]}HBNpHBHLUvYeyFYPqVXcUV[eyF]}HbPoQG]oufH`HTXcmvYr}V]n QvPoqv[rucHcXdQFYdQFIBHF}v[tUf\AqVZgyV[eyF]}HrPeyF]eIgH`pTYgUf[dYt[nQWObXRX p}v\{LWXnMWKsUf\iYfIaAw[smCHxHBHB}f\dUf\C}F[oIWObLBNp`CLx@cH`Ht[rQVYr]UZdQG Z}HBLb@BSe]VYnQVPlevYnuVYnQWObLTYnQWYrIBHLUfYtuTXr]VZnucHqHBHRevYhQWSaIwYiy VObDcH`\UZdQGZ}HRLr@cH`pTYgUf[dYUZsefXlUVOb@cH``TYaQVYrYt[nQWObXRXp}v\{LWXn MWKsUf\iYfIaAw[smCHqHcH``TYaQVYrED[i]f[mUf[tucHCUf[tUf\b@bPoQG]ouVSaIwYiyVO bDcH`Pu[puTXr]VZnucHqHrK~h@OC}v[rQVZnEF]eMU^sQWYmICYSQW^lUFHSMVXlef[gucHUyv Xoyv\tIWXiyVYdIBHXQUZcmv\NUW[bUf\}HbSoIW[aqfH``UPxev\TeF]lUVPlevYnuVYnQWObT d[dIBHYQUZcmv\Vev\iIF[eucHqHBHSUgXgIWZdqTZnUvPoqv[rucHcLtPCMtPCIBHSUgXgIWZd qTZnUvUiQF]hucHpxRLb@RVTevXkMGSaIVYlMgUiMWZbqVY}HRLb@RPxUv\Lef[eMt[l}f\}HrH p@CLp@CLb@RPxUv\Lef[e]UZdQGZ}HBLnDCNb@BUiMvZsqTXbUF[F}f[tucHfDF\oMwNTeV[eMG HNUv]`Hu[mEf[fDF\oMwN``CHc@CLp@CLpHBHYED^iMGUiQG[eED[i]f[mUf[tucHEyFYb@RVTe vXkMgSuufXeIWObxt[ruVXlIBHC}v[rQVZnEF]eQU^pUVObpTZnqTZnIBHGIWZdqTZnUvTteG[e ucHS}F[iQfH``uQreFYVev\iIF[eucHpHBHGIWZded[FIw[nQWOb@cH``UPxev\Vev\iIF[eucH qHBHSUgXgIWZdqTZnUvTteG[eucHS}F[iQfH``uTuIvYreFYVev\iIF[eucHpHBHAaWYsQUZtqV YF}f[tucHfDF\oMwNTeV[eMGHNUv]`Hu[mEf[fDF\oMwN`DcL`LBLp@CLp@cH``EUiMvZsqTXbU F[SQW^lUVOb`t[ref^oyF]aqfH``EUiMvZsITYt]WYeyVObDcH`duQreFYVev\iIF[eucHpHBHY ED^iMGUiQG[e}d\iUf[tEF]i}f[}HBRoIWZz}f[tEF[b@RPxUv\}HRPuQw[mEF]iMfH`DD^eMWR nYd\oyF]}HBLb@RVAaWZsYUZsefXlUVObDcH``EUiMvZsYUZsefXlUVObDcH`PUZcmv\LUf[gQG Z}HbLb@rQreFYLef[eMt[l}f\}HrHydSNydSNb@RVSUgXgIWZdYUZsefXlUVOb@cH``EUiMvZsq TXbUF[sYUZsefXlUVObDcH`\d\iQFSiyVYWeFYtaVOb@cKqHBHYQUZcmv\LEfXeqvTteG[eucHH }f\iiw[nQWXlIBHYQUZcmv\BUF]wUVYnucHqHBHAaWYsQUZpMWOb@cHoxcB||dXjME]yqVY`PU^ pUVObPTYfEV]lQgH`XTZlqvPoqv[rucHcXdQp@CLpHBHLef[eMt[l}f\rtcHcXdQqPSNsHBHVUf \tevXaqVPseW[pQw[tUv\SQW^lUVObPTXsaVYdIBHFeF[lAUXtQWYryVObPTZa]v[nEF[Lef[eM gH`PUZmUfPe]VZnucHpHBHXuTYsaVObDSLb@BUuIV]lEf\}HBLb@BUiAGSeyvYtaVObPcH`XUYr QWZcEF[AMW^mAG]oQWYsYUZsefXlUVObDcH`PUZtqVYF}f[tucHfDF\oMwNsEf[sur\eIWZfYRX p}v\{@RLqHBHLef[eME]yqVY}HrToqVZdIBHLef[eMt[l}f\Def\eMF]i}f[XucHpHBHOIG]h}v YoyVXlAe\oiVYcQWZoyVOb@cH`DdYfUvXtYUZe]WZn]fPoaWObDcH`pTZnUvPoqv[rQTZrUvXte v[neUObDcH`PUZtqVYAqVZgyV[eyF]}HrPeyF]eIgH`pTZnUvPoqv[rQTZrUvXtev[niUObDcH` Xd\auVYsucHu@cH`@u[iyF]Sef^eucHqxRMb@RPrIw[wqTYn]F]hucHPIw[p}f\tev[nEF[b@RU MUv\hucHrTcH`XTZlqvPoqv[rQU^pUVObPTZcaf\ouVXtevXb@BVLef[eMgUiMWZbqVY}HRLb@b UiMWZbqVYAYF]eIWQnQVObDcH`pTZgaF]C}F[oIWObLbQFYdQFYdH`DDYaAG]iYWYMUv\hucHpH BHXMU]buVYsaVOb@cH`TD^tUf[sev[nucHFef[iQWYb@BSiyVYsYUZsefXlUVObDcH`dUSeMGZ} HRLqHBHFeF[lMt[l}f\rtcHcXCMyTSQDIBHSaVXdef[gucHSuv[oQGZb@BToef[tME]yqVY}HbQ iqF[eQvPiIwXlUv\b@bQiqF[C}F[oIGQiIWYcQWZoyFV}HBLb@BUiuVYEyFY}HRLpHBHCEV[eIW XC}v[rQVZnEF]eMWOb@cH`TESiyVYsYUZsefXlUVObDcH`XUZsefXlUfPeYv[rUfPe]VZnucHqH BHFeF[lME]yqVY}HRQvUf[OQFYb@bQiqF[C}F[oIGQiIWYcQWZoyVV}HBLb@bUeIG]iMVXlEt\y uF\t}F]eMwPoqv[rucHc`CLx@CNpHBHUMU]buVYsaVOb@cH`XTZlqvPoqv[rQTZrUvXtev[niUO bDcH`XUYrQWZcEF[AMW^mAG]oQWYs]UZdQGZ}HBLnHcH`XUSeMGZ}HbLuHBHYqTZnUv\Vev\iIF [eucHqHBHYMU]buVYsaVOb@cH`PU]bUFQiEV[eQWYrucHrxRMb@bQiqF[eQVObDcH`hUSeMGZ}H RLqHBHP}VZnQw\Vev\iIF[eucHpHBHLef[eMt[l}f\}HrHp@CLpXdQb@BSiyVYWeFYtaVOb@cKs TcH`pTYgUf[dUd[tIW^}HBLb@bULef[eMgUiMWZbqVY}HRLb@bUSUgXmUv\hucHpHBHTeF\AyvY lUVOb@cKtDCNx\SNpHCLt\SNb@BUiAwTteG[eucHFeF[lUFYb@rReUF\UAgUeMF]oIWObDcH`PT ZsMv[nQWZnUWZtewTeEf\caVObDcH`tTYsafUiMWZbqVY}HBLb@rPl}v\eQVOb@cH`@u[iyF]C} F[oIWObLRLyDSNw@cH`pTZnUvPoqv[rQU^pUVObXD[aQgHoxcB||dXjME]yqVY`PU^pUVObLT]r YWYrPfH`pTYgUf[dMt[l}f\}HBSiyVYC}F[oIgH`pTYgUf[dUd[tIW^}HRLb@RUMUv\hucHqHSL b|bOJpsTcUf[eICY`dFY}HB\l}F]OIfZeMF]wHBHC}f[sev\tUf[tucHqHBHLUfYtucHpHBHB}F ]t}V[}HBLbxCOC}v[rQVZnEF]eMU^sQWYmICYSQW^lUFHAaWYsQUZtqVYF}f[tucHfDF\oMwNTe V[eMGHNUv]`Hu[mEf[fDF\oMwN`DcL`LBLp@CLp@cH`PUZcmv\LEfXeqfQoyF]}HbIaAw[smCUi uVYsAbSe]GHR}V[ayfIaAw[smCHx@rHp@CLp@CLb@rTcEF[iyvY}HRUnMv[nMG]rEVZnUFYb@RP xUv\}HRPuQw[mEF]iMfH`DD^eMGUiAw\}HBLb|bOJpsSbivTteG[eABUyAWY}HrPuIg]eICYb@B Se]VYnQvPoqv[rucHLef[eMt[l}f\b|bOJpsPo}f\def[aQWYSew\tUV[rPFHiQVOb@G[oQwSbi VYcQGNb@rPoyv\iMG]eyF]}HRLb@RPxUv\TeF]lUfQoyF]}HbIaAw[smCUiuVYsAbSe]GHR}V[a yfIaAw[smCHqHCHc@CLp@CLpHBHTevXkMGSaIVYlYt[nQWObXRXp}v\{PUZmUv\`xTYwAbTouVX nYRXp}v\{@BN`LBLp@CLp@cH`LuXaqVZn]VObTe[c}f[sQg\aef[eQfH`DD^eMWObDT]t}V[aQW ZcIBHAaWYsQUZpMWOb@cH`DT]t}fUiUv]iyvYB}F^}HRLuHbO|DD^eMwSrevYiyFV`XUXlucHpH bOppsKAaWYs}d\i]VZnaeOJpSPxUv\OIWZgef[YAbUaqVOb@cH~@COoDD^eMwSrevYiyVV~h@OX ED^iMGUiQG[eyC^||BVAaWZsQUZtqVY~h@OYED^iMGUiQG[eyS^||RVAaWZsQUZtqVY~h@OXQUZ cmv\AyvXh}f\`XUXlucHpHbOppsKXQUZcmv\AyvXh}f\~h@OXQUZcmv\Dev\tEf[cUFHVEF[}HB LbxCL||BVTevXkMGQiMG]ayvXeycB|dEUiMvZsEd[cav[rAbUaqVOb@cH~@COodEUiMvZsEd[ca v[rycB|dEUiMvZsQTZsQWXnMVY`XUXlucHpHbOppsKYQUZcmv\Dev\tEf[cUfOJpsSbivTteG[e ABUyAWY}HrPuIg]eICYb@BSe]VYnQvPoqv[rucHLef[eMt[l}f\b|bOJpsSbifLdABUyAWY}HrP uIg]eICYb@RZducHpqv[t}dXjUvXtEcH`Lt[nMWZsQWYnQWObDcH`XUZsefXlUVObDcH`@u[iyF ]SQW^lUVObXTZlqVYdMTZrMF[eMgH`pTZnUvUiQF]hucHpxrLuHBHLef[eME]yqVY}HrToqVZdI BHUuTYsaVObPSNb@BSiyVYC}F[oIGUyAWY}HbQlEF]b@BSiyVYC}F[oIWObLBLp@CLp@cH`pTZn Uv\Vev\iIF[eucHqHBHDev\c}f[tef[ueF]yMUYaIwXhucHqHbO|TD^pIGHOAG]}HRUNEV[eIbO qEC^||RQxAg\~h@OEaG\rArSpQWObTUSiyfH~@COoTD^pIgOJpSQxAg\`|D\tucHUuTXxIbOqps KEaG\rycB|TD^pIGHOAG]}HBVFUg[cQWZoyfH~DWLxqsKEaG\rycB|TD^pIGHOAG]}HRVFUg[cQ WZoyfH~HsLjLWZnaRLq|bLj@URjDWLxeBHm@rLjLWZnaRMoHcJPedJqEC^ipsKEaG\rycB|dT[g ICY~pCToqV^rPFHFeF[lUFY}HBLb@rPl}v\eQVOb@cH~pCTrxCL`@COo@eL~h@OPIcOpxBLr@CN sLsLsLsLsLsL`\cKvDsLpXCMv`cL||BTrxcB|@eL~@cKpPSLvXcMvXcMvXcMw@RLtxbLp@cMsTs LuDCM||BTrxcB|@eL~@cKpXcLu@RLxxBNvdSNy`CNvdSM||BTrxcB|@eL~@cKp`sLsLsLsLsLsL sLs@bLpxRNwXSNt\SMrPcM||BTrxcB|@eL~@cKq@CMqXcMvXcMvXsM`HCLnHCLyTsMxLCLuLCOo @eL~h@OPIcOpxRLrTCHqXcKvHSNsdcLrPcMqpsKPIcOJpCTrxCLnDCMu`sLsLsLsLsLs@RLpxbM vPsLyTSMuLcL||BTrxcB|@eL~@cKqXcMvXcMvXcMvXsM`LcKpTSMpXCLuTCNtdCOo@eL~h@OPIc OpxRLx\SM`tRMnHsLydCMxPCLwXCOo@eL~h@OPIcOpxbLp`sLsLsLsLsLsLCHmDsLnDcMvHSLvX CMt`COo@eL~h@OPIcOpxbLrdSLvXcMvXcMv\CHmDSNn\CLxTCLtXCLydCOo@eL~h@OPIcOpxbLu @RKrPcKpHCLxXsMxPSMspsKPIcOJpCTrxCLnHsMp`sLsLsLsLsLs@RKrTcKuLSMrdcLpTcMupsK PIcOJpCTrxCLnHSNqXcMvXcMvXcMw@RKrPcKpLCMyDcLt@CLxpsKPIcOJpCTrxCLnLSLrTCHmDS NnXCNrPcLpHsMy`COo@eL~h@OPIcOpxrLsLsLsLsLsLsLsLCHmDsL||BTrxcB|@eL~@cKsTCMqX cMvXcMvXsM`tBMn`CLsLCMv@sLq`CM||BTrxcB|@eL~@cKs\SM`LcKy@SLx@cMtPCLsHCOo@eL~ h@OPIcOpxrLyTCNsLsLsLsLsLCHqHcKpPsMt@sLsDcMupsKPIcOJpCTrxCLnPSLvXcMvXcMvXcM w@RLxxbMs`sMpTCMpLCM||BTrxcB|@eL~@cKtLsMu@bLrxBNx@CMxDsMuLcM||BTrxcB|@eL~@c KtTCNsLsLsLsLsLsL`HCMnHsMwXcLqLCLuDCOo@eL~h@OPIcOpxBMwdSLvXcMvXcMv\CHrHcKvd sMs@SLpXsLxpsKPIcOJpCTrxCLnTCHq`cKs`CMw\cMsDCLypsKPIcOJpCTrxCLnTcLp`sLsLsLs LsLs@RLqxRNs@CNpDsMsdsL||BTrxcB|@eL~@cKuPSLvXcMvXcMvXsM`PcKqdCMxdCLqdcMxdCO o@eL~h@OPIcOpxRMvHSM`trLn`CLu\cLvTcMyXcM||BTrxcB|@eL~@cKu`sLsLsLsLsLsLsL`tR LqxBLr\SLw`cLxLSL||BTrxcB|@eL~@cKv@CMqXcMvXcMvXsM`tRLvxRMsLcLuXCMqHsM||BTrx cB|@eL~@cKvHSM`tRLyxbMqTsMpTcMp`SL||BTrxcB|@eL~@cKvPSMxLsLsLsLsLsL`tRLyxBNx TCLw`CNu\sM||BTrxcB|@eL~@cKvXcMvXcMvXcMvXsM`tRLwxrLr@SMp`CLwTsM||BTrxcB|@eL ~@cKv`sMu@RKqHcKr\cLpDCMq\SMwpsKPIcOJpCTrxCLn\CLxLsLsLsLsLsLs@RKuxBMqTCLw@c LuXcMwpsKPIcOJpCTrxCLn\cLyDcMvXcMvXcMw@bLnLsLv\CMrPSMv`SM||BTrxcB|@eL~@cKwT CHyxRNtdsMvdcLtDCMypsKPIcOJpCTrxCLn\sMp`sLsLsLsLsLs@RLvxBMpLCNuDsMsTcM||BTr xcB|@eL~@cKwdSLvXcMvXcMvXsM`HCLn`cLtHCNrPCMrdCOo@eL~h@OPIcOpxBNqHSM`HcLnTSN uDSNwHSNrTCOo@eL~h@OPIcOpxBNsLsLsLsLsLsLsLCHrDcKtLSNxLcMxXSNspsKPIcOJpCTrxC Ln`SMtDcMvXcMvXcMw@RLwxBMu\sLuHcMuXcL||BTrxcB|@eL~@cKx\SM`DSLnDSLqPCLtXcMpP COo@eL~h@OPIcOpxBNyTCNsLsLsLsLsLCHsxRLwDSNw\SLtLsMrpsKPIcOJpCTrxCLndSLvXcMv XcMvXcMw@RKuxrLxHSLvHCMtDSNspsKPIcOJpCTrxCLndsLwTCHmDsLnPCNw`CNx\CM||BTrxcB |@eL~@cKyTCNsLsLsLsLsLsL`tbLpxRLsLSLpTSNv@SM||BTrxcB|@eL~@cKy\SNqXcMvXcMvXs M`tbLtxBMxTsMvXsLqDsM||BTrxcB|@eL~DCHmHcM||BTrxcB||BToqV^rPfOJpsKIuvYrPfOJp sKOIfZrPfOJpsSbifLdABUyAWY}HrPuIg]eICYb@RZducHpqv[t}dXjUvXtIcH`Lt[nMWZsQWYn QWObDcH`XUZsefXlUVObDcH`@u[iyF]SQW^lUVObXTZlqVYdMTZrMF[eMgH`pTZnUvUiQF]hucH pxrLuHBHLef[eME]yqVY}HrToqVZdIBHUuTYsaVObPSNb@BSiyVYC}F[oIGUyAWY}HbQlEF]b@B SiyVYC}F[oIWObLbQFACLp@cH`pTZnUv\Vev\iIF[eucHqHBHDev\c}f[tef[ueF]yMUYaIwXhu cHqHbO|TD^pIGHOAG]}HRUNEV[eIbOqEC^||RQxAg\~h@OEaG\rArSpQWObTUSiyfH~@COoTD^p IgOJpSQxAg\`|D\tucHUuTXxIbOqpsKEaG\rycB|TD^pIGHOAG]}HBVFUg[cQWZoyfH~DWLxqsK EaG\rycB|TD^pIGHOAG]}HRVFUg[cQWZoyfH~HsLjLv[saRLnLsMuTVKqhBTIebJsef[hDSLoHc JPedJqEC^i@RK`LcJc}v\hXcKrTSYmHcJPeTJjLWZnaRMoHcJPedJqEC^ipsKEaG\rycB|dT[gI CY~pCToqV^rPFHFeF[lUFY}HBLb@rPl}v\eQVOb@cH~pCTrxCL`@COo@eL~h@OPIcOpxBLr@CNs LsLsLsLsLsL`XcKx\CNrTsLuDSLxLCOo@eL~h@OPIcOpxBLtDcMvXcMvXcMvXsM`DcLn`cLvDSM ydsLx\COo@eL~h@OPIcOpxBLvHSM`DsMn@sLsdsLpdCLwTCOo@eL~h@OPIcOpxBLxLsLsLsLsLs LsLsL`DCNndSLwPCLvTcMtLCOo@eL~h@OPIcOpxRLpPSLvXcMvXcMv\CHq`cKqdCMqLsLydcLvp sKPIcOJpCTrxCLnDcLu@RLtxRNr@cMvdSNt@SM||BTrxcB|@eL~@cKqPSMxLsLsLsLsLsL`dcKt `cMrdcMxHsLtXCOo@eL~h@OPIcOpxRLvXcMvXcMvXcMv\CHrxRMvLSNsDsMsDsLvpsKPIcOJpCT rxCLnDCNwTCHmPcKy\SMu@CLsHcLtdCOo@eL~h@OPIcOpxbLp`sLsLsLsLsLsLCHmDcLnDsMtHc MyPSNs`COo@eL~h@OPIcOpxbLrdSLvXcMvXcMv\CHmDCNnDSLpLsMwLSMs\COo@eL~h@OPIcOpx bLu@RKrHcKpDSMwHCMxHSNupsKPIcOJpCTrxCLnHsMp`sLsLsLsLsLs@RKrLcKs\CMwXCMv\CNy psKPIcOJpCTrxCLnHSNqXcMvXcMvXcMw@RKrDcKydCLy`sMsDcLvpsKPIcOJpCTrxCLnLSLrTCH mDCNn@SLr\CLvXSLuHCOo@eL~h@OPIcOpxrLsLsLsLsLsLsLsLCHmDSLndSLt`cLtPSLyXCOo@e L~h@OPIcOpxrLuPSLvXcMvXcMv\CHmPcKtLCNxdCLtXcMypsKPIcOJpCTrxCLnLsMu@rLnTCLp` CNsTcMxPsM||BTrxcB|@eL~@cKsdSMxLsLsLsLsLsL`DCLndsLsXsLv\sLtdCOo@eL~h@OPIcOp xBMqXcMvXcMvXcMv\CHqXcKyTSMpTCNq\sLwpsKPIcOJpCTrxCLnPsLwTCHr@cKxPcLpDsLy`SN xpsKPIcOJpCTrxCLnPSMxLsLsLsLsLsLs@bLrxRLtLSNtHCMqDSM||BTrxcB|@eL~@cKt\SNqXc MvXcMvXsM`HCLn\sLyLsLvHCMvDCOo@eL~h@OPIcOpxRM`DcMn`SMpDCLvHCNwTCOo@eL~h@OPI cOpxRMr@CNsLsLsLsLsLCHqDcKpDcLpDcMxPCMspsKPIcOJpCTrxCLnTCMqXcMvXcMvXcMw@BMn @CLu@SLs@sMs@CN||BTrxcB|@eL~@cKuXcLu@RKsxbLt\cMpHSLr@SLqpsKPIcOJpCTrxCLnTCN sLsLsLsLsLsLs@RKyxrMy`SLyPsMuPCMwpsKPIcOJpCTrxCLnXCLtDcMvXcMvXcMw@RKqPcKwdc MwTCMsDsL||BTrxcB|@eL~@cKvHSM`tRLwxbMp@SLs@cLrDSL||BTrxcB|@eL~@cKvPSMxLsLsL sLsLsL`tRLwxBNuPSMt@CLyDsL||BTrxcB|@eL~@cKvXcMvXcMvXcMvXsM`tRLuxRMt@sMwDCMt \SL||BTrxcB|@eL~@cKv`sMu@RKq@cKy\cMr`cMu\SNupsKPIcOJpCTrxCLn\CLxLsLsLsLsLsL s@RKtxrMwLSNw@cLxHCMvpsKPIcOJpCTrxCLn\cLyDcMvXcMvXcMw@bLnHsLwDsMvTSNtPCM||B TrxcB|@eL~@cKwTCHyxRLqdcLqDCNpdCNspsKPIcOJpCTrxCLn\sMp`sLsLsLsLsLs@RLtxRNtX SMsDCNwHSM||BTrxcB|@eL~@cKwdSLvXcMvXcMvXsM`DCNndcLuXCNs@SLsXCOo@eL~h@OPIcOp xBNqHSM`HCLnPSNxLSLsXcMw\COo@eL~h@OPIcOpxBNsLsLsLsLsLsLsLCHqdcKtDCMpLCNpDCN xpsKPIcOJpCTrxCLn`SMtDcMvXcMvXcMw@RLuxrMvLCNsTSNrLcM||BTrxcB|@eL~@cKx\SM`dc KyXSNv\cLyDsMwDCOo@eL~h@OPIcOpxBNyTCNsLsLsLsLsLCHrxrMsDcMsXcMsXCLrpsKPIcOJp CTrxCLndSLvXcMvXcMvXcMw@RKuxBLv@cMvXcMuTsMvpsKPIcOJpCTrxCLndsLwTCHmDcLnPCMq DcLwdCMtpsKPIcOJpCTrxCLndSMxLsLsLsLsLsLs@RKq`cKtdCLq@sLwPCMqpsKPIcOJpCTrxCL ndsMyDcMvXcMvXcMw@RKrHcKtTSLudsMp`cLqpsKPIcOJpCTrxSL`tbLsxBNrdcMt`CNsdcL||B TrxcB||BToqV^rPfOJpsKIuvYrPfOJpsKOIfZrPfOJpsSbifLdABUyAWY}HrPuIg]eICYb@RZdu cHpqv[t}dXjUvXtMcH`Lt[nMWZsQWYnQWObDcH`XUZsefXlUVObDcH`@u[iyF]SQW^lUVObXTZl qVYdMTZrMF[eMgH`pTZnUvUiQF]hucHpxrLuHBHLef[eME]yqVY}HrToqVZdIBHUuTYsaVObPSN b@BSiyVYC}F[oIGUyAWY}HbQlEF]b@BSiyVYC}F[oIWObLBLp`CLp@cH`pTZnUv\Vev\iIF[euc HqHBHDev\c}f[tef[ueF]yMUYaIwXhucHqHbO|TD^pIGHOAG]}HRUNEV[eIbOqEC^||RQxAg\~h @OEaG\rArSpQWObTUSiyfH~@COoTD^pIgOJpSQxAg\`|D\tucHUuTXxIbOqpsKEaG\rycB|TD^p IGHOAG]}HBVFUg[cQWZoyfH~DWLxqsKEaG\rycB|TD^pIGHOAG]}HRVFUg[cQWZoyfH~HsLjLv[ saRMnTSYmDcJPeTJjLWZnaRLq|bLj@URjDWLxeBHm@rLjLv[sabLnTSYmDcJPeTJjLWZnaRMoHc JPedJqEC^ipsKEaG\rycB|dT[gICY~pCToqV^rPFHFeF[lUFY}HBLb@rPl}v\eQVOb@cH~pCTrx CL`@COo@eL~h@OPIcOpxBLr@CNsLsLsLsLsLsL`tRLnXSLrdCMvTCNqDSM||BTrxcB|@eL~@cKp PSLvXcMvXcMvXcMw@RKsxBLuPSLy\SMpLsMypsKPIcOJpCTrxCLn@cMrTCHmPcKq\sLqHSNwTSM rHCOo@eL~h@OPIcOpxBLxLsLsLsLsLsLsLsL`tBMn`SMxTCNyLsMsLsM||BTrxcB|@eL~@cKq@C MqXcMvXcMvXsM`tRMn@SMrHsMw\cMsXcL||BTrxcB|@eL~@cKqHSM`tBMn\SMuPsLtdSNuTCN|| BTrxcB|@eL~@cKqPSMxLsLsLsLsLsL`tBMn@cLwdCNtPcMp`SN||BTrxcB|@eL~@cKqXcMvXcMv XcMvXsM`tbLndCNpHcMwDsLyTcL||BTrxcB|@eL~@cKq`sMu@RKqxrMu`CMt@cMuDsLqpsKPIcO JpCTrxCLnHCLxLsLsLsLsLsLs@RKpxRMrTCMrXSNuXSNwpsKPIcOJpCTrxCLnHcLyDcMvXcMvXc Mw@BLnTcMpDCMpTSMtLcL||BTrxcB|@eL~@cKrTCHqxrLvPcLv\sLvHSMxpsKPIcOJpCTrxCLnH sMp`sLsLsLsLsLs@RLn\SNtXsLvPcLrDSL||BTrxcB|@eL~@cKrdSLvXcMvXcMvXsM`DcKxDcLq TCMvDCLw\COo@eL~h@OPIcOpxrLqHSM`DcKtLSMuLCMu`SMuXCOo@eL~h@OPIcOpxrLsLsLsLsL sLsLsLCHpxrMs`sLsXSLwXSLxLCOo@eL~h@OPIcOpxrLuPSLvXcMvXcMv\CHm@cKqXSLqLCNtXS Nr@cL||BTrxcB|@eL~@cKs\SM`tRLnDSLu\CNrXcLrTCN||BTrxcB|@eL~@cKsdSMxLsLsLsLsL sL`tRLndcMyLSNqdcLuXSL||BTrxcB|@eL~@cKtDcMvXcMvXcMvXsM`tbLnTsMwTSNqXSMv\sL| |BTrxcB|@eL~@cKtLsMu@RKrxBNr\cLw\SMt\SLrpsKPIcOJpCTrxCLnPSMxLsLsLsLsLsLs@RK rxbMuHCLuLSLrDCLypsKPIcOJpCTrxCLnPsMyDcMvXcMvXcMw@RKrxBLtDcMudcMpDCLqpsKPIc OJpCTrxCLnTCHmDcKpPCMqXSMpLsLyTCOo@eL~h@OPIcOpxRMr@CNsLsLsLsLsLCHpxbLsdsLyD sLrdCMrXCOo@eL~h@OPIcOpxRMtDcMvXcMvXcMv\CHqxbMv\cLsPCMq@SNtpsKPIcOJpCTrxCLn TcMrTCHsxBLwPCMqdSLu@cLvpsKPIcOJpCTrxCLnTCNsLsLsLsLsLsLs@BMnHSNsPSNqLcLy@CN ||BTrxcB|@eL~@cKv@CMqXcMvXcMvXsM`TcKq\SMv@sMx@CNu`COo@eL~h@OPIcOpxbMrTCHuxb MpdCMq`CLtLSLupsKPIcOJpCTrxCLnXCMu`sLsLsLsLsLs@RMnTsLuHSNq\CNtPSL||BTrxcB|@ eL~@cKvXcMvXcMvXcMvXsM`PcKyTsLp\CLs`CMsdCOo@eL~h@OPIcOpxbMx\SM`LcKyHcLsPSMr @sLr\COo@eL~h@OPIcOpxrMp`sLsLsLsLsLsLCHrxRMuTcLr@SMsdsLvpsKPIcOJpCTrxCLn\cL yDcMvXcMvXcMw@RLn@CLrPsMx@SMsLSM||BTrxcB|@eL~@cKwTCHm@cKuXSMpdCNpPCMr`cL||B TrxcB|@eL~@cKw\CLxLsLsLsLsLsL`tRLndsMw`SMxXSLsLSM||BTrxcB|@eL~@cKwdSLvXcMvX cMvXsM`trLn@CNxHCLpTCNtXCM||BTrxcB|@eL~@cKxDcLu@RKsxrMx`SMyLSLsDCMvpsKPIcOJ pCTrxCLn`sLsLsLsLsLsLsLs@RKtxBLrPCMsHSLwLCMwpsKPIcOJpCTrxCLn`SMtDcMvXcMvXcM w@RKsxBNp@SLp@cLuHsLrpsKPIcOJpCTrxCLn`sMu@RKsxRLw\CMx@CLw`CLvpsKPIcOJpCTrxC Ln`SNu`sLsLsLsLsLs@RKrxbLv\SLsXSNyHCN||BTrxcB|@eL~@cKyDcMvXcMvXcMvXsM`tRLnH SLsLcLtHSNtDcM||BTrxcB|@eL~@cKyLsMu@RKpxRLwPsMuTSMpLCMy\COo@eL~h@OPIcOpxRNu `sLsLsLsLsLsLCHpxbMyXsLwDSNx`SLx`COo@eL~h@OPIcOpxRNwdSLvXcMvXcMv\CHqxbLwPsL yXSLqXsLvpsKPIcOJpCTrxSL`DcKt\cMv\cLsTcLs\COo@eL~h@Oo@u[legLdycB||RRm]fLdyc B||rSbifLdycB||dXjICY`PU^pUVObLT]rYWYrPfH`dFY}HB\l}F]OIfZeMF]tHBHC}f[sev\tU f[tucHqHBHVev\iIF[eucHqHBHP}VZnQwTteG[eucHFeF[lUFYCef\cqVYsIBHLef[e]UZdQGZ} HBLnHcH`pTZnUvTteG[eucHS}F[iQfH`TUSeMGZ}HBMyHBHLef[eMt[l}f\TeG\eucHFqVXtIBH Lef[eMt[l}f\}HrHFYdQFACLb@BSiyVYsYUZsefXlUVObDcH`PTZsMv[nQWZnUWZtewTeEf\caV ObDcH~pSQxAg\`|D\tucHUyTXmUfH~DWLxqsKEaG\rycB|TD^pIGHOAG]}HRUMef[bxCL||RQxA g\~h@OEaG\rArSpQWObTUSaagH~DCOoTD^pIgOJpSQxAg\`|D\tucHXYT]nMF]i}f[bxS\q`GOo TD^pIgOJpSQxAg\`|D\tucHYYT]nMF]i}f[bxcLshrXoMGJqxRLj@URihr\iyFJqDsKrhBTIiR\ q`WJ||RQxAg\~h@OIuvYrPfO|@u[legLdAbQiqF[eQVOb@cH`LD[oMWYducHpHbO|@eL~@CHpps KPIcOJpCTrxCLn@cLp`sLsLsLsLsLsLCHm\cKw@SMrHsLq\SNqPCOo@eL~h@OPIcOpxBLtDcMvX cMvXcMvXsM`tRLtxBMrHsMr`CLx@sM||BTrxcB|@eL~@cKpXcLu@RKqdcKrdSLtDCLr@cLspsKP IcOJpCTrxCLn@CNsLsLsLsLsLsLsLCHmHSLnXCNwDcMrHCLwpsKPIcOJpCTrxCLnDCLtDcMvXcM vXcMw@RKrDcKs@cLx\sMp`SMypsKPIcOJpCTrxCLnDcLu@RKq`cKq`sMxDSMvLcMspsKPIcOJpC TrxCLnDCMu`sLsLsLsLsLs@RKqHcKwPSLrdSLxDSLwpsKPIcOJpCTrxCLnDcMvXcMvXcMvXcMw@ RKuxbMvDCMxTCMpTCNxpsKPIcOJpCTrxCLnDCNwTCHrxRLtPCLuXsLr@CMtpsKPIcOJpCTrxCLn HCLxLsLsLsLsLsLs@RNnXsMt\SMuPCMq@CN||BTrxcB|@eL~@cKrHSNqXcMvXcMvXsM`DSMndcM uHcMuTSNtpsKPIcOJpCTrxCLnHSM`HCLnHCLyHSLwdCMrLCOo@eL~h@OPIcOpxbLw@CNsLsLsLs LsLCHrDcKxXcLu`CNq@CLwpsKPIcOJpCTrxCLnHSNqXcMvXcMvXcMw@bLpxrMqLCMsLcMqLCOo@ eL~h@OPIcOpxrLqHSM`DcMndCLy@cMrPcMsHCOo@eL~h@OPIcOpxrLsLsLsLsLsLsLsLCHq@cKy LsMqPSNyLsMtpsKPIcOJpCTrxCLnLSMtDcMvXcMvXcMw@rLnTcMsHcLtPsLsHCN||BTrxcB|@eL ~@cKs\SM`tBMnHcMwPcMtHCLvPSM||BTrxcB|@eL~@cKsdSMxLsLsLsLsLsL`tRLqxRMuDSLqPS MqdCM||BTrxcB|@eL~@cKtDcMvXcMvXcMvXsM`tRLwxrLuPCLt`CNwHcL||BTrxcB|@eL~@cKtL sMu@RKr@cKyLcLsdSNxXcMqpsKPIcOJpCTrxCLnPSMxLsLsLsLsLsLs@RKrDcKxHsMtXSMsDSNw psKPIcOJpCTrxCLnPsMyDcMvXcMvXcMw@RKqdcKyHCMu@CNtTSLtpsKPIcOJpCTrxCLnTCHmDSM nPcMwPcMu\sMuHCOo@eL~h@OPIcOpxRMr@CNsLsLsLsLsLCHmdcKpHsMv\sMsLCLs\COo@eL~h@ OPIcOpxRMtDcMvXcMvXcMv\CHmDcKtLCLvPsMvXSLuLCOo@eL~h@OPIcOpxRMvHSM`XcKsPSNw\ CMp\SNsHCOo@eL~h@OPIcOpxRMxLsLsLsLsLsLsLCHqLcKsDcMrLCLpTCLspsKPIcOJpCTrxCLn XCLtDcMvXcMvXcMw@RLxxRMwTsMpLSLwLCOo@eL~h@OPIcOpxbMrTCHrDcKtTsLydSLsLcMspsK PIcOJpCTrxCLnXCMu`sLsLsLsLsLs@bLqxRMxHSLsHSLqTSM||BTrxcB|@eL~@cKvXcMvXcMvXc MvXsM`DCNndCMsXSNyLCNqXCOo@eL~h@OPIcOpxbMx\SM`DsLn`sMvdCLxdsLvLCOo@eL~h@OPI cOpxrMp`sLsLsLsLsLsLCHwxBLsDcLvLcLuTcMspsKPIcOJpCTrxCLn\cLyDcMvXcMvXcMw@RKp xrMqTsMp\CLrdcLwHCOo@eL~h@OPIcOpxrMu@RKxxrLw@SNsHSLuXcMwpsKPIcOJpCTrxCLn\sM p`sLsLsLsLsLs@RKqPcKyTsLq@sLp@cMupsKPIcOJpCTrxCLn\SNqXcMvXcMvXcMw@RKqdcKvDC NtXsLrdsMxpsKPIcOJpCTrxCLn`SLrTCHmHSLn\cMxdcMyDCMrpsKPIcOJpCTrxCLn`sLsLsLsL sLsLsLs@RKrDcKqHCNyTSLq`SL||BTrxcB|@eL~@cKxTCMqXcMvXcMvXsM`tRLwxrMx@CMuHSLs DCOo@eL~h@OPIcOpxBNwTCHmDcLnDSMr\CLy`sMxXCOo@eL~h@OPIcOpxBNyTCNsLsLsLsLsLCH mPcKyXsMqLCMr\cLpXCOo@eL~h@OPIcOpxRNqXcMvXcMvXcMv\CHrxBNuTSLvdCLw@SLtpsKPIc OJpCTrxCLndsLwTCHq@cKsDSLt\sLu`SLspsKPIcOJpCTrxCLndSMxLsLsLsLsLsLs@RLvxBMtT SNvdCMpXcM||BTrxcB|@eL~@cKy\SNqXcMvXcMvXsM`HCLnPsMrHCNv`SNvTCOo@eL~h@OPIcOq @bLqxBNwPcLydCNwPCN||BTrxcB||BToqV^rPfOJpsKIuvYrPfOJpsKOIfZrPfOJpsSbifLdABU yAWY}HrPuIg]eICYb@RZducHpqv[t}dXjUvXtUcH`Lt[nMWZsQWYnQWObDcH`XUZsefXlUVObDc H`@u[iyF]SQW^lUVObXTZlqVYdMTZrMF[eMgH`pTZnUvUiQF]hucHpxbLb@BSiyVYSQW^lUVObL u[leFYb@RUMUv\hucHtdcH`pTZnUvPoqv[rQU^pUVObXD[aQgH`pTZnUvPoqv[rucHc`CLx@CLp HBHLef[eMgUiMWZbqVY}HRLb@BQiMwXoyF]iyV]iQW^SUVXrMFZ}HRLbxCOEaG\rArSpQWObTeS auVYbxS\q`GOoTD^pIgOJpSQxAg\`|D\tucHUuTZnIbOppsKEaG\rycB|TD^pIGHOAG]}HRUMEF ^bxSL||RQxAg\~h@OEaG\rArSpQWOb`eQuyvXtev[nIbOqEC^||RQxAg\~h@OEaG\rArSpQWObd eQuyvXtev[nIbOrLcJc}v\h`cKrTSYmDcJPeTJjLWZnaRLq|bLj@URjDWLxeBHm@rLjLv[sarLn \SMeuRLj@URihr\iyFJu|bLj@URjDWLxeBOoTD^pIgOJpSRm]fLdyCOP}F[yICY`XTZlqVYducH pHBHCqv[sUFY}HBLbxCOPIcOp@BL||BTrxcB|@eL~@cKpHCLxLsLsLsLsLsLs@RKwxBLyPCNy@S Mx\SNupsKPIcOJpCTrxCLn@CMqXcMvXcMvXcMv\CHmDsLnHSNyHsMwLcLyHCOo@eL~h@OPIcOpx BLvHSM`tRLwxBNsXsLpDCMwPsM||BTrxcB|@eL~@cKp`sLsLsLsLsLsLsLs@RKr@cKqPSLxPCLp TSNqpsKPIcOJpCTrxCLnDCLtDcMvXcMvXcMw@RKqdcKyLcMsTsLq@SLypsKPIcOJpCTrxCLnDcL u@RKq\cKrXCLr`SNxLsLypsKPIcOJpCTrxCLnDCMu`sLsLsLsLsLs@RKqHcKtXCNuHcMw@cMrps KPIcOJpCTrxCLnDcMvXcMvXcMvXcMw@RKvxRLxPSMv@cLsPsLypsKPIcOJpCTrxCLnDCNwTCHpx rMwdcMvPSNtLcMpHCOo@eL~h@OPIcOpxbLp`sLsLsLsLsLsLCHwxRMr`CLpdSLpDcMxpsKPIcOJ pCTrxCLnHcLyDcMvXcMvXcMw@RLsxRLyTSLpHSNwDsL||BTrxcB|@eL~@cKrTCHq\cKpTsMr`cM qPcMtpsKPIcOJpCTrxCLnHsMp`sLsLsLsLsLs@RLxxbMrTcLy\SLsXcL||BTrxcB|@eL~@cKrdS LvXcMvXcMvXsM`DsMn\CLv`sLuHSNtLCOo@eL~h@OPIcOpxrLqHSM`DCMnPsLpdsMy@CNq\COo@ eL~h@OPIcOpxrLsLsLsLsLsLsLsLCHyxbLsDsLsXsMtDSMrpsKPIcOJpCTrxCLnLSMtDcMvXcMv XcMw@bLn\SNp@SNwLcMqXSL||BTrxcB|@eL~@cKs\SM`tBMn@CMy`sLudSLyLSN||BTrxcB|@eL ~@cKsdSMxLsLsLsLsLsL`tRLpxrLyLsLuTSNpdCOo@eL~h@OPIcOpxBMqXcMvXcMvXcMv\CHmDS MnPCLxLCNrTCNr\COo@eL~h@OPIcOpxBMs\SM`tRLxxBMsLCLsDSLx`cL||BTrxcB|@eL~@cKtT CNsLsLsLsLsLsL`tRLyxBLv@SNvXCLuLCOo@eL~h@OPIcOpxBMwdSLvXcMvXcMv\CHmDsMnDSNs dCNuDsMxdCOo@eL~h@OPIcOpxRM`tRLsxBLuTCLxDcMq`sM||BTrxcB|@eL~@cKuHCLxLsLsLsL sLsL`trMnDcMpHSNtXCNu`CN||BTrxcB|@eL~@cKuPSLvXcMvXcMvXsM`tBLnHSMrdCMwXCNtLS LvpsKPIcOJpCTrxCLnTcMrTCHvxrMyDsLp`cLudSNypsKPIcOJpCTrxCLnTCNsLsLsLsLsLsLs@ RLsxBLwXCMx@sMydSN||BTrxcB|@eL~@cKv@CMqXcMvXcMvXsM`DsMn`CLpdCLu@sLrpsKPIcOJ pCTrxCLnXcLu@bLpxrLudSNp@CLuPcL||BTrxcB|@eL~@cKvPSMxLsLsLsLsLsL`HCLnPcLsHSM vdsLrLCOo@eL~h@OPIcOpxbMvXcMvXcMvXcMv\CHq\cKy\sMvHSMw@cLspsKPIcOJpCTrxCLnXC NwTCHqLcKsHCNsXcMsPCMtpsKPIcOJpCTrxCLn\CLxLsLsLsLsLsLs@rMn@cMpXcLr\cLu@SL|| BTrxcB|@eL~@cKwHSNqXcMvXcMvXsM`tBLn@sLuLSNvPCMr\sLqdCOo@eL~h@OPIcOpxrMu@RKw xBLvTsLudcLudSLspsKPIcOJpCTrxCLn\sMp`sLsLsLsLsLs@RKqLcKqPSMpPCMxPcL||BTrxcB |@eL~@cKwdSLvXcMvXcMvXsM`tRLwxRMqLcLs\SMq`cL||BTrxcB|@eL~@cKxDcLu@RKqdcKvHC NxHSLsdcLqpsKPIcOJpCTrxCLn`sLsLsLsLsLsLsLs@RKqdcKrLSNvPSLxTsLqpsKPIcOJpCTrx CLn`SMtDcMvXcMvXcMw@RKqXcKtDCMsHCLrLSMspsKPIcOJpCTrxCLn`sMu@RKqDcKuLcLyTcMy TSLspsKPIcOJpCTrxCLn`SNu`sLsLsLsLsLs@RKuxbLs\SNr`cLqXCNrpsKPIcOJpCTrxCLndSL vXcMvXcMvXcMw@RLnXCMxdCLsTcMsXCN||BTrxcB|@eL~@cKyLsMu@BNnHsLqdCMqHcLwHSL||B TrxcB|@eL~@cKyTCNsLsLsLsLsLsL`DsLnXSMvdSNyLsMtdCOo@eL~h@OPIcOpxRNwdSLvXcMvX cMv\CHq\cKrHSLp\cMtPsLspsKPIcOJpCTrxSL`DCNnPcMrXsMsPCNspsKPIcOJpsKP}F[yICY~ h@OodT[gICY~h@Oo|dXjICY~h@OVeVYwef[gIt[xaUSiyFHVEF[}HRKqTVKpDCLbxSKqxBLeuRL ppsKVeVYwef[gIt[xaUSiyfOJpcUiUv]iyvYB}F^XuTXxAbUaqVObDcKp@CLp@CLp@CLqHbOqxB L||bUiUv]iyvYB}F^XuTXxycB|XUZe]WZn]fPoaWVMef[`XUXlucHmHcMn@CLp@CLp@CLuHbOmH cKv@CLp@CLp@SLeECOoXUZe]WZn]fPoaWVMef[~h@OVeVYwef[gIt[xeUSaaGHVEF[}HbLtxbLw \cMrDsLq@cLbxcLnPcLw\cMrDsLqTVL||bUiUv]iyvYB}F^YuTXxycB|`TZnQw\`LuXaqVZn]VO bTe[c}f[sQg\aef[eQfHoxCOoLt[oIGYiyVXtUvTyMG]eufLdycB||rTcUf[eICY~h@OoLTXnYW XsycB %%%%%%%%%%%%%%%%%%%%%% End /document/N4QRHX0D.xvz %%%%%%%%%%%%%%%%%%%%% %%%%%%%%%%%%%%%%% Start /document/graphics/Image90.gif %%%%%%%%%%%%%%%% GedQxdSXxEP}@@H@@@@@@|sB@@@@@xEP}@@`@~sxcin\{OLJgtj}bszMo{`abcdefghijkl mnopqrstuvwxyz{|}~@CJ\HqbFObLireL[z|IthRgRmjukXsj]Kwn^BNlxqdK[~LztjWsnm{w pcK_N}zvocOo~}|{{O`AJx`DVhaGbXbJnHcMzxcPFidSRYeV^IfYjyf\vig_BZhbNJieZZE@\j jtdzjnzB@q~zljD[ktbKIvfKobl[o@sQjCG\qXP|mFk\DHolsRd|sRsPtS[}AU_muYkmt\wms_C nrbO^qe[NphgNoksNnn~lqKoktWojwcoizoOi}{oOYkARpBZpCbpDjpErpFzpGBqHJqIBQNujK FPTmi~oEGsHE@XaRfH}xD@i`bFJIItjTuZiJYFe\YBLi|hI_rFN_IDli^riBxYz@z]xFQuyCDZ] JzBPJxXz\TzOMjC\Z\~fUeZChz[nJupZWYdxBkWWJXgjYG{LK[VOKZqfZJ\[[@tzZJg]ek[St] Yk\i[Zv{Zq{\UD`c[[}{YPgbE|`OtbOJcQ\Y^L_OLbh\`qkdICfMLZwlXVghclaF}NI]HayiELh AciaLkKmjamfe}ji]_S}lqmeGd{~MjOyJw]oENqs\n}}po]rMNsk|gIDpO~X[]Wf~scnriNYc}v sngk~wA_x}juuR|R_pczW^sY_IcowAT{yNVO_VjOyoos~_Q]~pg{q[ZG}I_vgTa_Xa~M`bW`l `PWCV`BHDZ^}GCV^|gI|KhgAz_FufmaWQzQQdXIZbgHJfbjxJrbdhU|`HWqiarR~uaWXFVBahBZ cTHNF`GXZA`~ALbaHhEvcWmy`|XLR`hGb@IXoHeoHSVd_xQvdZYEfW{AQv]JiAPePqWfdowX~ DeYPFfMfWZe`yYNdSxUjSiT_`]whQFerIPveoW[zf}iXn^@ZOMgpiLNWDT^`gHIVN^Nj_fhOYdN hBIenefydrgPierfKYgbftbhVeaJaBjyidzdYifRgoDiVEoVejRpJUHkYj^j[QZmnJ|jmBe~ziD lRQgahkJdZ~svjQhnbk^z_J]izpVJRKqFmgJtzhT{hhmdiu^iLkrjjK{ead\[hXn]KnbnBt[elb KWnH\e{zcDVfnn^r}fna[~ZQPBTmfO[}zn|{FEI~D@oiq[rRpZBCkojjCcIQ\ZvjSleXqNa]~nt kdLR[\n^q]IFGS[DqThG{etbj|JsrmlK{bxKcmmrlhLrt|N^sl[NGs{\gXs}|T|s@]THtCME[tY RQctLaRotJQS{tHATGuFqTSuDaU_uBQVku@AWwu~pWCv|`XOvzPY[vx@ZgvvpZsvlr[Gwr}\Swu m]_wx]^kw{M_ww~}_CxAn`OxD^a[xGNbgxJ~bsxMncxP^dKySgxT^e[yWNfgyZ~fsy]ngy`^h KzcNiWzf~iczinjozl^k{zoNkW@@@pN %%%%%%%%%%%%%%%%%% End /document/graphics/Image90.gif %%%%%%%%%%%%%%%%%